Sunteți pe pagina 1din 163

OBIECTIVE DE REFERIN I

EXEMPLE DE ACTIVITI DE NVAARE


1. Cunoaterea i nelegerea conceptelor, a terminologiei i a procedurilor de
calcul
Obiective de referin
La sfritul clasei a VI-a elevul va fi
capabil :

Exemple de activiti de nvare


Pe parcursul clasei a VI-a se recomand
urmtoarele activiti :

1.1.s utilizeze noiuni de logica


1.2.s foloseasc metode i principii
adecvate n rezolvarea unor probleme

-rezolvarea unor probleme de logica


-rezolvarea unor probleme folosind
principiul paritii sau partiia n clase
- folosirea metodei reducerii la absurd n
anumite demonstraii
-probleme care se rezolv folosind principiul
lui Diriclet
-probleme care se rezolv folosind principiul
invariantului
-rezolvarea de probleme folosind regula de
trei simpl sau regul de trei compus
-probleme de ordonare prin comparare
-probleme de numrare
-rezolvarea unor ecuaii i inecuaii dificile
folosind diverse tehnici
-rezolvarea unor probleme cu ajutorul
ecuaiilor
-formularea unor probleme pornind de la o
ecuaie
-exerciii de recunoatere a unor numere
naturale sau expresii divizibile cu alte
numere date folosind criteriile de
divizibilitate sau descompunerile n factori
-exerciii de calcul a numrului de divizori a
unui numr folosind descompunerea n
produs de puteri de numere prime
-calculul unor sume folosind diverse tehnici
-exerciii de determinare a valorii unor
expresii
-calculul unor probabiliti
- rezolvarea unor probleme de micare care
i au originea n fizic folosind relaiile ce
se stabilesc ntre mrimi : proporionalitate
direct sau invers
-probleme cu coninut practic care se rezolv
folosind reducerea la scar

1.3.s foloseasc diferite metode de


rezolvare a ecuaiilor i inecuaiilor i s
utilizeze ecuaii i inecuaii pentru
rezolvarea problemelor
1.4.s aplice criterii, proprieti i noiuni
de divizibilitate n demonstraii

1.5.s efectueze calcule cu numere ntregi


i raionale pozitive
1.6.s utilizeze matematica n rezolvarea
problemelor puse la alte discipline

-probleme de numrare i interpretarea lor


folosind noiunea de probabilitate
-rezolvarea problemelor cu procente
-rezolvarea problemelor de coliniaritate i
concuren
-probleme de construcii geometrice
-rezolvarea de probleme folosind metoda
triunghiurilor congruente
-folosirea criteriilor de paralelism n
rezolvarea unor probleme
-folosirea n demonstraii a proprietilor
triunghiului isoscel i a triunghiului
echilateral

1.7.s foloseasc metode specifice n


rezolvarea problemelor de geometrie

1.8.s recunoasc i s utilizeze


proprietile figurilor geometrice n
demonstraii

2.Dezvoltarea capacitii de a emite judecai de valoare pentru rezolvarea


problemelor inventiv i euristic-creative
Obiective de referin
La sfritul clasei a VI-a elevul va fi capabil

Exemple de activiti de nvare


Pe parcursul clasei a VI-a se
recomand urmtoarele activiti :

2.1.s analizeze, s elaboreze strategii de


rezolvare i s rezolve probleme dificile

-analizarea problemei n scopul


nelegerii ei
-elaborarea unui plan de rezolvare i
rezolvarea problemei
-verificarea rezultatului obinut i
analiza rezolvrii
-formularea unor concluzii pornind de
la o ipoteza data
-deducerea unor condiii necesare i
suficiente pentru demonstrarea unei
concluzii
-identificarea unor algoritmi de
rezolvare valabili pentru clase de
probleme
-analizarea mai multor metode de
rezolvare i alegerea celei mai eficiente

2.2.s formuleze probleme pornind de la un


model sau enun parial

2.3.s gseasc metode de lucru valabile pentru


clase de probleme

3.Dezvoltarea capacitii de a face conexiuni cognitive n cadrul disciplinei i a


ariei curriculare
Obiective de referin
La sfritul clasei a VI-a elevul va fi
capabil :
3.1.s utilizeze raionamente inductive
n rezolvarea problemelor dificile din
domeniile studiate
3.2.s-i nsueasc o gndire flexibil
i abstract specific matematicii

Exemple de activiti de nvare


Pe parcursul clasei a VI-a se recomand
urmtoarele activiti :
-exerciii i probleme n rezolvarea crora se
folosesc diferite raionamente
-folosirea soluiilor unei probleme pentru
rezolvarea altora din aceeai sfer cognitiv
-probleme din algebr care se rezolv cu
metode geometrice sau probleme de geometrie
care se rezolv algebric

4.Dezvoltarea capacitii de a comunica utiliznd limbajul matematic


Obiective de referin
La sfritul clasei a VI-a elevul va fi
capabil :

Exemple de activiti de nvare


Pe parcursul clasei a VI-a se recomanda
urmatoarele activitat

4.1.s diferenieze informaiile matematice


dintr-un enun dup natura lor

-sesizarea informaiilor cu caracter general


dintr-o ipoteza
-redactarea demonstraiilor sau rezolvrilor
utiliznd terminologia adecvat
-formularea de propoziii reciproce,
analizarea lor i stabilirea valorii lor de
adevr

4.2.s formuleze reciproce ale unor


propoziii i s studieze valoarea lor de
adevr

5.Dezvoltarea interesului i a motivaiei pentru studiul i aplicarea matematicii n


contexte variate
Obiective de referin
La sfritul clasei a VI-a elevul va fi
capabil :

Exemple de activiti de nvare


Pe parcursul clasei a VI-a se recomand
urmtoarele activiti

5.1.s sesizeze importanta aplicrii


noiunilor de matematica n probleme cu
coninut practic
5.2.s manifeste originalitate n abordarea
unor metode alternative de rezolvare
5.3 s manifeste interes pentru folosirea
tehnologiilor informaiei n studiul
matematicii

-argumentarea prin exemplificare


-utilizarea unor metode variate n rezolvarea
unei probleme
-utilizarea unor soft-uri pentru nvarea
matematicii

CONINUTURI
ALGEBR
1. Sume
2. Divizibilitatea n mulimea numerelor naturale
3. Cteva principii i metode de rezolvare a problemelor de matematic
3.1. Principiul paritii
3.2. Probleme de numrare
3.3. Principiul lui Dirichlet
3.4. Principiul invariantului
3.5. Probleme de logic
3.6. Probleme de ordonare
3.7. Metoda reducerii la absurd
4. Rapoarte i proporii
4.1. Scara unui plan
4.2. Scara unei hri
4.3. Probabiliti
4.4. Procente
4.5. Titlul unui aliaj
4.6. Proporii
4.7. ir de rapoarte egale
4.8. Proporionalitate direct. Proporionalitate invers
4.9. Regula de trei simpl. Regula de trei compus
5. Numere ntregi
5.1. Divizibilitate n mulimea numerelor ntregi
5.2. Determinarea valorii unei expresii ce depinde de un exponent natural
5.3. Ecuaii i inecuaii
GEOMETRIE
1. Segmente
2. Unghiuri
3. Geometria bazata pe raionament i demonstraie
3.1. Cazurile de congruen ale triunghiurilor
3.2. Metoda triunghiurilor congruente
3.3. Proprietile triunghiului isoscel i echilateral
3.4. Paralelism
3.5. Patrulatere
3.6. Concurena liniilor importante n triunghi
3.7. Probleme de coliniaritate
3.8. Probleme de concuren
3.9. Construcii geometrice

1. Calculul unor sume de numere


n multe probleme elevii aplic n rezolvarea lor calculul unor sume de numere
naturale consecutive, numere pare consecutive, numere impare consecutive, dar i
sume de numere raionale pozitive.
Parcurgnd aceast tem se face o pregtire pentru nelegerea ulterioar a
demonstraiei relative la calculul unor sume de numere folosind metoda induciei
matematice.
1.1.Introducerea simbolului sum i a proprietilor lui
n matematic pentru prescurtarea scrierii unor sume se folosete simbolul .
n

Prin

a
k =1

nelegem sum de ak de la k =1 pn la k = n.

Prezentm n continuare cteva exemple de folosire a acestui simbol:


a) Suma primelor n numere naturale:
n

1 + 2 + 3 + .. + n se scrie

k =1

b) Suma ptratelor primelor numere naturale:


12 + 22 + 32 + + n2 se scrie

k =1

c) Suma cuburilor primelor n numere naturale se scrie:


13 + 23 + 33 + .. + n3 se scrie

k =1

Alte exemple de utilizare a simbolului sum:


p

d)

= 1 + 1 + .. + 1 = p

k =1

p termeni
q

e)

(1)2i = 1 + 1 + 1 + .. + 1 = q p

i=p

de q p ori
n

f)

a
k =1

= a1 + a2 + + an

1.2. Proprieti ale simbolului sum


1. Suma unei sume (diferene) este egal cu suma (diferena) sumelor:
n

k =1

(ak bk) =

k =1

ak

bk

k =1

11

2. Dac toi termenii sumei conin acelai factor el poate fi scos ca factor comun
n afara sumei:
n

ak =

k =1

k =1

ak,

Probleme rezolvate
R1.3.1.Suma primelor n numere naturale se calculeaz dup formula:
n

k=

k =1

n (n + 1)
2

(1)

Demonstraie:
n

k = 1 + 2 + 3 . + (n 1) + n

k = n + (n 1) + (n 2) + . + 2 + 1

k =1
n
k =1

k = (n + 1) + (n + 1) + . + (n + 1)

k = n(n + 1) : 2

k =1
n

k =1

k=

k =1

n (n + 1)
2

R1.3.2. Suma ptratelor primelor n numere naturale este dat de formula:


n

k =1

k2 =

n (n + 1)(2n + 1)
6

(2)

Demonstraie:
Calculm mai nti suma:
n

(2k 1) = 1 + 3 + 5 + . + 2n 1

k =1

innd seama de formula (1) i de proprietile simbolului sum avem:


n

k =1

(2k 1) =

k =1

2k +

(1) = 2

k =1

k+

k =1

= n(n + 1) n = n2 + n n = n2
Aadar pentru orice k 1, avem:
1 + 3 + 5 + . + (2k 1) = k2

k =1

(3)

12

(1) = 2

n (n + 1)
n =
2

Folosind relaia (3) avem:


1
= 12
1+3
= 22
1+3+5
= 32
.
1 + 3 + 5 + . + (2k 1)
= k2
.
1 + 3 + 5 + . + (2k 1) + . + (2n 1) = n2
n

1n + 3(n 1) + 5(n 2) + . + (2k 1)(n k + 1) + ..+ (2n 1) =

k2

k =1

Relaia precedent se poate scrie prescurtat astfel:


n

(2k 1)(n k + 1) =

k =1

k2 (4)

k =1

innd seama de proprietile sumei relaia (4) se poate scrie:


n

(2k 1)(n k + 1) =

k =1

k =1
n

= (n + 1)

(2k 1) 2

k =1

k =1

(2k 1)(n + 1) +
k2 +

k =1
n

k =1

k = (n + 1) n2 2

k =1

n (n + 1)
=
Deci (n + 1) n2 2 k2 +
2
k =1
n
n (n + 1)

3 k2 = n2(n +1) +
2
k =1
n

( 2k2 + k) =

k =1

k =1

k =1

k2 +

n (n + 1)
2

k2

n (n + 1)(2n + 1)
: 3 de unde
2
n (n + 1)(2n + 1)
k2 =
6

k2 =

R1.3.3. Numrul triunghiular este un numr de forma

n (n + 1)
, unde n este un
2

numr natural.
Denumirea este justificat pentru c aceste numere pot fi materializate n
triunghiuri dreptunghice alctuite din puncte. Se observ c numerele triunghiulare
se obin prin adunarea succesiv a numerelor din irul natural:
1; 1 + 2 = 3; 1 + 2 + 3 = 6; 1 + 2 + 3 + 4 = 10 ;; 1 + 2 + 3 + . + n =

13

n (n + 1)
2

Trebuie calculat suma:

k =1

k (k + 1) 1
=
2
2

k =1

1
k +
2
2

k =1

innd seama de formulele (1) i (2), obinem:

k (k + 1) 1 n (n + 1)(2n + 1) 1 n (n + 1)
=
+
=
2
2
2
6
2
k =1
n (n + 1) 2n + 1 1 n (n + 1) 2n + 4
+ =

=
=

2
6
2
2
6
n (n + 1) 2(n + 2) n (n + 1)(n + 2)

=
=
6
6
2
n

Bibliografie
D. Constantinescu, Olimpiada de matematic clasele V-VIII, Ed. Teora 1999, pag 2852;pag 125-138
D. Andrica, V. Berinde, Al. Blaga, G.Both, O. Pop, Concursul Grigore Moisil Ed. I-XV,
Ed. Hub Press 22 Baia-Mare 2001, pag 39,45,78
D. Brnzei

i colectivul: Matematica n concursurile colare, Ed. Paralela 45,

2000,2001,2002 pag 27-54,119-135(2000);pag 27-54,117-130(2001);pag 18-34,82-92(2002)


D. Brnzei, D. Zaharia, M. Zaharia : Aritmetic-Algebr-Geometrie, Ed.Paralela 45
2002, pag 5-21
Acad. N. Teodorescu coordonator Culegere de probleme pentru clasele V-VIII, SSM
1987, pag 53-68
Foaia matematic (Chiinu) 3/1996, pag 24-31

14

2 Divizibilitatea n mulimea numerelor naturale


Dintre toate operaiile aritmetice, cea mai capricioas este mprirea.
Ea dispune de proprieti speciale, de un caracter deosebit.
Toate particularitile mpririi au favorizat apariia unor noiuni ca: numere
prime, cel mai mare divizor comun, cel mai mic multiplu comun, criterii de
divizibilitate.
Dezvoltarea teoriei divizibilitii a dus treptat la o serioas extindere a ntregii
teorii a numerelor.
n multe probleme de determinare a unor numere naturale folosim noiunile
studiate la divizibilitatea numerelor.
Reamintim teorema mpririi cu rest i cele mai importante noiuni ale
divizibilitii numerelor.
2.1. Teorema mpririi cu rest
Pentru oricare dou numere naturale a i b cu b 0, exist i sunt unice dou
numere naturale q i r astfel nct a = bq + r i r < b.
a dempritul
b mpritorul
q ctul mpririi
r restul mpririi
Proprietatea 2.1.1. Dac adugm lui a un multiplu a lui b, restul mpririi nu se
schimb.
Fie a = bq + r | + mb
a + mb = bq + r + mb = b(q + m) + r = b q1 + r
Proprietatea 2.1.2. Dac nmulim dempritul i mpritorul cu un numr,
restul se nmulete cu acel numr.
Din a = bq + r | m, obinem
am = bqm + rm, unde r m < mb
Proprietatea 2.1.3. Dac numerele a i b se mpart cu un numr atunci i restul se
mparte cu acel numr.
Fie a = bq + r , r < b
Dac a = m a1 i b = m b1, atunci avem a1 m = b1 m q + r | : m,
a1 = b1 q +

r
m

Proprietatea 2.1.4. Dac dou numere dau acelai rest la mprirea cu un numr m,
diferena lor este divizibil cu m.
Din a = m q1 + r i b = m q2 + r deducem a = m q1 + r
b = m q2 + r
a b = m (q1-q2)

15

2.2. Divizibilitatea n N
Definiia 2.2.1. Numrul natural a este divizibil cu numrul natural b dac
exist numrul natural c astfel nct a = b c
Notm: a M b ( a se divide cu b )
b | a ( b divide pe a )
b este divizorul lui a
a este multiplul lui b
Obs. Numrul natural a este divizibil cu numrul natural b dac restul mpririi
lui a la b este zero.
Proprieti
Propoziia 2.2. 1. Dac a este divizor a lui b i c atunci este divizor i a lui b c.
Din a | b b = m1 a
a | c c = m2 a
nsumnd cele dou egaliti membru cu membru obinem:
b + c = m1a +m2 a = a ( m1+m2 )= m3 a
Scznd cele dou egaliti, rezult c :
b c = m1 a m2a = a( m1-m2 ) = a m3 ( b c)
Propoziia 2.2.2. Dac a este divizor a lui b i c , oricare ar fi numerele naturale x i y,
a va fi divizor i pentru bx + cy.
Din a | b b = m1a
a | c c = m2a
nmulim prima egalitate cu x i a doua cu y i obinem :
bx = m1 a x
cy = m2 a y
Adunm membru cu membru i obinem :
bx + cy = m1a x + m2 a y = a ( m1 x + m2 y ) = m a a | ( bx +cy )
Propoziia 2.2.3. Dac a este divizor a lui b i b divizor a lui c atunci a este
divizor a lui c.
Din a | b b = m1a
b | c c = m2b
nlocuind n egalitatea a doua pe b obinem:
c = m1m2a = m a a | c
Proprietatea 2.2.4. Dac a | b i b | a atunci a = b .
Din a | b b = m1a
b | a a = m2 b
Substituind n prima egalitate pe a obinem :
b = m1m 2 b | : b
1 = m1 m2 ; m1, m2 N m1 = m2 = 1 a = b
Definiia 2.2.2. N umrul natural p , p 2 este prim dac se divide numai cu 1 i cu el
nsui.
1 i p se numesc divizorii mpirii.
16

Obs : 10. Un numr care nu este prim se numete compus.


20. Numrul 2 este singurul numr natural prim i par.
Propoziia 2.2.5.Cel mai mare divizor comun al numerelor naturale a i b este un
numr natural d , care :
divide pe a i b ;
este divizibil cu orice divizor a lui a i b.
Notm : c.m.m. d.c. sau ( a; b)
Obs : 10 . Dac ( a; b) = 1 , atunci numerele a i b se numesc prime ntre ele .
Propoziia 2.2.6. Cel mai mic multiplu comun al numerelor a i b este un numr
natural m , care :
este multiplu a lui a i b ;
orice alt multiplu a lui a i b se divide cu el .
Notaie : c.m.m.m. c. sau [a; b ]
Propoziia 2.2.7. Dac a i b sunt numere naturale atunci avem :
a b = (a ; b)[a; b]
2.3. Determinarea unor numere prime n condiii date
Probleme rezolvate
R2.3.1. Determinai numerele prime a i b tiind c 28 a + 21b =2030.
Soluie: 2030 M 2
21b M 2 ,dar 21 M 2
28a M 2
atunci, b M 2 i b este numr prim atunci b = 2.
nlocuim n egalitatea dat i obinem :
28a+212 = 2030
28a = 2030 42
28a =1988 |: 28
a = 71
Numerele sunt : a = 71 , b = 2 .
R2.3.2. S se gseasc numerele naturale p astfel nct numerele p , p2 + 4 ,
p2 + 6 s fie simultan prime .
Soluie :
( ) p numr natural prim el are una din formele: 5k , 5k + 1 , 5k + 2 , 5k + 3 ,
5k + 4.
Vom demonstra c p are forma 5k i cum p este prim rezult c p = 5 .
Fie p = 5k + 1 p2 = (5k + 1 ) 2 =M5 + 1
P2 +4 =M5+1 + 4 =M5 + 5 = M5 (p2 + 4) M 5
b) p = 5k + 2 p2 = (5k +2 )2 =M5 + 4 p2 +6 = M5 + 10 = M5
( p2 + 6 ) M 5
c) p = 5k + 3 p2 = ( 5k + 3 ) 2 = M5 + 9
p2 +6 = M5+ 9+ 6 = M5 + 15 = M5 (p2 + 6) M 5
d) p = 5k + 4 p2 = ( 5k + 4 ) 2 = M 5 + 16 p2 +4 =
17

= M5+ 16 + 4 = M5 + 20 =M5 (p2 + 4 ) M 5


Din a), b) , c), d) rezlt c p este de forma p =5k i p numr prim atunci p =5 i p2 + 4 =
29 , p2 + 6 =31, deci sunt numere prime.
A doua soluie :
Ultima cifr a lui p poate fi 2 sau cifra impar : 1, 3, 5, 7, 9, atunci ptratul lui
va avea ultima cifr 4, 1, 9, 5 u( p2 ) = 4 u ( p2 + 6 ) = 0
( p2 + 6 ) M 5 ; u ( p2 ) =1 u ( p2 + 4 ) = 5 ( p2 + 4 ) M 5 , u ( p2 ) = 9
u ( p2 + 6 ) =5 ( p2 6 ) M 5 ; u ( p2 ) = 5 u ( p ) = 5 i p este prim p=5.
R2.3.3. S se determine toate numerele naturale n i p pentru care numerele:
p , p+ 3n , p+ 3n+1. p + 3n +2. p+3n +3 sunt prime.
Soluie:
Dac p este numr impar atunci numerele p + 3n , p + 3n + 1 , p+ 3n + 2 ,
n +3
p + 3 sunt numere pare , deci nu sunt prime rezult c p este numr par i prim deci
p =2
Ultima cifr a puterilor consecutive a lui 3 poate fi : 1,3,7,9 atunci unul dintre
numerele p + 3n, p + 3n+1, p + 3n +2 sau p+ 3n +3 va avea ultima cifr 5 deci va fi divizibil
cu 5 i atunci nu va fi prim dect n cazul n care este egal cu 5.
A tunci : p + 3n =5 2 + 3n = 5 3n =3 n =1
p =2 ; p + 3n =5 ; p + 3n+1= 11 ; p +3n+2 = 29 i p + 3n +3 = 83 sunt numere
prime .
Dac p+ 3n+1 = 5 3n+1 = 3 n =0 , atunci avem :
p=2
p + 3n =3
p + 3n+1= 5
p + 3n+2= 11
p + 3n+3 = 29 sunt numere prime .
Dac p + 3n+2 = 5 3n+2 = 3 imposibil.
Soluiile sunt : 1) p = 2 i
2) p = 2
n=0
n=1
2.4.Probleme care se rezolv folosind teorema mpririi cu rest, cel mai mare
divizor comun i cel mai mic multiplu comun
Probleme rezolvate
R2.4.1. Determinai cel mai mic numr natural care mprit la numerele
naturale a, b, c d resturile a k ; b k ; c k , k N* i k < min(a,b,c).
Soluie:
Fie n numrul cutat, atunci avem:
n = ac1 + a k
n + k = a ( c1 + 1 ) = Ma
n + k = b ( c2 + 1 ) = Mb

n = bc2 + b k + k
n = cc3 + c k
n + k = c ( c3 + 1 ) = Mc

18

n + k este multiplu comun al numerelor a, b, c, i pentru c este cel mai mic rezult c
n + k =[a, b, c] n = [a,b,c] k.
n condiiile n care n1 n n2 vom determina multiplii comuni care
ndeplinesc condiia dat , apoi calculm numrul n.
Exemplu: Aflai cel mai mic numr natural care mprit pe rnd la
5,6,7,8, d resturile 4,5,6,7.
Soluie :
Fie n numrul , atunci:
n = c15 + 4
n + 1 = 5 ( c1 + 1 ) = M5
n = c2 6 + 5
n + 1 = 6 ( c2 + 1 ) = M6
n = c37 + 6
+1
n + 1 = 7 ( c3 + 1 ) = M7

n +1 = 8 ( c4 + 1 ) = M8
n = c48 + 7
n + 1 multiplu comun al numerelor 5,6,7,8 i pentru c este cel mai mic rezult c n + 1
=[5,6,7,8] n + 1 = 840 n = 839 .
n cazul n care se impune condiia ca n s fie cuprins spre exemplu ntre 800 i
2003 atunci n + 1 { 840; 2840; 3840 } problema avnd trei soluii distincte.
R2.4.2. Determinai cel mai mic numr natural care mprit la numerele naturale a, b,
c obinem de fiecare dat restul r , r < min (a,b,c) .
Soluie:
Fie n numrul care trebuie determinat :
n = a c1 + r
n r = ac1 = Ma
n = b c2 + r
-r
n r = bc2 = Mb

n r =c c3 = Mc
n = c c3 + r
nr
n r este multiplu comun al numerelor a, b, c i pentru c este cel mai mic
= [a,b,c] n = [a, b,c] + r.
Dac asupra lui n se impune o condiie vom considera toi multiplii comuni
care ndeplinesc condiia pentru a determina numrul n .
Exemplu: Determinai numerele naturale cuprinse ntre 1200 i 5200 care
mprite la 20 ;28 ;36 s dea de fiecare dat restul 5.
Soluie:
Fie n numrul, atunci avem :
n = 20c1 + 5
n 5 = 20 c1 = M20
-5

n 5 = 28 c2 = M28

n = 28c2 + 5
n = 36c3 + 5
n 5 = 36 c3 = M36
n 5 este multiplu comun al numerelor 20;28; 36.
Aflm c.m.m.m.c al numerelor [20;28;36] =1260
n 5 {1260;12602;12603;12604}
n 5 = 1260 n = 1265
n 5 = 2520 n = 2525
n 5 = 3780 n = 3785
n 5 = 5040 n = 5045
Problema are patru soluii: 1265 ; 2525 ; 3785 i 5045

19

R2.4.3.Numerele a,b,c mprite la acelai numr natural dau resturile r1, r2 , r3 . S se


afle numrul la care au fost mprite.
Soluie:
Fie n mpritorul, n < min (a;b; c)
a = nc1 + r1 | -r1
a - r1 = nc1
n | a-r1

b r2 = nc2
n | b-r2
b = nc2 + r2 | -r2
c r3 = nc3
n | c-r3
c = nc3 + r3 | -r3
n este divizor comun al numerelor a r1 , b r2 , c r3 , i n >max (r1;r2;r3)
Aflm cc.m.m.d.c. al numerelor a- r1; b- r2 ; c- r3 ; i lum pentru n valorile celui mai
mare divizor comun i divizorii si mai mari dect max (r1,r2 ,r3).
Exemplu: Numerele 1333 i 351 dau resturile 13 i respectiv 15 la mprirea
cu acelai numr natural diferit de zero. Aflai acest numr.
Soluie :
Fie n mpritorul , n > 15 1333 = n c1 13 |-13
1320 = nc1
351 = n c2 - 15 |-15 336 = nc2
n | 1320 i n | 336 n divizor comun al numerelor 1320 i 336.
Aflm c.m.m.d.c a celor dou numere: (1320; 336) = 233 = 24.
Singura soluie este n =24 pentru c divizorii ceilali alui 24 sunt mai mici dect 15.
2.5. Determinarea a dou numere naturale cnd cunoatem c.m.m.d.c. al lor i
produsul sau suma numerelor
Probleme rezolvate
R2.5.1. Determinai numerele a i b naturale pentru care: (a, b) = 15 i
ab = 6300
Soluie:
Din (a;b) =15 a = 15k i b = 15p unde k, p N* i (k; p) = 1
nlocuim pe a i b n relaia ab = 6300 i obinem : 15 k15p = 6300 | :225
kp = 12 1) k =1, p = 12 a= 15, b =180
2) k =12, p = 1 a = 180, b = 15
3) k = 3, p = 4 a = 45, b = 60
4) k = 4, p = 3 a = 60, b = 45
R2.5 2. S se afle numerele a i b naturale , tiind c cel mai mic multiplu
comun al lor este m i produsul lor este p.
Soluie:
Dac a, b N* atunci [ a;b](a;b) = ab
Din aceast relaie rezult c (a;b) =

ab
ab
p
, notam
= = c (a;b) = c a
[a; b]
[a; b] m

= ck , b = cp , k,p N* , (k;p) = 1.
20

Rezolvarea se face analog cu problema precedent.


R2.5.3. Determinai numerele naturale a i b tiind c (a; b) = d i a + b = s.
Soluie:
(a; b) = d a = dk , b = dp , unde k,p N i (k;p) =1
nlocuim pe a i b n a + b = s i obinem : dk + dp = s | :d ,
k+p=

s
N , pentru c d | s. Determinm perechile de numere (k;p) ce verific
d

egalitatea , apoi numerele a i b.


2.6. Fracii reductibile . Fracii ireductibile
Pentru a demonstra c o fracie este ireductibil trebuie s artm c
numrtorul i numitorul ei sunt numere prime ntre ele , numrtorul i numitorul
fiind numere naturale .
Fie a, b N*, a i b sunt prime ntre ele dac ( a; b ) = 1 unde ( a; b ) este
c.m.m.d.c. al numerelor a i b.
Probleme rezolvate
R2.6.1. Se consider fracia :

5n + 3
, n N*. Artai c fracia este ireductibil.
3n + 2

Soluie:

Presupunem c ( ) d astfel nct :


d | 5n + 3
i d | 3n +2
d | 3(5n + 3) i d | 5( 3n + 2 )
d | 5(3n + 2 ) 3( 5n +3 )
d | 15n +10- 15n 9 d | 1 d =1 numrtorul i numitorul sunt
numere naturale prime ntre ele rezult c fracia este ireductibil.
R2.6.2.Artai c fracia :

10n + 3
, n N este ireductibil.
15n + 4

Soluie:

Calculm c.m.m.m.c al numerelor 10 i15 [10 ; 15 ] = 30 , 30:10 =3;


15 = 2 . Fie d cel mai mare divizor comun al numerelor 10n +3 i 15n +4
d |10n + 3 i d | 15n+4
d | 3(10n + 3) i d | 2(15n +4)
d | 30n + 9 - 30n 8
d |1 d =1 fracia este ireductibil.

30 :

Reductibilitatea fraciilor
Pentru a arta c o fracie care depinde de o variabil natural este reductibil,
procedm astfel:
21

Fie fracia:

3n + 1
, n N.
2n + 3

Determinai numerele naturale n pentru care fracia este reductibil.


Soluie:

Fie d divizorul comun al numerelor 3n +1 i 2n + 3


d | 3n + 1 i d | 2n + 3
d | 2(3n + 1) i d | 3(2n + 3)
d | 6n 9 6n 2
d | 7 d = 7 pentru c 7 este numr prim
7 |3n + 1 i 7 | 2n + 3
7 | (3n + 1) (2n + 3)
7 | 3n + 1 - 2n -3
7 | n 2 n 2 = 7k , k N
n = 7k+ 2 n {2;9;16;23;;7k+2;}
Cel mai mic numr pentru care fracia este reductibil este n =2.

Bibliografie
C. Nstsescu,C. Ni, C. Vraciu, Aritmetic i algebr, EDP 1993
D. Buneag, F. Boboc, D. Piciu, Aritmetic i teoria numerelor, Ed. Universitaria
Craiova 1999
D. V. George, Cunotine vechi i noi despre divizibilitate, Ed. tiinific i
enciclopedic 1990
I. Petric i colectivul, Manual pentru clasa a VI-a, Ed. Petrion 1998
C. Popovici, I. Ligor, V. Alexianu, Matematic-Aritmetic-Algebr, EDP Bucureti
1996
G. Turcitu, I. Rizea, C. Basarab, M. Duncea, Manual clasa a VI-a, Ed. Radical 1998
T. Udrea, D. Nuescu, Manual clasa a VI-a, EDP 1998
Gheorghe i Alina Drugan; Ion i Mihaela Ghica, Matematica n concursurile colare,
Ed. Teora 1998, pag 34-38
A. Blaga, O.Pop, R. Pop. G. Buth, Matematica-Auxiliar la manualele de matematic,
Ed. Gil Zalu 2001, pag 20-28
D. Brnzei, D. i M. Goleteanu, S. Ulmeanu, V. Gorgot, I. erdean: Matematica n
concursurile colare, Ed. Paralela 45, 2000,2001,2002
D. Andrica, E. Jecan, D. Vlcan, I. Bogdan, Probleme calitative n matematica de
gimnaziu,Ed. Gil Zalu 1998, pag 21-44
C. Moroti, M. Giurgiu, D. Radu, R. tefan, A. Ciupitu, G. Drugan, I. Ghica, Matematic-exerciii i probleme pentru clasa a VI-a, Ed. Meteor Press 2002, pag 12-17

22

3.Cteva principii i metode de rezolvare a problemelor de matematic


3.1. Principiul paritii
n matematica elementar ntlnim multe probleme care folosesc noiunea de
paritate. Principiul paritii const n separarea cazurilor pare i impare dintr-o situaie.
Regulile paritii:
- suma a dou numere pare este un numr par
- suma a dou numere impare este un numr par
- suma dintre un numr par i altul impar este un numr impar
- produsul a dou numere pare este un numr par
- produsul a dou numere impare este un numr impar
- produsul dintre un numr par i un numr impar este un numr par.
Prezentm n continuare cteva probleme rezolvate care folosesc principiul
paritii.
R3.1.1. Demonstrai c dac suma a dou numere ntregi este un numr impar,
produsul lor este un numr par.
Soluie. Fie a i b numerele. Din ipotez a + b = 2n + 1 , n N . Deci unul din
numerele
a
sau
b
este
par.
Fie
a = 2k .
Atunci
b = 2n + 1 a = 2n + 1 2k = 2(n k ) + 1 , adic b este impar. Atunci a b este
produsul dintre un numr par i altul impar, deci va fi impar.
R3.1.2. Demonstrai c 2 n (n 2, n N ) se poate scrie ca o sum de dou
numere naturale impare consecutive, iar 3n se poate scrie ca o sum de trei numere
naturale consecutive i ca sum a trei numere impare consecutive.
Soluie. Pentru orice n 2 , n N , 2 n este numr par. Avem:

2 n = 2 2 n1 = 2 n1 + 2 n1 = (2 n1 1) + (2 n1 + 1)
Pentru n 2 , n N , 2 n 1 1 i 2 n 1 + 1 sunt impare consecutive.
Pentru orice n 2 , n N , 3n N i
3n = 3 3n1 = 3n1 + 3n1 + 3n1 = (3n1 1) + 3n + (3n1 + 1)
Numerele 3n 1 1 , 3n i 3n 1 + 1 sunt consecutive pentru n 2 .
Mai avem c

3n = 3n1 3 = 3n1 + 3n1 + 3n1 = (3n1 2) + 3n1 + (3n1 + 2) ,


unde 3n 1 2, 3n 1 , 3n 1 + 2 sunt impare consecutive.
R3.1.3. Se consider irul numerelor naturale de la 1 la 1979 adic:
1,2,3,4,...,1977,1978,1979. Luai la ntmplare oricare dou numere din acest ir i
nlocuii-le cu modulul diferenei lor. La fiecare operaie de acest fel numrul
numerelor din ir scade cu unu (fiindc am nlocuit dou numere cu unul) i vom
obine, n final, un singur numr. Artai c acest numr este par.
Soluie. La fiecare etap a operaiei descrise, numrul numerelor impare din ir
rmne neschimbat sau descrete cu doi, deoarece dac, n primul caz, lum un numr
23

par i unul impar, modulul diferenei lor este impar, deci numrul impar l-am nlocuit
cu altul impar, iar n al doilea caz dac lum dou numere impare, modulul diferenei
lor este un numr par, deci numrul numerelor impare scade cu doi. n irul
1,2,3,...,1979 avem (1+1979):2 numere impare, adic 990.
La fiecare pas rmne un numr par de numere impare i atunci ultimul numr
va fi cu siguran par.
R3.1.4. Se consider numerele impare k , n1 , n2 ,..., nk . S se demonstreze c
printre numerele:

n + n n + n1
n1 + n2 n2 + n3
,
,..., k 1 k , k
exist un numr impar de
2
2
2
2

numere impare.
Soluie. Suma a dou numere impare este un numr par, deci numerele

n + n1
n1 + n2 n2 + n3
sunt naturale. S presupunem c printre acestea se afl
,
,..., k
2
2
2

un numr par de numere impare. Atunci suma lor

n + n1
n1 + n2 n2 + n3
+
+ ... + k
= n1 + n2 + ... + nk
2
2
2

este un numr par. Dar aceeai sum este suma unui numr impar de numere impare
deci este un numr impar. Contradicie. Deci presupunerea fcut a fost fals, deci
printre numerele considerate n ipotez exist un numr impar de numere impare.
3.2. Probleme de numrare
Probleme de numrare ntlnim n diverse situaii din viaa cotidian. n
matematica colar sunt frecvente problemele de numrare ca de exemplu: numrul
divizorilor unui numr, numrul triunghiurilor, numrul patrulaterelor dintr-o anumit
configuraie, numrul cifrelor unui numr, numrul termenilor unui ir, etc. Prezentm
n continuare cteva probleme care conduc la operaia de numrare.
3.2.1. Numrul divizorilor i suma divizorilor unui numr natural
3.2.1.1. a) Numrul divizorilor unui numr natural
Fie a un numr natural compus ce are urmtoarea descompunere n factori

primi: a = p1 1 p 2 2 .... pn n , unde p1 , p 2 ,..., p n sunt numere prime iar

1 , 2 ,..., n N, n N . Pentru a obine numrul divizorilor lui a formm tabelul:


p10
p 02
...
p 0n

p11
p12
...
p1n

p12
p22
...
pn2

...
...
...
...
24

p11
p2 2
...
pn n

1 + 1 termeni
2 + 1 termeni
n + 1 termeni

(1)

Observm c:
1) Oricare numr din tabel este un divizor pentru a.
2) Linia nti conine 1+1 termeni, linia a doua conine 2+1 termeni,...,
ultima linie conine n+1 termeni.
3) Dac nmulim pe rnd fiecare numr din linia nti cu fiecare numr din
linia a doua obinem (1 + 1)( 2 + 1) divizori ai lui a. nmulind apoi pe fiecare din
aceste numere cu fiecare numr din linia a treia obinem (1 + 1)( 2 + 1)( 3 + 1)
numere i fiecare din acestea sunt divizori ai lui a. Continund raionamentul obinem
(1 + 1)( 2 + 1)( 3 + 1)...( n + 1) numere care sunt divizori ai lui a.
4) n numrul acestor divizori este inclus numrul nsui i divizorul 1.
Am obinut astfel urmtoarea

Teorema 3.2.1. Numrul divizorilor numrului a = p1 1 p 2 2 .... pn n este

(1 + 1)( 2 + 1)...( n + 1) .
3.2.1.2. b) Suma divizorilor unui numr natural
S calculm nti suma:

S = 1 + x + x 2 + ... + x n

(2)

Avem

x S = x + x 2 + ... + x n1 + x n + x n+1

(3)

Din (3) i (2) sczute membru cu membru obinem:

x S S = ( x + x 2 + x 3 + ... + x n + x n+1 ) (1 + x + x 2 + ... + x n )


care se mai scrie S ( x 1) = x n +1 1 , de unde
S=

x n+1 1
x 1

(4)

cu x 1 .
Scriem produsul de n sume, avnd termenii pe cele n linii din tabelul (1) i
obinem:
(1 + p1 + p12 + ... + p11 )(1 + p2 + p22 + ... + p2 2 )...(1 + pn + pn2 + ... + pn n ) (5)
Cu relaia (4), (5) devine

p n +1 1
p11 +1 1 p2 2 +1 1

... n
p1 1
p2 1
pn 1
Am obinut astfel

Teorema 3.2.2. Suma divizorilor numrului a = p1 1 p 2 2 .... pn n este

pn n +1 1
p11 +1 1 p2 2 +1 1
S=

...
p1 1
p2 1
pn 1

25

Probleme rezolvate
R3.2.1. Fie S suma divizorilor naturali ai numrului 2001. S se arate c 5S
este numr natural ptrat perfect.
Soluie. Fiindc 2001=31231291, suma divizorilor numrului 2001 este:

32 1 232 1 29 2 1
= 4 24 30

3 1 23 1 29 1
Atunci 5 S = (2 3 3 5) 2 = 120 2 , deci 5S este ptrat perfect.
S=

R3.2.2. S se arate c ptratul produsului tuturor divizorilor naturali ai


numrului 2001 este 20018.
Soluie. Avem urmtoarea
Lema 3.2.1. Dac d1 , d 2 ,..., d n sunt toi divizorii naturali ai numrului n
atunci avem relaia:
(d1 d 2 ...d k ) 2 = n k
(*)
Fiindc 1 i n sunt i ei divizori, considernd d1 < d 2 < ... < d k obinem:

d1 =

n
n
n
, d2 =
,..., d k =
dk
d k 1
d1

relaii care nmulite membru cu membru dau

d1 d 2 ...d k =

n n
n

...
d k d k 1 d1

de unde (d1 d 2 ...d k ) 2 = n k .


n cazul nostru 2001 = 31 231 291 , numrul divizorilor lui 2001 este:
(1+1)(1+1)(1+1)=8. Pentru cei opt divizori naturali ai numrului 2001 avem relaia (*)
(d1 d 2 ...d 8 ) 2 = 20018 .
3.3. Principiul lui Dirichlet
Matematicianul german Peter Gustav Dirichlet (1805-1859) a elaborat un
principiu extrem de simplu cu aplicaii neateptate n variate domenii, principiu care-i
poart numele i pe care-l enunm mai jos, fiind o metod de demonstraie de tipul
urmtor.
"Dac repartizm n + 1 obiecte n n cutii, atunci cel puin dou obiecte vor fi
n aceeai cutie."
Justificare: Considerm cazul cel mai nefavorabil aeznd n fiecare cutie cte
un obiect. Deci am folosit n cutii i n obiecte. Obiectul cu numrul n + 1 trebuie pus i
el ntr-o cutie oarecare. Dar n acea cutie exist deja un obiect. Aadar n acea cutie
exist deja un obiect pus anterior. n acea cutie vor fi dou obiecte.
Forma general a principiului lui Dirichlet este urmtoarea:
"Dac aezm kn + 1 obiecte n n cutii, atunci cel puin k + 1 obiecte, k N ,
vor fi n aceeai cutie."
26

n literatura matematic principiul lui Dirichlet este ntlnit i sub denumirea


de "principiul cutiei", cu precizarea c denumirea de "cutie" desemneaz "grupe de
obiecte", stabilite dup anumite criterii, iar "obiectele" desemneaz lucruri, numere,
figuri geometrice, etc. Prezentm n continuare cteva probleme ale cror soluii se
bazeaz pe principiul de mai sus.
Probleme rezolvate
R3.3.1. La un turneu de ah au participat n 2 ahiti. S se demonstreze c
n orice moment al turneului dinaintea ultimei runde cel puin doi ahiti au acelai
numr de victorii.
Soluie. n orice moment al turneului dinaintea ultimei runde, fiecare ahist a
jucat maximum n 2 partide i a putut obine 0,1,2,..., n 2 victorii, deci n total
n 1 posibiliti (cutii). Fiindc la turneu au participat n ahiti rezult c cel puin doi
ahiti au acelai numr de victorii naintea ultimei runde.
R3.3.2. Artai c n orice mulime format din 5 numere naturale exist dou
a cror diferen este divizibil cu 4.
Soluie. La mprirea unui numr cu 4 obinem unul din resturile 0,1,2,3 (deci
patru cutii). Fiindc avem 5 numere (5 obiecte i 4 cutii) rezult c cel puin dou
numere vor da acelai rest la mprirea cu 4. Ele sunt de forma x = 4k + r i
y = 4l + r . Atunci diferena lor este x y = 4(k l ) , adic un numr divizibil cu 4.
R3.3.3. ntr-o coal sunt 367 elevi. S se demonstreze c exist cel puin doi
elevi care-i serbeaz ziua n aceeai zi a anului.
Soluie. Un an are 365 sau 366 zile. Considernd cazul cel mai nefavorabil
cnd n fiecare zi a anului ar fi nscut cte un elev, nseamn c n total ar fi nscui
365 sau 366 elevi, dar n total sunt 367 elevi. Deci al 367-lea elev a fost i el nscut
ntr-o zi a anului n care a mai fost nscut un elev. Deci ntr-o zi s-au nscut 2 elevi,
deci cei doi i vor serba ziua de natere n aceeai zi.
R3.3.4. Fiind date n + 1 numere naturale (n 0) atunci cel puin dou dintre
ele dau acelai rest la mprirea cu n.
Soluie. Folosim teorema mpririi cu rest. Fiind date numerele naturale a i b
(b 0) exist n mod unic numerele naturale q i r astfel ca
a = b q + r cu r < b .
n cazul problemei noastre numerele fiind mprite la n exist pentru rest n
valori posibile: 0,1,2,...,n-1. Fiindc mprim n+1 numere vor exista n+1 resturi, dintre
care cel mult n sunt diferite. Rezult c cel puin dou dintre cele n+1 numere mprite
la n dau acelai rest.
R3.3.5. S se arate c oricum am alege 7 numere ptrate perfecte (distincte)
exist cel puin dou a cror diferen se divide cu 10.
Soluie. Dac a este numrul a crui ptrat este a 2 atunci la mprirea cu 10 a
lui a obinem unul din resturile: 0,1,2,3,4,5,6,7,8,9. Atunci a 2 , la mprirea cu 10 va
da unul din resturile: 0,1,4,5,6,9. Fiindc avem 7 ptrate perfecte i numai resturile
0,1,4,5,6,9, deci exist cel puin dou ptrate perfecte care dau acelai rest la mprirea
cu 10, deci diferena lor se divide cu 10.

27

3.4. Principiul invariantului


Invariantul este o mrime, o relaie, sau o proprietate care rmne neschimbat
n urma aplicrii sau interveniei unei transformri.
Deci o situaie iniial este supus n mod repetat unor transformri. De obicei
se cere s se demonstreze c n urma acestor transformri nu se poate ajunge la o
anumit form. Aceasta se poate face alegnd caracteristica obiectului care a fost supus
transformrii, adic "invariantul" transformrii. Dac n final obiectul nu posed
"invariantul" atunci el nu poate fi obinut n urma transformrilor descrise.
Probleme rezolvate
R3.4.1. Considerm un numr natural cruia i schimbm n mod arbitrar
ordinea cifrelor. Este posibil ca diferena dintre numrul iniial i cel final s fie 2003?
Soluie. Restul mpririi numrului la 9 este acelai cu restul mpririi sumei
cifrelor sale la 9. Suma cifrelor este aceeai, rezult c restul mpririi numrului la 9
este un invariant.
R3.4.2. Pe o tabl sunt scrise semne de "+" i "". tergem dou semne i le
nlocuim cu un semn, dup urmtoarea regul: dac cele dou semne terse sunt
identice le nlocuim cu "+", iar dac tergem dou semne diferite le nlocuim cu "".
Artai c ultimul semn care rmne dup un numr de pai nu depinde de ordinea
alegerii perechilor.
Soluie. n acest caz paritatea numrului de minusuri va fi invariantul. Dac la
nceput numrul de minusuri este impar, ultimul semn care va rmne este minus, iar
dac la nceput numrul de minusuri este par, la sfrit va rmne plus.
R3.4.3. Trei greieri se gsesc pe o dreapt n ordinea: A, B, C. Ei ncep s sar
capra, adic s sar unul peste altul (dar nu peste doi odat). Pot fi n aceeai ordine
dup 2003 srituri?
Soluie. n urma unei srituri de acest fel numrul perechilor de greieri
inversai crete sau se micoreaz cu 1 (proprietatea invariant). Dup un numr impar
de srituri (2003) va exista un numr impar de perechi de greieri inversai. Deci nu se
poate obine ordinea iniial (ce nu conine o astfel de pereche).
R3.4.4. O camer are dimensiunile podelei de 7m i 10m. n cele patru coluri
ale camerei se aeaz cte un dulap avnd baza ptrat cu latura de 1m. S se arate c
rmne din suprafaa podelei nu poate fi acoperit cu plci dreptunghiulare de
dimensiuni 3m1m.
Soluie. Se mparte camera ntr-o reea de ptrate cu latura 1m pe care le
vopsim n trei culori: rou, alb, negru ca mai jos:
RANRANRANR
ANRANRANRA
NRANRANRAN
RANRANRANR
ANRANRANRA
NRANRANRAN
RANRANRANR

28

Obinem 24 de R, 23 de A, 23 de N. Eliminnd colurile rmn 20 de ptrele


roii, 23 de ptrele albe, 23 de ptrele negre. Dar oricum am aeza o plac de 31
ea acoper un ptrel rou, unul alb i unul negru. Dac s-ar putea acoperi suprafaa cu
un numr ntreg de plci ar trebui s existe acelai numr de ptrele pentru fiecare
culoare.
3.5. Probleme de logic
Am inclus aici cteva probleme a cror rezolvare se realizeaz printr-o serie de
judeci logice ce solicit inventivitate, perspicacitate, etc. i foarte puin calcul.
Probleme rezolvate
R3.5.1. Mama a observat c din dulap au disprut cinci tablete de ciocolat.
Ele puteau fi luate de cei trei copii: A, B, C. Fiind trai la rspundere, ei au dat mai
nti urmtoarele rspunsuri:
A: N-am luat nici o ciocolat!
B: N-am luat nici o ciocolat!
C: N-am luat nici o ciocolat!
Dup un nou "interogatoriu" copiii au fcut urmtoarele declaraii:
A: B a luat mai multe tablete dect C!
B: (ctre A): Mini!
C: Toate au fost luate de A i B!
A (ctre C): Mini!
Aflai cte tablete de ciocolat au fost luate de ctre fiecare copil, tiind c
fiecare a fcut attea declaraii false cte tablete de ciocolat a luat.
Soluie. Fiindc au "disprut" 5 ciocolate i s-au fcut 7 declaraii, rezult c
cinci declaraii erau false iar 2 (7-5) adevrate.
La al doilea "interogatoriu" prima afirmaie a lui A este fie adevrat i atunci
afirmaia lui B este fals, fie este fals i atunci afirmaia lui B este adevrat. Tot din
"interogatoriul" al doilea afirmaia lui C este fie adevrat i atunci cea de-a doua
afirmaie a lui A este fals, fie este fals i atunci cea de-a doua afirmaie a lui A este
adevrat. Deci rezult c cele dou afirmaii adevrate au fost fcute la cel de-al
doilea "interogatoriu", deci la primul "interogatoriu" toi copiii au fcut declaraii false,
de unde rezult c fiecare din cei trei copii a luat cel puin o ciocolat. Deci a doua
afirmaie a lui C este fals, deci C a luat dou tablete de ciocolat. B face numai dou
afirmaii, deci el nu poate lua mai multe ciocolate dect C, rezult c la al doilea
"interogatoriu" prima afirmaie a lui A este fals, deci afirmaia lui B este adevrat.
Deci A a luat dou ciocolate, B a luat o ciocolat, iar C a luat dou ciocolate.
R3.5.2. ntr-un bloc locuiesc familiile A, B, C, D, E, F, G, H, I, K, L, M, N, O,
P, R. La parter i la fiecare etaj locuiesc cte dou familii. Se mai tie c: Familia A
locuiete cu dou etaje mai jos ca familia B, iar aceasta cu ase etaje mai sus ca familia
C. Familiile F i G locuiesc la acelai etaj. Familia M locuiete cu patru etaje mai sus
29

ca familia N i cu dou etaje mai jos ca familia F. Un etaj deasupra familiei N locuiete
familia O. Familia A locuiete cu trei etaje mai sus ca familia R, iar familia P locuiete
cu cinci etaje mai jos dect familia G.
a) Cte etaje are blocul?
b) La ce etaj locuiete familia A?
Soluie. a) Fiindc la parter i la fiecare etaj locuiesc dou familii, iar n tot
blocul locuiesc 16 familii, rezult c blocul are opt nivele (parter i apte etaje).
b) Diagramele alturate (stabilite conform enunului) pun n eviden modul
cum sunt distribuite n bloc familiile N, O, M, F, G, P precum i C, R, A, B. Blocul
avnd opt nivele, rezult c familia N poate locui numai la parter sau la etajul nti.
Aceeai remarc i pentru familia C. Familiile N i C nu pot locui la acelai nivel,
pentru c ar trebui ca familiile O, P, R s locuiasc la acelai etaj, situaie imposibil,
pentru c la un nivel pot locui numai dou familii. Dac familia N locuiete la parter
atunci familia C ar trebui s locuiasc la etajul nti, situaie imposibil deoarece ar
rezulta c familiile O, P, C locuiesc la acelai etaj. Dac familia N locuiete la etajul
nti, atunci familia C locuiete la parter. Urmrind comparativ diagramele observm
c este o situaie posibil pentru c la un nivel pot locui numai dou familii.
Acestea fiind precizate putem stabili distribuiile familiilor n bloc: Familiile F
i G la etajul apte, familia B la etajul ase, familia M la etajul cinci, familia A la etajul
patru, familiile P i O la etajul doi, familiile N i R la etajul nti, iar familia C la
parter. n cele ase locuri neocupate se vor distribui familiile D, E, H, I, K, L dup
voie.
F

R
C

R3.5.3. La un turneu de fotbal particip 15 echipe, fiecare dintre acestea


jucnd cu toate celelalte. Pentru victorie se acord 3 puncte, pentru meci egal 2 puncte,
iar pentru nfrngere un punct. n clasamentul ntocmit la sfritul turneului nu exist
echipe cu acelai numr de puncte. tiind c ultima echip are 21 de puncte, s se arate
c prima a fcut cel puin un meci nul.
Soluie. Fiecare echip a disputat 14 meciuri. Numrul meciurilor disputate a
fost

14 15
= 105 deoarece fiecare meci a fost numrat de dou ori (i cnd a jucat A
2

cu B i cnd a jucat B cu A). Fiindc echipa clasat pe ultimul loc are 21 de puncte, iar
n clasament nu sunt echipe cu acelai numr de puncte rezult c numrul de puncte
este mai mare sau egal cu
30

21 + (21 + 1) + (21 + 2) + ...(21 + 14) = 21 15 +

14 15
= 420
2

Fiindc la fiecare meci s-au acordat 4 puncte, rezult c numrul de puncte


acordat a fost 1054=420. Deci echipele au obinut punctajele: 21, 22,23,...,34, 35.
S artm c echipa de pe locul nti cu 35 puncte a fcut cu siguran cel puin
un meci nul. Presupunem c nu a fcut nici un meci nul. Atunci dac x este numrul
victoriilor i y numrul nfrngerilor avem:
x + y = 14 i 3x + y = 35 ,
de unde x =

7
21
i y = . Dar x, y trebuie s fie naturale. Deci presupunerea fcut
2
2

este fals, atunci echipa de pe locul nti a fcut cel puin un meci nul.
R3.5.4. La un concurs de atletism particip trei echipe: A1 , A2 , A3 , fiecare cu
trei concureni. Concurentul care sosete primul primete 18 puncte, cel care sosete al
doilea 16 puncte, cel care sosete al treilea 14 puncte,..., cel care sosete ultimul
primete dou puncte. Punctajul unei echipe este suma punctelor obinute de cei trei
reprezentani ai si. Aflai ce loc a ocupat fiecare echip tiind c:
i) Primele trei locuri au fost ocupate de concureni de la echipe diferite.
ii) Fiecare concurent de la echipa A2 avea n faa sa un concurent de la echipa

A1 .
iii) Concurenii echipei A3 au sosit unul dup altul.
Soluie. Din prima i a treia condiie rezult c cei trei reprezentani ai echipei
A3 au sosit al treilea, al patrulea i al cincilea. Din primele dou condiii rezult c
primul a sosit un concurent de la echipa A1 , iar al doilea un concurent de la echipa A2 .
Din cele de mai sus i din a doua condiie rezult c al aselea i al optulea au sosit
concurenii de la echipa A1 , iar al aptelea i al noulea au fost concurenii de la echipa

A2 . Deci echipa A1 a acumulat 18+8+4=30 (puncte), echipa A2 a acumulat


16+6+2=24 (puncte), iar echipa A3 a acumulat 14+12+10=36 (puncte). Deci pe locul
nti se afl echipa A3 , pe locul doi echipa A1 , iar pe locul trei echipa A2 .
R3.5.5. Opt ahiti particip la un turneu, jucnd fiecare cu fiecare. Pentru
fiecare victorie un juctor primete un punct, pentru remiz un jumtate de punct, iar
pentru nfrngere nu primete nici un punct. La sfritul turneului primii doi clasai au
obinut punctaje diferite, iar cel de-al doilea a obinut attea puncte cte au obinut
ultimii patru ahiti mpreun. S se afle cum s-a ncheiat partida dintre ahitii clasai
pe locurile trei i cinci.
Soluie. Fiindc au fost opt juctori i fiecare a jucat cu fiecare, un ahist a
jucat apte partide i ar fi putut ctiga cel mult apte puncte (cnd nvingea n toate
cele apte partide). Ultimii patru ahiti au jucat ntre ei ase partide. (Dac A, B, C , D
sunt ultimii ahiti, au jucat: A cu B , A cu D , B cu C , B cu D i C cu D ).
31

Deci ultimii patru ahiti au realizat mpreun cel puin ase puncte. Deci ahistul de pe
locul doi a obinut cel puin ase puncte (deoarece el a obinut un numr egal de puncte
cu suma ultimilor patru). Fiindc primii doi juctori au punctaje diferite nseamn c al
doilea a obinut exact ase puncte, cci dac obinea 6,5 puncte primii doi aveau acelai
punctaj, iar dac ar fi obinut apte puncte era pe primul loc. Deci ahitii de pe
ultimele patru locuri au obinut exact ase puncte, aceasta nseamn c ei au pierdut
toate partidele jucate mpotriva primilor patru clasai. Deci ahistul de pe locul cinci a
pierdut partida susinut cu cel de pe locul trei.
R3.5.6. n trei couri sunt mere. Cte mere sunt n fiecare co tiind c n
primele 2 mpreun este un mr, n ultimele 2 mpreun este cel puin un mr, iar n
ultimul i al treilea mpreun, numai unul.
Soluie. Dac mrul din primele 2 couri s-ar afla n primul co, atunci din a
treia condiie ar rezultat c n al treilea co nu se afl nici un mr, deci al doilea i al
treilea co ar fi goale i astfel nu ar avea loc a doua condiie. n concluzie primul co
este gol, al doilea co conine un mr, iar al treilea co conine tot un mr.
3.6. Probleme de ordonare
Pentru a stabili care dintre dou numere a i b este mai mare, putem folosi mai
multe procedee, dintre care cele mai des ntrebuinate sunt:
1) Stabilim semnul diferenei a b .
Dac a b > 0 , atunci a > b .
Dac a b = 0 , atunci a = b .
Dac a b < 0 , atunci a < b .
2) Dac numerele a i b sunt pozitive i b 0 , comparm raportul

a
cu 1.
b

a
< 1 , atunci a < b .
b
a
Dac = 1 , atunci a = b .
b
a
Dac > 1 , atunci a > b .
b
Dac

3) n unele situaii este suficient s demonstrm existena unui numr c situat


ntre cele dou numere (exemplu: din a < c < b , rezult a < b ).
n unele situaii avem nevoie de metode ingenioase pentru a rezolva
problemele. Exist cazuri cnd operaia de ordonare ajut la dovedirea egalitii a dou
numere x i y prin stabilirea simultan a inegalitilor x y i y x .
Probleme rezolvate
R3.6.1. Comparai numerele 3111 cu 1714 .
32

Soluie. 3111 < 3211 = (2 5 )11 = 2 55 < 2 56 = (2 4 )14 = 1614 < 1714 .
R3.6.2. Scriei n ordine cresctoare numerele:
4 22 ,334 ,2 44 ,637 .
Soluie. 637 < 64 7 = (2 6 ) 7 = 2 42 < 2 44 = (2 2 ) 22 = 4 22 < 2 51 = (2 3 )17 =

= 817 < 917 = (32 )17 = 334 . Deci 637 < 2 44 = 4 22 < 334 .
R3.6.3. S se arate c pentru orice numere naturale a i b avem

a + 2 360 b + 51800
<
a3240
b + 71440
Soluie. 2 360 = (2 3 )120 = 8120 ; 3240 = (32 )120 = 9120 .
Atunci obinem c 2 360 < 3240 , de unde rezult c a + 2 360 < a + 3240 i deci
a + 2360
<1
b + 3240
51800 = (55 ) 360 = 3125360 ; 71440 = (7 4 ) 360 = 2401360
1800

Rezult c 5

>7

1440

, de unde rezult c b + 5

1800

>b+7

1440

b + 51800
i deci
> 1.
b + 71400

Deci prima fracie din ipotez este subunitar iar a doua este supraunitar i atunci
relaia cerut este adevrat.
R3.6.4. Comparai fraciile A i B unde

A=

2 + 2 2 + 2 3 + ... + 21997
1 1
1
+ 2 + ... + 1997
2 2
2

3 + 32 + ... + 31331
1 1
1
+ 2 + ... + 1331
3 3
3
i a doua cu 31332 i obinem:

i B =

Soluie. Amplificm prima fracie cu 21998

21998 (2 + 2 2 + 2 3 + ... + 21997 )


= 21998 = (2 3 ) 666 = 8666
1998
1997
2 + 2 + ... + 2
1332
3 (3 + 32 + ... + 31331)
= 31332 = (32 ) 666 = 9 666 .
B=
2
1331
3 + 3 + ... + 3
Deci A > B .

A=

3.7. Metoda reducerii la absurd


Metoda reducerii la absurd este o metod specific de demonstraie n
matematic. La baza acestei metode st una din legile fundamentale ale logicii clasice:
legea terului exclus, ce are urmtorul enun:
Din dou propoziii contradictorii una este adevrat, cealalt fals, iar a treia
posibilitate nu exist.

33

Legea terului exclus nu ne precizeaz care din cele dou propoziii este
adevrat i care este fals.
Cnd la dou propoziii contradictorii aplicm legea terului exclus este
suficient s stabilim c una dintre ele este fals pentru a deduce c cealalt este
adevrat.
Metoda reducerii la absurd const n a admite n mod provizoriu, ca adevrat
propoziia contradictorie propoziiei de demonstrat, apoi pe baza acestei presupuneri se
deduc o serie de consecine care duc la un rezultat absurd, deoarece ele contrazic sau
ipoteza problemei date sau un adevr stabilit mai nainte. Mai departe raionm astfel:
dac presupunerea ar fi fost adevrat, atunci n urma raionamentelor logic corecte ar
fi trebuit s ajungem la o concluzie adevrat, deoarece am ajuns la o concluzie fals,
nseamn c presupunerea noastr a fost fals. Aceasta duce la concluzia c
presupunerea fcut nu este posibil i rmne ca adevrat concluzia propoziiei date.
Metoda reducerii la absurd nu se reduce la propoziia c "a demonstra o
propoziie este acelai lucru cu a demonstra contrara reciprocei ei", deoarece pot aprea
i situaii n care nu se contrazice ipoteza ci o alt propoziie (un rezultat cunoscut, o
axiom, o teorem). Metoda reducerii la absurd se folosete att n rezolvarea
problemelor de calcul (de aflat) ct i la rezolvarea problemelor de "demonstrat".
Metoda este des utilizat n demonstrarea teoremelor reciproce, precum i n
demonstrarea teoremelor de unicitate.
Probleme rezolvate

39n + 4
este ireductibil.
26n + 3
Soluie. Presupunem c fracia este reductibil i fie d = (39n + 4,26n + 3) cu
d N * , d 1 . Din d | (39n + 4) i d | (26n + 3) obinem c d | (78n + 8) i
d | (78n + 9) , de unde rezult c d | [78n + 9 (78n + 8)] , deci d | 1 , de unde rezult
d = 1 . Fals.
R3.7.1. Artai c pentru orice n N fracia

R3.7.2. S se arate c nu exist numere ntregi a pentru care numerele

14a + 5 17a 5
i
s fie simultan ntregi.
9
12

14a + 5
17a 5
i
9
12
17a 5
14a + 5
s fie simultan ntregi, adic pentru b, c Z ,
= b i
= c , de unde
9
12
14a + 5 = 9b i 17 a 5 = 12c i scznd prima relaie din a doua obinem:
3a 10 = 12c 9b , de unde 10 = 3a 12c + 9b sau 10 = 3(a 4c + 3b) . Atunci
Soluie. Presupunem c exist numere ntregi a astfel ca

obinem c 3 divide pe 10, ceea ce este absurd.

34

R3.7.3. Considerm trei drepte diferite d1 , d 2 , d 3 concurente ntr-un punct O.


Artai c cel puin unul din unghiurile formate are msura mai mare sau cel puin
egal cu 60.
Soluie. Folosim metoda reducerii la absurd. Presupunem concluzia fals, adic
nu exist un unghi cu msura mai mare sau egal cu 60. Atunci cele ase unghiuri
formate ar avea suma msurilor mai mic dect 360. Am ajuns la o contradicie
deoarece suma msurilor unghiurilor n jurul unui punct este 360. Deci presupunerea
fcut este fals, deci exist cel puin un unghi cu msura de 60.

35

4. Rapoarte i proporii
Rapoarte

a
; a i b se numesc
b

Raportul numerelor raionale a i b, b 0 , este expresia

termenii raportului.
Ctul termenilor unui raport se numete valoarea raportului.
Exemplu: valoarea raportului

3,5
este 0,5.
7

Termenii unui raport se exprim ntotdeauna cu aceeai unitate de msur.


Aplicaiile rapoartelor n practic sunt: scara unui plan, scara unei hri,
probabilitatea realizrii unui eveniment, procente, titlul unui aliaj.
4.1. Scara unui plan
Prin scara unui plan nelegem raportul dintre distana din plan i distana din
realitate dintre aceleai dou puncte, ambele distane fiind exprimate cu aceeai unitate
de msur.
Remarc. De obicei, numrtorul raportului prin care se exprim scara este 1.
Model. Figura de mai jos reprezint planul unui apartament. Acest plan este

realizat la scara

1
. Aceasta nseamn c la 1 cm din desen corespund, n realitate,
100

100cm. Cu alte cuvinte, n plan lungimea sufrageriei este de 5cm, iar n realitate este de
500cm, adic de 5m.
La planul din figur s se determine:
a) limea, n centimetri, a dormitorului
b) dimensiunile, n centimetri, ale buctriei
c) perimetrul, n centimetri, a holului
d) aria, n cm2 a sufrageriei.
Soluie.
a) Limea dormitorului de 3m, din realitate, este n plan de 3cm.
b) Dimensiunile de 2m i 3m ale buctriei, din realitate, sunt n plan de 2cm,
respectiv 3cm.
c) Holul are dimensiunile de 8m i 2m, n realitate, deci n plan ele vor fi 8cm
i 2cm, rezult c perimetrul holului n plan este de 20cm.
d) Sufrageria are dimensiunile de 5m i 4m, n realitate, deci n plan 5cm i
4cm, rezult c aria sufrageriei n plan este 20cm2.
Probleme rezolvate
R4.1.1. Care este scara planului unei grdini, dac o latur a grdinii, care are
125m, este reprezentat n plan printr-un segment lung de 25cm?
36

Soluie. Aplicnd definiia scrii unui plan, ca fiind raportul dintre distana din
plan i distana din realitate, ambele exprimate n aceeai unitate de msur, scara
planului este

25
1
, adic
.
12500
500

R4.1.2. O grdin n form de dreptunghi, are pe un plan cu scara de

1
300

dimensiunile de 4cm i 5cm. Ce suprafa, n hectare, are grdina n teren?


Soluie. n planul cu scara

1
, lungimea de 1 cm corespunde la 300 cm din
300

realitate. Dimensiunile grdinii vor fi 4300cm i 5300cm, adic 12m i 15m. Aria
grdinii este de 0,018ha.
R4.1.3. Planul unui parc are scara de

1
.
200

a) n plan se afl un loc de form circular, cu raza de 1m, ce reprezint lacul.


Ci centimetri are raza cercului n plan?
b) Spaiul de joac pentru copii este, n teren, un ptrat cu aria de 100m2. Ce
arie are n plan spaiul de joac pentru copii?
Soluie. a) n planul cu scara de

1
, lungimea de 1cm corespunde la 200cm
200

din teren. Dac raza cercului este n teren 1m, adic 100cm, ea corespunde n plan unei
lungimi de 0,5cm.
b) Latura ptratului din teren are 10m, deci 1000cm, iar n plan latura
ptratului are 1000:200=5cm. Rezult c aria ptratului n plan este de 25cm2.
4.2. Scara unei hri
Prin scara unei hri nelegem raportul dintre distana de pe hart i distana
din realitate dintre aceleai dou puncte, distanele fiind msurate cu aceeai unitate de
msur, iar numrtorul raportului prin care se exprim scara este 1.
Model. n figura de mai jos, harta Romniei este realizat la scara

1
,
10000000

aceasta nsemnnd c la 1cm de pe hart corespund 10000000cm=100km n realitate


(teren).

37

De exemplu, distana pe osea, dintre Bucureti i Braov, pe hart, este 17mm,


iar distana din teren d o determinm astfel:

1
1,7
=
d = 17000000cm = 170km .
10000000 d
Distana real dintre oraele Bacu i Bucureti este de 300km. Care este
distana, n centimetri, pe harta cu scara
Avem

1
?
10000000

1
x
=
x = 3cm .
10000000 30000000

4.3. Probabilitate
Definiie. Probabilitatea realizrii unui eveniment este raportul dintre numrul
cazurilor favorabile realizrii evenimentului i numrul cazurilor posibile (ale
experienei).
Remarc. Probabilitatea realizrii unui eveniment este un numr mai mare sau
egal cu 0 i mai mic sau egal cu 1.
Evenimentul imposibil are probabilitatea 0.
Evenimentul sigur are probabilitatea 1.
Model. ntr-o urn sunt bile numerotate de la 1 la 50. Care este probabilitatea
ca extrgnd o singur bil numrul obinut s fie ptrat perfect?
Soluie. n total, exist 50 cazuri posibile i 7 cazuri favorabile (apariia
numrului 1, 4, 9, 16, 25, 36, 49) deci probabilitatea cerut este p =

38

7
.
50

Probleme rezolvate
R4.3.1. Care este probabilitatea ca aruncnd dou zaruri, s obinem dou fee
nsumnd: a) 9 puncte; b) un numr prim de puncte.
Soluie. La aruncarea a dou zaruri exist 66=36 cazuri posibile.
a) Numrul cazurilor favorabile obinerii sumei 9 puncte este 4 (3+6, 4+5, 5+4,
6+3), deci probabilitatea cerut este

4
1
, adic .
36
9

b) Numrul cazurilor favorabile obinerii sumei un numr prim de puncte este


15 (1+1, 1+2, 2+1, 1+4, 2+3, 3+2, 4+1, 1+6, 2+5, 3+4, 4+3, 5+2, 6+1, 5+6, 6+5),
rezult c probabilitatea cerut este

15
5
, adic
.
36
12

R4.3.2. ntr-un co sunt 6 plicuri albe i 4 plicuri roii. Un copil, legat la ochi,
extrage dou plicuri. Calculai probabilitatea evenimentelor:
E1 : s extrag dou plicuri de aceeai culoare

E2 : s extrag dou plicuri de culori diferite.


Soluie. Numrul cazurilor posibile este 910=90.
a) Numrul cazurilor favorabile realizrii evenimentului E1 : 65+43=42,

42
7
, adic p ( E1 ) =
.
90
15
b) Numrul cazurilor favorabile realizrii evenimentului E 2 : 64+46=48 (sau
48
8
, adic p ( E 2 ) =
.
90-42); probabilitatea realizrii evenimentului E 2 este
90
15
Remarc. Probabilitatea realizrii evenimentului E 2 se putea calcula i
7
8
1 = , pentru c singurele situaii posibile la extragerea a dou plicuri din co
15 15

probabilitatea realizrii evenimentului E1 este

este ca ele s fie de aceeai culoare sau de culori diferite.


R4.3.3. O carte cu 270 de pagini este deschis la ntmplare. S se determine
probabilitatea evenimentelor urmtoare:
A: numrul paginii din stnga este numr par
B: numrul paginii din dreapta este multiplu de 5
C: numrul paginii din stnga este multiplu de 6
D: numrul paginii din dreapta este divizibil cu 7.
Soluie. Numrul paginii din stnga este ntotdeauna par, deci P ( A) = 1 .
Numrul paginii din dreapta este ntotdeauna numr impar, deci trebuie s
numrm multipli impari ai lui 5, mai mici sau egali cu 265; ei sunt 51, 53, 55,...,
553=265, deci n total

53 1
27 1
+ 1 = 27 cazuri favorabile, de unde P ( B) =
= .
135 5
2

39

Numrul paginii din stnga este numr par ntotdeauna i el trebuie s fie
multiplu de 6 mai mic dect 270; obinem 61, 62, 63,..., 645=270, 45 cazuri
favorabile, de unde P (C ) =

45 1
= .
135 3

Numrul paginii din dreapta este numr impar, divizibil cu 7 i mai mic dect
270, obinem 71, 73, 75,..., 737, deci n total
unde P ( D) =

37 1
+ 1 = 19 cazuri favorabile, de
2

19
.
135

4.4. Procente
Definiie. Un raport de forma

p
, p Q , p 0 , se numete raport
100

p
i se citete "p la sut" sau "p procente".
100
p
Pentru a afla ct reprezint p% dintr-un numr dat a, calculm
a .
100

procentual. Scrierea p% nseamn

Pentru a afla un numr necunoscut x cnd tim c p% din x reprezint b,


calculm x = b :

p
.
100

Model 1. La faza naional a olimpiadei de matematic particip 600 de elevi.


Din numrul total de participani 5% primesc premiul I, 10% premiul al II-lea, 15%
premiul al III-lea i 20% premii speciale i meniuni. Ci elevi primesc premiul I, dar
premiul al II-lea, dar premiul al III-lea? Ce procent din numrul elevilor care au primit
premii speciale i meniuni reprezint numrul elevilor cu premiul I?
Soluie. Pentru a afla ci elevi au obinut premii i meniuni, avem:

5
10
600 = 30 elevi primesc premiul I,
600 = 60 elevi primesc premiul al II-lea,
100
100
15
20
600 = 90 elevi primesc premiul al III-lea i
600 = 120 elevi primesc
100
100
premii speciale i meniuni.
Apoi,

x
120 = 30 , de unde x = 25 , deci 25% din numrul elevilor care au
100

primit meniuni i premii speciale reprezint numrul elevilor care au primit premiul I.
Model 2. Pentru a cumpra un tricou, o persoan pltete 150000lei, ceea ce
reprezint 30% din suma pe care o are. Ce sum are persoana?

40

30
s = 150000 , unde s este suma pe care o are persoana.
100
100
De aici rezult c s = 150000
, deci s = 500000 , persoana deine suma de
30
Soluie. tim c

500000lei.
Probleme rezolvate
R4.4.1. O suprafa de 150ha este arat n trei zile, astfel: n prima zi 40% din
suprafa, a doua zi 30% din rest, iar a treia zi ce a mai rmas.
a) Cte hectare s-au arat zilnic?
b) Ce procent din ntreaga suprafa s-a arat a doua zi? Dar a treia zi?

40
150 = 60 ha. Restul dup prima zi este
100
30
150-60=90ha. n a doua zi s-au arat
90 = 27 ha, iar a treia zi restul, adic 90100
Soluie. a) n prima zi s-au arat

27=63ha.
b) Avem

x
150 = 27 , de unde rezult c x = 18 , deci a doua zi s-a arat
100

18% din suprafaa total.


La fel,

y
150 = 63 , de unde rezult y = 42 , deci a treia zi s-a arat 42% din
100

suprafaa total (sau 100%-40%-18%).


R4.4.2. Dup ce un turist a parcurs 38% dintr-un drum, constat c i-au mai
rmas de parcurs cu 4,8 km mai mult dect a parcurs. Ce lungime are drumul i ct a
parcurs turistul?
Soluie. Dac dintr-un drum se parcurg 38%, rezult c rmne din el de
parcurs 62%, deci 4,8km reprezint diferena dintre partea rmas i partea parcurs,
deci 24% din drum. Avem

24
x = 4,8 , unde x este lungimea drumului. Rezult
100

x = 20 , drumul are o lungime de 20km.


38
Turistul a parcurs
20 = 7,6 km.
100

R4.4.3. Dup dou reduceri consecutive de preuri, prima de 10%, iar a doua
de 20%, un obiect cost 153000lei. Care a fost preul iniial al acestui obiect?
Soluie. Notm cu x preul iniial al obiectului. Prima reducere de pre este

10
90
x i preul obiectului dup prima reducere este de
x ; a doua reducere este
100
100

41

20 90
18

x =
x , iar dup a doua reducere costul obiectului este
100 100
100
90
18
72
x
x=
x , ceea ce reprezint 153000lei.
100
100
100
72
Avem
x = 153000 , de unde rezult x = 212500 , deci preul iniial al
100

de

obiectului a fost 212500lei.


Remarc. Problema poate fi rezolvat i folosind metoda mersului invers.
Preul final, 153000lei reprezint 80% din preul obiectului dup prima ieftinire. Se
poate calcula preul dup prima ieftinire 153000 :
191250lei

reprezint

90%

din

preul

iniial.

80
= 191250 lei. Preul de
100
Calculm

preul

iniial

90
191250 :
= 212500 lei.
100
R4.4.4. Un autocar are de parcurs un traseu n patru etape, astfel: n prima
etap parcurge 30% din traseu, n a doua etap parcurge 20% din rest, n a treia etap
25% din noul rest i i mai rmn pentru a patra etap 126km de parcurs. Ce lungime
are drumul?
Soluie. Se noteaz cu x lungimea drumului. n prima etap se parcurge

30
70
20 70
14
x , rest
x ; n a doua etap se parcurge

x =
x , rest
100
100
100 100
100
70
14
56
25 56
14
x
x=
x ; n a treia etap se parcurge

x =
x , rest
100
100
100
100 100
100
56
14
42
42
x
x=
x , ceea ce reprezint 126km. Avem
x =126, de unde
100
100
100
100
100
x = 126
, x = 300 . Lungimea drumului a fost de 300km.
42
R4.4.5. Numrul bc reprezint 4% din numrul abc . S se calculeze
a + b + c (b 0) .
4
1
abc , deci bc = (100a + bc) , de unde rezult c
Soluie. tim c bc =
100
25
1
24
bc = 4a + bc , adic
bc = 4a . De aici se deduce c bc M 25 , pentru c 4a este
25
25
natural.
Dac bc = 25 rezult c 24=4a, deci a=6, a + b + c = 13 . Dac bc = 50 sau

bc = 75 nu se obine a cifr.

42

4.5. Titlul unui aliaj


Definiie. Titlul unui aliaj este raportul dintre masa metalului preios coninut
de aliaj i masa aliajului.

Titlul aliajului =

masa metalului pretios


m
, deci T =
.
masa aliajului
M

Observaie. Asemntor titlului unui aliaj, se poate defini concentraia unei


soluii (amestec).
Concentraia soluiei (amestecului) =

masa substantei
masa solutiei (amestecului)

Model.
1. Se face un aliaj, topind la un loc, 16g aur i 234g cupru. Care este titlul
aliajului?
Soluie. Titlul aliajului =

masa metal pretios


16
16
, deci T =
=
, de unde
masa aliajului
16 + 234 250

rezult titlul aliajului 0,064.


2. Concentraia de sare dintr-o soluie este 17%. Ce cantitate de sare se gsete
n 27,5kg de soluie?
Soluie. Concentraia soluiei reprezint raportul dintre masa substanei i masa
soluiei. Avem

17
x
=
, de unde x 100 = 17 27,5 , deci x = 4,675 . n 27,5kg
100 27,5

soluie se afl 4,675g sare.


Probleme de amestec i aliaje
Frecvent n practic se ntlnesc probleme de acest tip. n funcie de datele i
cerinele lor n general, aceste probleme se mpart n dou categorii.
Probleme de amestec i aliaj de categoria I
n aceste probleme se cunosc:
a) cantitile care se amestec: m1 , m2 ,..., mn
b) calitile lor: c1 , c2 ,..., cn .
Se cere: c) calitatea amestecului.
Calitatea diverselor produse, substane, aliaje etc. care se amestec, se exprim
prin: grade de temperatur, lei, grade de trie sau n cazul aliajelor prin titlu.
Teorema 4.5.1. Dac amestecm produse de calitile c1 , c2 ,..., cn n
cantitile m1 , m2 ,..., mn (n N) , atunci calitatea amestecului este dat de relaia:
43

C=

m1 c1 + m2 c2 + ... + mn cn
m1 + m2 + ... + mn

(1)

Demonstraie. Vom demonstra teorema n ipoteza c produsele respective sunt


aliaje cu titlurile t1 , t 2 ,..., t n (deci c1 = t1 , c2 = t 2 ,..., cn = t n ) i n cantitile

m1 , m2 ,..., mn . Deci s artm c titlul noului aliaj este:


m t + m2 t 2 + ... + mn t n
T= 1 1
m1 + m2 + ... + mn
Fie t1 =

m'
m1'
m'
, t 2 = 2 ,..., t n = n , unde m1' , m2' ,..., mn' sunt cantitile de
m1
m2
m2

metal preios din fiecare aliaj. Masa total a metalului preios din aliajul obinut prin
topire la un loc a aliajelor date este:

m = m1' + m2' + ... + mn' = m1 t1 + m2 t 2 + ... + mn t n


Masa total a aliajului nou obinut este M = m1 + m2 + ... + mn . Deci titlul
noului aliaj este:

T=

m m1 t1 + m2 t 2 + ... + mn t n
=
M
m1 + m2 + ... + mn

Observaii. 1) Expresia (1) exprim media aritmetic ponderat a numerelor


c1 , c2 ,..., cn care au ponderile m1 , m2 ,..., mn .
2) Media aritmetic ponderat se obine de fapt ca o medie aritmetic obinuit
innd seama c fiecare numr intr n aceast medie cu o anumit pondere.
3) Media aritmetic a unor numere este o medie aritmetic ponderat n care
fiecare pondere este egal cu 1.
Probleme de amestec i aliaj de categoria a II-a
n aceste probleme se cunosc:
a) calitile produselor care se amestec
b) calitatea amestecului
c) cantitatea total a amestecului.
Se cer: d) cantitile care se amestec.
Teorema 4.5.2. Dac amestecm dou produse de caliti c1 , respectiv c2 , n
cantitile m1 , respectiv m2 i obinem un amestec de calitate c, atunci are loc relaia:

c c2 m1
=
c1 c m2

44

(2)

Demonstraie. Din teorema (1) obinem c =

m1c1 + m2 c2
, care prin nlocuire
m1 + m2

n relaia (2) conduce la o propoziie adevrat:

m1c1 + m2 c2
c2
m1 + m2
m c + m2 c2 m1c2 m2 c2 m1 (c1 c2 ) m1
c c2
=
=
= 1 1
=
m1c1 + m2 c2
m1c1 + m2 c1 m1c1 m2 c2 m2 (c1 c2 ) m2
c1 c
c1
m1 + m2

Probleme rezolvate
R4.5.1. Se amestec 5kg de bomboane cu preul 54000lei/kg cu 2kg de
bomboane cu preul de 48000lei/kg i cu 3kg de bomboane cu preul de 66000lei/kg.
Ct este preul unui kilogram de bomboane ce rezult n urma amestecului celor trei
caliti de bomboane?
Soluie. Folosim relaia (1) i obinem preul unui kilogram de amestec:

5 54000 + 2 48000 + 3 66000


= 56400 lei.
5+ 2+3

R4.5.2. Un aliaj de fier i nichel are titlul de 0,600, iar un alt aliaj din aceleai
metale are titlul 0,250. Se topesc aceste aliaje mpreun i rezult un alt aliaj cu masa
de 14kg. Ct este masa fiecrui aliaj, dac titlul noului aliaj este 0,300?
Soluie. Vom folosi formula (2), unde c = 0,300 , c1 = 0,600 , c2 = 0,250 ,

m1 este masa primului aliaj, m2 este masa celui de al doilea aliaj. Deci:
m
m
0,300 0,250 m1
1
1
=
. Obinem 1 = . tiind c m1 + m2 = 14 i 1 = ,
0,600 0,300 m2
m2 6
m2 6
obinem m1 = 2 kg i m2 = 12 kg.
Remarc. Problema se poate rezolva i cu ajutorul formulei (1). Fie m1 , m2

m1 0,600 + m2 0,250
= 0,300 i
m1 + m2
m1 + m2 = 14 . Dac m2 = 14 m1 , avem m1 0,6 + (14 m1 ) 0,25 = 0,3 14 , de
unde m1 0,6 m1 0,25 = 0,7 , deci rezult m1 = 2 i m2 = 12 .

masele celor dou aliaje folosite. Vom avea

R4.5.3. Se topesc mpreun dou aliaje formate din aceleai metale, care au
masele de 3kg i respectiv 2kg. Titlul primului aliaj este 0,150, iar titlul noului aliaj
este 0,400. Aflai titlul celui de al doilea aliaj.
Soluie. Aplicm formula (1), unde m1 = 3 kg, t1 = 0,150 , m2 = 2 kg i

T = 0,400 .

Avem:

T=

m1t1 + m2 t 2
,
m1 + m2

45

iar

prin

nlocuire

se

obine:

3 0,150 + 2 t 2
= 0,400 . Efectund calculele 2t 2 + 0,45 = 2 , de unde rezult c
3+ 2
t 2 = 0,775 . Titlul celui de al doilea aliaj este 0,775.
R4.5.4. O soluie de ap cu alcool cntrete 600g i are concentraia de 0,250.
Ct alcool trebuie s adugm pentru a se obine o soluie cu concentraia de 0,400?
Soluie. Se calculeaz cantitatea de alcool existent n 600g soluie, innd cont
de definiia concentraiei (raport dintre masa alcoolului i masa soluiei). Avem:

a
, de unde a = 150 g alcool. Notm cu x cantitatea de alcool care se
600
150 + x
, de
adaug pentru a obine o soluie de concentraie 0,400 i avem: 0,400 =
600 + x
unde 150 + x = 0,4 x + 240 , deci 0,6 x = 90 , iar x = 150 . Trebuie s adugm 150g
0,250 =

de alcool pentru a obine o soluie de concentraie 0,400.


R4.5.5. Un inel din aur de 14 carate are 6g. Printr-o nou prelucrare inelul are
18 carate. S se afle masa inelului dup prelucrare.
Soluie. Facem precizarea c n tehnic, atunci cnd metalul preios dintr-un
aliaj este aurul, titlul se exprim n carate (k). Aurul pur are titlul 24k, deci dac un
aliaj are titlul 18k, nseamn c din ntreaga mas a aliajului 18 pri sunt aur, iar 6
pri sunt din metal nepreios; titlul este

18
= 0,750 sau 18k.
24

n cazul acestei probleme se pot ivi dou situaii:


a) Printr-un procedeu oarecare se separ metalul nepreios din coninutul
inelului i se ndeprteaz din acesta o cantitate, astfel nct aliajul respectiv s aib
18k.
Fie x cantitatea de metal nepreios care se ndeprteaz pentru ca inelul s aib

14
18
6g = 3,5 g aur. Deci, (6 x)
= 3,5 , de unde se
24
24
1
2
1
obine x = 1 g. Inelul va cntri 6 1 = 4 g.
3
3
3

titlul de 18k. Inelul conine:

b) Se adaug aur pur astfel nct aliajul obinut s aib titlul de 18k. Fie y
cantitatea de aur pur ce trebuie adugat. Deci, (6 + y )

18
= 3,5 + y . Rezolvnd
24

aceast ecuaie se obine y = 4 . Inelul va avea n final masa 6+4=10g.

4.6. Proporii
Definiie. Egalitatea a dou rapoarte se numete proporie.
Termenii celor dou rapoarte se numesc termenii proporiei.
Orice proporie are patru termeni.
46

Forma general a unei proporii este:

a c
= , a, b, c, d Q , b 0 , d 0 .
b d

Termenii a i d se numesc extremii proporiei.


Termenii b i c se numesc mezii proporiei.
Proprietatea fundamental a proporiilor: n orice proporie produsul extremilor
este egal cu produsul mezilor.
Aflarea unui termen necunoscut al unei proporii: fiind dat proporia

a c
= ,
b d

conform proprietii fundamentale a proporiilor a d = b c , de unde rezult c

a=

bc
bc
ad
ad
, d=
, b=
i c =
.
d
a
c
b
produsul mezilor
Deci: un extrem =
celalalt extrem
produsul extremilor
un mez =
celalalt mez

Definiie. Unul dintre extremii sau mezii, egali ntre ei, ai unei proporii se
numete media proporional (geometric) a celorlali doi termeni.
Proporii derivate cu aceeai termeni
Regul. Dac ntr-o proporie se schimb extremii ntre ei lsnd mezii
neschimbai, se obine tot o proporie, numit proporie derivat cu aceiai termeni ca
proporia iniial.
Din proporia

a c
d c
= , aplicnd regula de mai sus obinem proporia = cu
b d
b a

aceiai termeni ca proporia iniial.


Regul. Dac ntr-o proporie se schimb mezii ntre ei lsnd extremii
neschimbai, se obine tot o proporie, numit proporie derivat cu aceiai termeni ca
proporia iniial.
Din proporia

a c
a b
= , aplicnd regula de mai sus obinem proporia = cu
b d
c d

aceiai termeni ca proporia iniial.


Regul. Dac ntr-o proporie se schimb extremii ntre ei i mezii ntre ei, se
obine tot o proporie, numit proporie derivat cu aceiai termeni ca proporia iniial.
Din proporia

a c
d b
= , aplicnd regula de mai sus obinem proporia = cu
b d
c a

aceiai termeni ca proporia iniial.


Remarc. Ultima regul de obinere a proporiilor derivate cu aceiai termeni
se mai poate enuna i astfel: dac ntr-o proporie se inverseaz rapoartele se obine tot
o proporie numit proporie derivat cu aceiai termeni ca proporia iniial.
47

Observaie. n general, fiind date patru numere distincte a, b, c, d , care


formeaz proporia

d c
= ,
b a
b d
2) = ,
a c

1)

a c
= , cu aceste numere se mai pot forma proporiile:
b d
a b d b
= . = i
c d c a
c d b a c a
= , = , = .
a b d c d b

Ultimele proporii sunt identice cu cele de la 1), datorit simetriei relaiei de


egalitate.
Model. Scriei toate proporiile cu termenii: 3, 6, 7, 14.
Soluie. Se constat c 314=67. Dac 3 i 14 sunt extremi, iar 6 i 7 sunt

3 7
= . Obinem:
6 14
14 7
= (prin schimbarea extremilor ntre ei)
6 3
3 6
(prin schimbarea mezilor ntre ei)
=
7 14
14 6
= (prin inversarea rapoartelor)
7 3

mezi, avem

Dac 3 i 14 sunt mezi, iar 6 i 7 sunt extremi, avem

6 14
= . Obinem:
3 7

7 14
(prin schimbarea extremilor ntre ei)
=
3 6
6 3
= (prin schimbarea mezilor ntre ei)
14 7
7 3
= (prin inversarea rapoartelor).
14 6
Proporii derivate cu ali termeni
Fie proporia

ad = bc .

a c
= . Conform proprietii fundamentale a proporiilor, avem
b d

Regula 1. Dac amplificm unul din rapoartele unei proporii cu un numr,


diferit de zero, obinem o proporie cu ali termeni.

48

Avem a d = b c . nmulind ambii membri ai egalitii cu numrul n, obinem

adn = bcn , de unde rezult

a cn
an c
sau
=
= .
b dn
bn d

Prin procedeul indicat de regula 1 se poate obine o infinitate de proporii cu


ali termeni dect cei ai proporiei iniiale.
Exemplu. Fie proporia

5
3
= . Prin amplificarea primului raport cu 2,
20 12

10 3
= , o proporie cu ali termeni. Prin amplificarea celui de al doilea
40 12
5
30
, o proporie cu ali termeni.
raport cu 10, obinem
=
20 120

obinem

Regula 2. Dac simplificm unul din rapoartele unei proporii cu un numr,


diferit de zero, obinem o proporie cu ali termeni.

a d
= , avem a d = b c . nmulim ambii membri ai egalitii
b d
1
a d bc
b
a
(n 0) , obinem
=
, ceea ce se poate scrie
cu
d = c sau
n
n
n
n
n
a:n c
a c:n
.
= . Asemntor, =
b:n d
b d :n
9
6
Exemplu. Fie proporia
= . Prin simplificarea primului raport cu 3,
15 10
3 6
obinem
= , o proporie cu ali termeni. Prin simplificarea celui de al doilea
5 10
9 3
raport cu 2, obinem
= , o proporie cu ali termeni.
15 5
Fie proporia

Regula 3. Dac nmulim ambii numrtori (sau ambii numitori) ai unei


proporii cu un numr, diferit de zero, obinem tot o proporie, dar cu ali termeni.

a c
= , avem a d = b c . nmulim ambii membri ai proporiei
b d
1
cu n i obinem adn = bcn , de unde prin nmulirea ambilor membri cu
i
bd
adn bcn
an cn
realizarea simplificrilor, obinem
, adic
=
= .
bd
bd
b
d
1
i
Asemntor din ad = bc , prin nmulirea ambilor membri cu
bdn
ad
bc
a
c
efectuarea simplificrilor, obinem
, adic
.
=
=
bdn bdn
bn dn
Fie proporia

49

Exemplu. Fie proporia

2 10
.
=
5 25

Prin nmulirea numrtorilor cu 3, obinem


numitorilor cu 4, obinem

6 30
, iar prin nmulirea
=
5 25

2
10
, proporii derivate cu ali termeni.
=
20 100

Regula 4. Dac mprim ambii numrtori (sau ambii numitori) ai unei


proporii cu un numr, diferit de zero, obinem tot o proporie, dar cu ali termeni.

a c
1
= , avem a d = b c . nmulind ambii membri cu
b d
n
ad bc
c
a
(n 0) , obinem
= , ceea ce se poate scrie d = b . Dup nmulirea
n
n
n
n
c
a
d b
1
i realizarea simplificrilor, obinem n
ambilor membri cu
= n , adic
bd
bd
bd
a:n c:n
.
=
b
d
1
Asemntor din ad = bc , nmulind ambii membri cu , (n 0) , obinem
n
1
d b
i efectuarea simplificrilor
a = c . Dup nmulirea ambilor membri cu
b d
n n

n n
d
b
a
c
n = n , adic a = c .
obinem:
b d b d
b:n d :n

n n n n
4 8
.
Exemplu. Fie proporia =
6 12
1 2
= , iar prin mprirea
Prin mprirea numrtorilor cu 4, obinem
6 12
4 8
numitorilor cu 3, obinem = , proporii derivate cu ali termeni.
2 4
Fie proporia

Observaii. Cu ajutorul celor patru reguli se pot obine o infinitate de proporii


derivate cu ali termeni dect ai proporiei iniiale.
Fie proporia

a c
= . Conform celor patru reguli se pot obine urmtoarele
b d

proporii derivate cu ali termeni:


50

a cn an c a : n c a c : n an cn a
c a:n c:n
=
= ;
= , =
= ;
=
=
;
,
;
;
b dn bn d b : n d b d : n b
d bn dn
b
d
a
c
=
(n 0) .
b:n d :n
a c
= , din ea se pot deduce urmtoarele
Proprietate. Fiind dat proporia
b d
proporii derivate cu ali termeni:

a
c
=
b+a d +c
a c
= , avem a d = b c , conform proprietii fundamentale a
Fie proporia
b d
proporiilor. Adunnd la ambii membri produsul ac, rezult ad + ac = bc + ac , de
unde scond factor comun, a (d + c) = c(b + a ) . mprind ambii membri cu
a
c
a (d + c)
c(b + a)
=
(b + a)(d + c) avem
=
, adic
.
(b + a)(d + c) (b + a)(d + c)
b+a d +c
a+b c+d
=
. Se demonstreaz asemntor, adunnd produsul bd n
P.2.
b
d
ambii membri ai egalitii ad = bc . Avem ad + bd = bc + bd , scoatem factor comun
1
d (a + b) = b(c + d ) , iar dup nmulirea ambilor membri cu
, obinem
bd
d ( a + b ) b (c + d )
a+b c+d
=
, adic
=
.
bd
bd
b
d
a
c
=
P.3.
( a b 0, c d 0 ). Se demonstreaz asemntor
a b cd
celorlalte. Se scad din ac ambii membri ai egalitii ad = bc , rezult
ac ad = ac bc , scond factor comun obinem a(c d ) = c(a b) . mprind
a (c d )
c ( a b)
=
, adic
ambii membri cu (c d )(a b) , avem
(c d )(a b) (c d )(a b)
a
c
=
.
a b cd
a b cd
=
P.4.
. Se demonstreaz scznd produsul bd din ambii membri ai
b
d
egalitii ad = bc , rezult ad bd = bc bd , scond factor comun avem
d (a b) = b(c d ) , iar dup mprirea ambilor membri cu bd, avem
d ( a b) b(c d )
a b cd
=
, adic
=
.
bd
bd
b
d
P.1.

51

P.5.

a+b c+d
=
a b c d

( a b 0 , c d 0 ). Se demonstreaz mprind

a+b c+d
membru cu membru egalitile de la P.2 i P.4, adic: b = d , care se poate
a b c d
b
d
a+b b
c+d d
a+b c+d

, adic
=
.
scrie
b a b
d cd
a b c d
a b c d
=
P.6.
( a + b 0 , c + d 0 ). Se demonstreaz inversnd
a+b c+d
rapoartele n proporia de la P.5.
P.7.

a +b a b
=
( c + d 0, c d 0 ). Se demonstreaz schimbnd mezii
c+d cd

ntre ei n proporia de la P.5.


Model. Fie proporia

12 6
= . S se obin proporii derivate cu ali termeni.
8 4

Soluie.

12 3 6
12 18
= .
=
i obinem
8 3 4
8 12
12 : 4 6
3 6
= i obinem = .
Aplicnd R.2 (n=4), avem
8: 4 4
2 4
12 5 6 5
60 30
=
=
i obinem
.
Aplicnd R3. (n=5), avem
8
4
8
4
12
6
12 6
=
= .
Aplicnd R.4 (n=2), avem
i obinem
8: 2 4: 2
4 2
12
6
12 6
=
= .
Aplicnd P.1, avem
i obinem
8 + 12 4 + 6
20 10
12 + 8 6 + 4
20 10
=
= .
i obinem
Aplicnd P2., avem
8
4
8
4
12
6
12 6
=
= .
Aplicnd P.3, avem
i obinem
12 8 6 4
4 2
12 8 6 4
4 2
=
Aplicnd P.4 avem
i obinem = .
8
4
8 4
12 + 8 6 + 4
20 10
=
= .
i obinem
Aplicnd P.5 avem
12 8 6 4
4
2
12 8 6 4
4
2
=
= .
Aplicnd P.6, avem
i obinem
12 + 8 6 + 4
20 10
Aplicnd R.1 (n=3), avem

52

Aplicnd P.7, avem

12 + 8 12 8
20 4
= .
=
i obinem
6+4 64
10 2

Probleme rezolvate

2a + 3b
a
= 0,6 . S se afle
.
3b
b
a 3
2a 6
= ,
Soluia 1. Din = , prin nmulirea numrtorilor cu 2, vom avea
b 5
b 5
2a 6
2a 2
=
= . Adunm
iar prin nmulirea numitorilor cu 3, obinem
sau
3b 15
3b 5
2a + 3b 2 + 5
2a + 3b 7
= .
=
, de unde
numitorii la numrtor i obinem
3b
5
3b
5
a 3
a b
Soluia 2. Din = , schimbnd mezii ntre ei obinem = = k (s-a notat
b 5
3 5
a
b
a
b
prin k valoarea rapoartelor
i ). Din = k rezult a = 3k , iar din = k rezult
3
5
3
5
2a + 3b 2 3k + 3 5k 21k 7
=
= .
=
b = 5k . Atunci:
3b
3 5k
15k 5
a 3
3b
Soluia 3. Din = rezult a =
i atunci:
b 5
5
6
3b
2 + 3b b + 3
2a + 3b
5 21 1 7
5
=
=
= = .
3b
3b
3b
5 3 5
x y
R4.6.2. S se afle numerele naturale x i y, diferite de 0, astfel ca =
i
2 3
4 x + y + 5( x + y )
= 3.
24
4 x + y + 5( x + y )
Soluia 1. Din
= 3 , obinem prin efectuarea calculelor de la
24
9x + 6 y
3(3x + 2 y )
3x + 2 y
numrtor
= 3 sau
= 3 , adic
= 3 , de unde rezult c
24
24
8
x y
=
3x + 2 y = 24 . Dar
i conform proprietii fundamentale a proporiilor
2 3
R4.6.1. Se d

53

3x = 2 y , care se nlocuiete n relaia precedent obinndu-se 2 y + 2 y = 24 sau


2y
y = 6 . Dar x =
, deci x = 4 .
3
x y
Soluia 2. Notm
= = k deci obinem x = 2k i y = 3k . Atunci
2 3
4 x + y + 5( x + y )
4 2k + 3k + 5(2k + 3k )
= 3 devine
= 3 , iar dup efectuarea
24
24
36k
calculelor obinem
= 3 , de unde k = 2 . Din x = 2k i y = 3k vom obine
24
x = 4 , y = 6.
R4.6.3. S se afle trei numere, tiind c raportul dintre primul i al doilea este
0,(6), raportul dintre al doilea i al treilea este 0,8(3), iar produsul dintre primul i al
treilea numr este 9331,2.
Soluie. Notm n ordine cele trei numere cu x, y, z. Din datele problemei,

x 2 y 5
x 2
z 6
=
= ,
=
= , prin nmulire
i xz = 9331,2 . Din
i
y 3 z 6
y 3
y 5
9331,2 4
xz 4
= , de unde rezult
= , deci
membru cu membru se obine
2
5
5
y
y2
x 2
= ,
y 2 = 11664 , sau y 2 = (2 2 33 ) 2 . Obinem y = 108 sau y = 108 . Din
y 3
2y
y 5
6y
= , rezult z =
, deci x = 72 sau x = 72 . Din
, deci
rezult x =
3
z 6
5
z = 129,6 sau z = 129,6 .
20a + 9b
= 5 , s se arate c a este 20% din b.
R4.6.4. tiind c
3b 2a
Soluie. Din relaia dat rezult c 20a + 9b = 5(3b 2a ) , iar dup efectuarea
a 6
a 1
=
= . Se
calculelor 20a + 9b = 15b 10a , de unde 30a = 6b sau
, deci
b 30
b 5
a b
schimb mezii ntre ei i se obine = = k , de unde a = k i b = 5k . Vom avea:
1 5
x
x
b = a , adic
5k = k , de unde x = 20 , deci 20% din a reprezint b.
100
100

obinem

R4.6.5. S se afle ariile a dou dreptunghiuri, tiind c raportul lungimilor lor


este

4
7
, raportul limilor lor este , iar diferena ariilor este 4.
3
9

54

Soluie. Notm L i L' lungimile celor dou dreptunghiuri i l, l' limile celor

L 4
l 7
= i raportul limilor = .
L' 3
l' 9
L 4
l 7
= i = ,
Diferena ariilor celor dou dreptunghiuri este Ll L' l ' = 4 . Din
L' 3
l' 9
Ll 28
=
, apoi facem proporii derivate
prin nmulirea membru cu membru, obinem
L' l ' 27
Ll L' l ' 28 27
4
1
=
=
, adic
, deci L' l ' = 108 . Din Ll 108 = 4 , rezult
L' l '
27
L' l ' 27
Ll = 112 . Deci ariile celor dou dreptunghiuri sunt 112 i 108.
1
R4.6.6. Suma a dou fracii cu acelai numrtor este 1 . Raportul
15
1
numitorilor este . S se afle cele dou fracii.
3
a
a
a a 16
Soluie. Fie
i
cele dou fracii (b, c 0) . Avem + =
, de unde
b
c
b c 15
a(b + c) 16
b 1
= . Raportul numitorilor este
= , de unde proporia
rezult c
bc
15
c 3
b+c 4
a 4 16
a 4
= . Prin nlocuire n relaia dinainte, avem = , de unde = . Suma
c
3
b 3 15
b 5
16
a 16 4 a 4
4
4
, = . Fraciile sunt i
celor dou fracii este
, deci =
.
15
c 15 5 c 15
5 15
dou dreptunghiuri. Avem raportul lungimilor

4.7. ir de rapoarte egale


Definiie. Un ir de rapoarte cu aceeai valoare, scrise sub forma

a c e
= = = ... , se numete ir de rapoarte egale.
b d f
Observaii. 1) Orice pereche de rapoarte din ir formeaz o proporie.
2) Amplificnd succesiv un raport cu mai multe numere diferite de zero, se
obine un ir de rapoarte egale:

a an ak
=
=
= ...
b bn bk
a an ak
=
Fie irul de rapoarte egale =
. S considerm raportul dintre suma
b bn bk
a + an + ak
. Scoatem factorul comun a la
numrtorilor i suma numitorilor
b + bn + bk
55

numrtor i b la numitor i obinem


Deci,

a
a(1 + n + k )
, care se simplific i rezult .
b(1 + n + k )
b

a an ak a + an + ak
=
=
=
.
b bn bk b + bn + bk

Proprietatea irului de rapoarte egale. ntr-un ir de rapoarte egale, raportul


dintre suma numrtorilor i suma numitorilor este egal cu fiecare din celelalte
rapoarte.

a c e
a c e a+c+e
= = , atunci = = =
.
b d f
b d f b+d + f
a c e 3
Model. tiind c = = = , s se calculeze:
b d f 4

n general, dac

a+c+e
3a + 4c + 5e
a 2 + c2 + e2
; b)
; c) 2
.
b+d + f
3b + 4d + 5 f
b +d2 + f 2
a 3
a c e
a c e a+c+e
Soluie. a) Dac = = , atunci = = =
, dar = ,
b d f
b d f b+d + f
b 4
a+c+e 3
= .
deci
b+d + f 4
a c e
b) Dac = = , atunci prin amplificarea primului raport cu 3, al celui de
b d f
3a 4c 5e
=
=
, de unde rezult c
al doilea cu 4 i al celui de al treilea cu 5, se obine
3b 4d 5 f
3a a 3
3a 4c 5e 3a + 4c + 5e
3a + 4c + 5e 3
= = , deci
=
=
=
= .
, dar
3b 4d 5 f 3b + 4d + 5 f
3b + 4d + 5 f 4
3b b 4
a)

c) Dac

a c e 3
9
a2 c2 e2
= = = , rezult c 2 = 2 = 2 = , prin ridicarea la
b d f 4
16
b
d
f

ptrat a fiecrui raport. Rezult, aplicnd proprietatea irului de rapoarte egale c

a2 c2 e2
a2 + b2 + c2
a2 9
, dar 2 =
, deci
=
=
=
b2 d 2 f 2 b2 + d 2 + f 2
16
b
a c e
= = = k , atunci
Remarc. Dac
b d f
ma + nc + pe
= k , m, n, p 0 .
mb + nd + pf

56

9
a2 + b2 + c2
= .
2
2
2
16
b +d + f
ma nc pe
=
=
= k , de unde
mb nd pf

Probleme rezolvate

2a 3b 5c
=
=
i c a + b + c = 119 , s se afle a, b, c.
3
5
4
2a 3b 5c
a b c
=
= , atunci = = , de unde rezult aplicnd
Soluia 1. Dac
3 5 4
3
5
4
2 3 5
R4.7.1. tiind c

proprietatea irului de rapoarte egale

a b c
a + b + c 119
30
= = =
=
= 119
= 30 .
3 5 4 3 5 4 119
119
+ +
2 3 5 2 3 5
30

Din

a
b
c
= 30 , rezult a=45, din = 30 , rezult b=50 i din = 30 , rezult c=24.
3
5
4
2
3
5
Soluia 2. Notm valoarea comun a rapoartelor cu k i avem:

3k
5k
4k
2a 3b 5c
=
=
= k , de unde a =
, b=
, c=
. nlocuind n a + b + c = 119 ,
2
3
5
3
5
4
3k 5k 4k
+
+
= 119 , de unde se obine dup efectuarea calculelor
se obine
2
3
5
111k
3 30
5 30
4 30
= 119 , adic k=30. Atunci a =
= 45 , b =
= 50 i c =
= 24 .
30
2
3
5
x y y z
= i
R4.7.2. S se determine numerele x, y, z naturale, tiind c = ,
3 8 6 2
a) x + y + z = 123 ; b) 5 x + y 8 z = 25 ; c) x 2 + y 2 + z 2 = 6489 .
x y
y z
=
= se poate forma un ir de
i
Soluie. Fiind date proporiile
3 8
6 2
rapoarte egale astfel: nmulim numitorii primei proporii cu 3 i nmulim numitorii
celei de a doua proporii cu 4, vom obine dou proporii derivate cu ali termeni i
anume,

x y
y z
x y z
=
= , de unde rezult =
= .
i
9 24
24 8
9 24 8
a) Aplicnd proprietatea irului de rapoarte egale, obinem:

x y z x + y + z 123
=
= =
=
= 3 , de unde x=27, y=72, z=24.
9 24 8 9 + 24 + 8 41

57

x y z
=
= = k , k fiind valoarea comun a fiecrui raport, atunci
9 24 8
x = 9k , y = 24k , z = 8k . Prin nlocuire n 5 x + y 8 z = 25 , se obine
45k + 24k 64k = 25 , adic 5k = 25 , de unde k=5. Avem x = 9 5 = 45 ,
y = 24 5 = 120 i z = 8 5 = 40 .
x y z
= , atunci prin amplificarea primului raport cu 5 i
Remarc. Dac =
9 24 8
5x y 8z
=
=
al celui de al treilea lui 8, obinem
, de unde aplicnd proprietatea
45 24 64
b) Avem

irului de rapoarte egale se obine

5 x y 8 z 5 x + y 8 z 25
=
=
=
=
= 5.
45 24 64 45 + 24 64 5
x
y
z
Avem = 5 , deci x=45,
= 5 , deci y=120 i = 5 , deci z=40.
9
24
8
2
2
x y z
x
y
z2
=
= , atunci
=
=
c) Dac
i aplicnd proprietatea irului
9 24 8
81 576 64
x2
y2
z2
x 2 + y 2 + z 2 6489
x2
=
=
=
=
= 9 . Avem
=9, x
de rapoarte egale
81 576 64 81 + 576 + 64 721
81
x
y2
y
= 3 , de unde x=27,
= 9 , y natural, deci
= 3 , de unde y=72,
natural, deci
9
576
24
z2
z
= 9 , z natural, deci = 3 , de unde z=24.
64
8
x y z
Remarc. Dac
=
= = k , atunci x = 9k , y = 24k , z = 8k . Prin
9 24 8
nlocuire n x 2 + y 2 + z 2 = 6489 , vom avea 81k 2 + 576k 2 + 64k 2 = 6489 adic
721k 2 = 6489 , de unde k=3. Obinem x=27, y=72, z=24.
x
y
y
z
=
=
i
cu x 0 , y 0 , z 0 . S se
R4.7.3. Fie
0, (1) 0, (3)
0, (5) 0, (7)
afle x, y, z, tiind c x + y + z = 123 .
x y
y z
=
= . nmulim
Soluie. Efectund transformrile, se obine
i
1 1
5 7
9 3
9 9
1
1 x y 1
x y
i obinem = , adic =
i
ambii membri ai celor dou egaliti cu
9
9 1 1 9
1 3
9 3
58

1 y 1 z
y z
x y
1
= , adic = . n egalitatea = nmulim fiecare membru cu
i
9 5 9 7
5 7
1 3
5
9
9
1 x 1 y
x y
(1).
obinem = , adic =
5 1 5 3
5 15
y z
1
1 y 1 z
n egalitatea = , nmulim fiecare membru cu i obinem = ,
5 7
3
3 5 3 7
y
z
=
(2).
adic
15 21
x y
z
= =
Din (1) i (2) rezult irul de rapoarte egale
i aplicnd
5 15 21
proprietatea irului de rapoarte egale avem

x y
z
x + y + z 123
= =
=
=
= 3.
5 15 21 5 + 15 + 21 41
x
y
z
Deci, = 3 , de unde x=15,
= 3 , de unde y=45 i
= 3 , de unde z=63.
5
15
21
R4.7.4. Fie a, b, c trei numere nenule, astfel nct:
2a = 5b = 9c = x(a + b + c) .
S se determine valoarea lui x.

a b c a+b+c
= = =
. Aplicnd
1 1 1
1
2 5 9
x
a b c a+b+c
= = =
, rezult c
proprietatea irului de rapoarte egale:
1 1 1 1 1 1
+ +
2 5 9 2 5 9
1 1 1 1
1 73
90
= + + , adic = , de unde x = .
x 2 5 9
x 90
73
a b c
R4.7.5. S se afle numere a, b, c, tiind c = = i abc = 576 .
3 4 6
a b c
Soluie. Notm
= = = k , de unde a = 3k , b = 4k i c = 6k .
3 4 6
nlocuind n abc = 576 , se obine 3k 4k 6k = 576 , de unde rezult k 3 = 8 , deci
k=2. Avem a = 3 2 = 6 , b = 4 2 = 8 , c = 6 12 = 72 .
Soluie. Relaia dat se poate scrie

59

4.8. Proporionalitate direct. Proporionalitate invers


Definiie. ntre dou mulimi finite de numere exist o proporionalitate
direct, dac se poate forma un ir de rapoarte egale, diferite de zero, astfel nct
numrtorii rapoartelor s fie elementele unei mulimi, iar numitorii elementele
celeilalte mulimi.
Exemplu. ntre mulimile {2,6,4} i {10,30,20} se stabilete o
proporionalitate direct, deoarece

2
6
4
=
=
.
10 30 20

Observaie. Dac elementele unei mulimi A finite de numere se pot obine


prin nmulirea elementelor unei mulimi B cu un numr dat n (n0), atunci ntre cele
dou mulimi exist o proporionalitate direct.
ntr-adevr, fie mulimea B={a,b,c}. Prin nmulirea elementelor ei cu numrul
n (n0), obinem mulimea A={an,bn,cn}. Cu elementele celor dou mulimi, A i B, se
poate forma un ir de rapoarte egale

a
b
c
1
=
=
(valoarea rapoartelor este
);
an bn cn
n

deci ntre cele dou mulimi A i B am stabilit o proporionalitate direct.


Exemple. 1) ntre mulimea ciocolatelor i mulimea costurilor lor se stabilete
o proporionalitate direct.
2) ntre viteza de deplasare i spaiul parcurs de un mobil n micare uniform,
se stabilete o proporionalitate direct.
3) ntre spaiul parcurs de un mobil cu vitez constant i timpul n care se
efectueaz deplasarea se stabilete o proporionalitate direct.
4) ntre numrul de robinete cu acelai debit i volumul de lichid acumulat se
stabilete o proporionalitate direct.
Model. S se determine trei numere direct proporionale cu 3, 9, 12, dac suma
lor este 40.
Soluie. Fie x, y, z cele trei numere. Vom avea
Aplicnd

proprietatea

irului

de

x y z
= =
i x + y + z = 40 .
3 9 12
rapoarte

egale,

avem

x y z
z + y + z 40 5
35
95
=
= . Se deduce c x =
= = =
= 5, y =
= 15 i
3 9 12 3 + 9 + 12 24 3
3
3
12 5
z=
= 20 .
3
x y
z
Remarc. Se poate aplica i metoda: = =
= k , de unde x = 3k ,
3 9 12
5
y = 9k , z = 12k i nlocuind n x + y + z = 40 , obinem 24k = 40 , deci k = .
3
Rezult x = 5 , y = 15 , z = 20 .

Definiie. ntre dou mulimi finite de numere exist o proporionalitate


invers, dac se poate forma un ir de produse egale, diferite de zero, astfel nct
60

mulimea primilor factori ai produselor s fie una din mulimi, iar mulimea celorlali
factori ai produselor s fie cealalt mulime.
Exemplu. ntre mulimile {9,12,18} i {4,3,2} se stabilete o proporionalitate
invers, deoarece 94=123=182.
Observaie. Dac mprim un numr dat, diferit de zero, cu elementele unei
mulimi finite de numere nenule, obinem o alt mulime astfel nct ntre cele dou
mulimi s existe o proporionalitate invers.
ntr-adevr, numrul n mprit succesiv la elementele mulimii A={a,b,c}, se

n n n
a b c

obine mulimea B = , , . Cu elementele acestor dou mulimi putem forma un


ir de produse a cror valoare este n; deci a

n
n
n
= b = c (valoarea produselor este
a
b
c

n); deci ntre cele dou mulimi A i B s-a stabilit o proporionalitate invers.
Exemple. 1) ntre numrul robinetelor, cu acelai debit i timpul de umplere al
unui rezervor se stabilete o proporionalitate invers.
2) ntre numrul muncitorilor i timpul de realizare a unei anumite lucrri, se
stabilete o proporionalitate invers.
3) ntre viteza constant de parcurgere a unei distane i timpul de deplasare, se
stabilete o proporionalitate invers.
4) ntre numrul de bancnote i valoarea bancnotelor cu care se pltete o
anumit sum, se stabilete o proporionalitate invers.
Remarc. ntre elementele mulimilor A = {a, b, c} i B = {m, n, p} se
stabilete o proporionalitate invers, deci a m = b n = c p . Aceast relaie este
echivalent cu: a :

a b c
1
1
1
= b : = c : sau
= = , de unde rezult c ntre
1 1 1
m
n
p
m n p
1 1 1
, , s-a stabilit o proporionalitate direct.
m n p

elementele mulimilor {a,b,c} i

Model. Dou numere sunt invers proporionale cu numerele 0,2 i 0,5. Suma
dintre dublul primului numr i al doilea numr este 24. S se afle aceste numere.
Soluie. Notnd x primul numr i y al doilea numr, relaiile dintre acestea,

1
1
x y
= y i 2 x + y = 24 . Avem = = k , de unde
5
2
5 2
x = 5k , y = 2k i prin nlocuire n 2 x + y = 24 se obine 10k + 2k = 24 , deci
k = 2 . Numerele sunt x = 10 , y = 4 .

conform problemei sunt: x

61

Probleme rezolvate
R4.8.1. Un ir de 5 numere este format astfel nct primele 3 numere sunt
direct proporionale cu 4, 5, 6, iar ultimele 3 numere sunt invers proporionale cu 4, 5,
6.
a) S se afle cele mai mici 5 numere naturale care satisfac cerinele puse.
b) S se afle cele 5 numere care satisfac condiiile cerute, dac suma lor este
476.
Soluie. a) Fie a, b, c, d, e cele 5 numere. Conform enunului

a b c
= = i
4 5 6

c d e
= = . n ultimul ir de rapoarte egale nmulim toi numitorii cu 60 i obinem
1 1 1
4 5 6
c
d
e
= = . Pentru a obine un raport comun aflm c.m.m.m.c. al numerelor 6 i 15
15 12 10
a b c
(numitorii lui c), care este 30. n relaia = = , nmulim toi numitorii cu 30:6=5
4 5 6
c
d
e
i n relaia
= =
nmulim toi numitorii cu 30:15=2 i obinem:
15 12 10
a
b
c
c
d
e
a
b
c
d
e
=
=
=
=
=
=
=
=
i
, de unde rezult c:
.
20 25 30
30 24 20
20 25 30 24 20
Cele mai mici numere naturale care satisfac aceast condiie, vor fi cele pentru care
valoarea comun a rapoartelor este 1; deci numerele cutate sunt 20, 25, 30, 24, 20.
b) Din

a
b
c
d
e
a+b+c+d +e
476
=
=
=
=
=
=
= 4,
20 25 30 24 20 20 + 25 + 30 + 24 + 20 119

rezult a=80, b=100, c=120, d=96, e=80.


R4.8.2. S se determine numrul abc , tiind c numerele ab , bc , ca sunt
direct proporionale cu numerele 3, 2, 6, iar suma cifrelor numrului abc este
divizibil cu 7.
Soluie. Scriem c ab , bc , ca sunt direct proporionale cu 3, 2, 6 i apoi
aplicm proprietatea irului de rapoarte egale:

ab bc ca ab + bc + ca 10a + b + 10b + c + 10c + a


=
=
=
=
=
3
2
6
3+ 2+ 6
11
11(a + b + c)
=
= a + b + c . Dar, suma cifrelor este divizibil cu 7 i este un numr
11
mai mic dect 27 (suma maxim este suma cifrelor numrului 999). Aceast sum
poate fi 7, 14 sau 21.

62

a) Dac a + b + c = 7 , atunci ab = 21 , bc = 14 , ca = 42 , adic a=2, b=1,


c=4, deci abc = 214 .
b) Dac a + b + c = 14 , atunci ab = 42 , bc = 28 , ca = 84 , adic a=4, b=2,
c=8, deci abc = 428 .
c) Dac a + b + c = 21 , atunci ab = 63 , bc = 42 , ca = 126 , imposibil.
VI.R4.8.3. O sum de bani a fost distribuit la trei persoane direct proporional
cu numerele

1 1 1
, , . n acest mod o persoan constat c primete cu 46200 lei mai
6 5 3

mult dect dac aceeai sum s-ar fi distribuit invers proporional cu 12, 10, respectiv 15.
a) Care a fost suma de bani?
b) Ct a primit fiecare din cele trei persoane?
Soluie. Notm cu s suma total de bani, cu a, b, c sumele ce revin celor trei
persoane distribuite direct proporional cu

1 1 1
, , i cu x, y, z sumele ce revin celor
6 5 3

trei persoane dac ar fi distribuite invers proporional cu 12, 10, 15. Avem:

a b c a+b+c
s
=
= = =
(1) i
1 1 1 1 1 1 7
+ +
6 5 3 6 5 3 10
x
y
z
x+ y+z
s
=
=
=
=
(2).
1
1
1
1 1 1
1
+ +
12 10 15 12 10 15 4
5s
10s
2s
. b=
i c =
, iar din relaia (2)
Din relaia (1) rezult c a =
7
21
21
s
2s
4s
i z =
. Comparm sumele obinute de fiecare persoan
rezult c x = , y =
3
5
15
5s s 2 s 2 s
10s 4s
prin cele dou procedee de mprire:
< ,
<
> . Doar a treia
i
21 3 7
5
21 15
persoan primete mai mult prin mprirea sumei direct proporional cu numerele

1 1 1
, , . Deci, 46200lei reprezint diferena dintre cele dou sume, avem
6 5 3
10s 4s
150s
84 s

= 46200 . Efectund calculele obinem

= 46200 , de unde
21 15
2115 2115
46200 2115
66s
= 46200 ; rezult s =
, deci s=7002115, s=220500. Suma
66
2115
iniial a fost de 220500lei.

63

b) Pentru a calcula ce sum revine fiecrei persoane, avem

a=

5 220500
2 220500
10 220500
= 52500 ; b =
= 63000 i c =
= 105000 .
21
7
21

Cele trei persoane au primit 52500lei, 63000lei i 105000lei.


R4.8.4. Numerele x + y, y + z , z + x sunt direct proporionale cu numerele 4,
6, 8.

xy + xz + yz
.
x2 + y2 + z2
b) Dac a,b,c{1,2,...,9}, a b c a , s se determine valorile maxime i
axy + bxz + cyz
.
minime ale raportului
x2 + y2 + z 2
x+ y y+z z+x
=
=
= k , de unde x + y = 4k ,
Soluie. a) Avem
4
6
8
y + z = 6k i z + x = 8k , iar prin adunare membru cu membru a celor trei egaliti
obinem 2 x + 2 y + 2 z = 18k , deci x + y + z = 9k . Dac x + y + z = 9k i
x + y = 4k , rezult c z = 5k . Dac x + y + z = 9k i y + z = 6k , rezult c
x = 3k . Dac x + y + z = 9k i z + x = 8k , rezult c y = k . Se obine
a) Aflai valoarea raportului

xy + xz + yz 3k 2 + 15k 2 + 5k 2 23
=
.
=
x 2 + y 2 + z 2 9k 2 + k 2 + 25k 2 35
axy + bxz + cyz
se obine cnd b=9, c=8 i
b) Valoarea maxim a raportului
x2 + y2 + z 2
a=7 (deoarece xz > yz > xy ) i este

7 3k 2 + 9 15k 2 + 8 5k 2 196 28
=
= .
35k 2
35
5
axy + bxz + cyz
Valoarea minim a raportului
se obine cnd b=1, c=2 i a=3
a2 + b2 + c2
3 3k 2 + 115k 2 + 2 5k 2 34
=
.
(deoarece xz > yz > xy ) i este
35k 2
35
R4.8.5. Aflai numerele a, b, c naturale, tiind c numerele a 3 , b 2 , c sunt
1 1 24
.
direct proporionale cu 8, 4, 2 i + =
a b c
a3 b2 c
=
= = k 6 , de unde rezult c a 3 = 8k 6 , b 2 = 4k 6 i
Soluie. Avem
8
4 2
1 1 24
c = 2k 6 , deci a = 2k 2 , b = 2k 3 i c = 2k 6 . Prin nlocuire n relaia + =
, se
a b c
64

k + 1 24
1
1
24
, de unde
+ 3 = 6 . Dup efectuarea calculelor obinem
=
2
2k
2k
2k
2k 3 2k 6
rezult c k 3 (k + 1) = 24 , dar k fiind numr natural avem k 3 (k + 1) = 2 3 3 , deci k=2.
obine

Se obine a=8, b=16, c=128.


R4.8.6. Se dau numerele naturale a, b, c, d astfel nct 5 a = b 7 , c este 60%
din b, iar raportul dintre c i d este 1,5. S se arate c a, b, c, d sunt invers

1
; 0,5.
3
a b
Soluie. Dac 5 a = b 7 , atunci
= . Se tie c c este 60% din b, deci
7 5
c 3
3
b c
c d
c = b , de unde rezult c = . Avem = , de unde = . Se poate scrie
3 2
d 2
5
5 3
a b c d
urmtorul ir de rapoarte egale: = = = . Conform definiiei proporionalitii
7 5 3 2

proporionale cu numerele 0,(142857); 0,2;

directe rezult c numerele a, b, c, d sunt direct proporionale cu 7, 5, 3, 2, de unde


rezult c a, b, c, d sunt invers proporionale cu numerele

1 1 1 1
, , , . innd cont c
7 5 3 2

1
1
1
= 0, (142857) , = 0,2 i = 0,5 , avem a, b, c, d sunt invers proporionale cu
7
5
2
1
numerele 0,(142857); 0,2; ; 0,5.
3
4.9. Regula de trei simpl. Regula de trei compus
Regula de trei simpl
Vom considera probleme n care intervin dou mulimi de cte dou numere
ntre care exist o proporionalitate direct sau o proporionalitate invers, iar unul din
numerele unei mulimi este necunoscut.
Procedeul care se folosete pentru determinarea numrului necunoscut dintruna din dou mulimi, alctuite fiecare din cte dou elemente, ntre care exist o
proporionalitate direct sau invers, se numete regula de trei simpl.
Aplicarea acestui procedeu, numit regula de trei simpl, pornete de la
aezarea datelor problemei ntr-o schem, care conduce la aflarea unui termen
necunoscut dintr-o proporie (n cazul mrimilor direct proporionale) sau la aflarea
unui factor necunoscut al unui produs, cnd cunoatem produsul i cellalt factor (n
cazul mrimilor invers proporionale). Practic, schema conduce la rezolvarea unei
ecuaii cu o singur necunoscut.

65

Model 1. 18kg de mere cost 126000lei. Ct cost 13kg de mere de aceeai


calitate?
Soluie. Aceast problem poate fi rezolvat prin metoda reducerii la unitate:
1) Aflm preul unui kilogram de mere. 126000:18=7000lei.
2) Aflm preul a 13kg de mere. 700013=91000lei.
Prin regula de trei simpl, datele problemei se aeaz astfel:
18kg mere..............................126000lei
13kg mere..............................
x
Aceast schem se citete: "Dac 18kg de mere cost 126000lei, atunci 13kg
de mere vor costa x lei".
Stabilim ce fel de proporionalitate exist ntre cele dou mulimi: a cantitilor
i a costurilor. Pentru aceasta considerm mulimea cantitilor {18,13} i mulimea
costurilor {126000,x}. ntre aceste dou mulimi exist o proporionalitate direct,
deoarece putem forma un ir de rapoarte egale,

126000 x
= , valoarea lor comun
18
13

fiind tocmai preul unui kilogram de mere. Apoi aflm termenul necunoscut al
proporiei: x =

126000 13
, deci x=91000(lei).
18

Model 2. 15 muncitori pot termina o lucrare n 20 zile. n cte zile vor termina
lucrarea 25 de muncitori?
Soluie. Aceast problem poate fi rezolvat prin metoda reducerii la unitate:
1) Aflm n cte zile termin lucrarea un muncitor. 1520=300zile.
2) Aflm n cte zile termin lucrarea 25 muncitori. 300:25=12zile.
Prin regula de trei simpl datele problemei se aeaz astfel:
15 muncitori..............................20 zile
25 muncitori.............................. x
Aceast schem se citete astfel: "Dac 15 muncitori termin lucrarea n 20 de
zile, atunci 25 muncitori o vor termina n x zile".
Stabilim ce fel de proporionalitate exist ntre cele dou mulimi: a numrului
de muncitori i a numrului de zile n care ei pot termina lucrarea. Pentru aceasta
considerm mulimile {15,25} i {20,x}. ntre aceste dou mulimi exist o
proporionalitate invers, deoarece putem forma un ir de produse egale 1520=25x,
valoarea lor comun fiind tocmai numrul de zile n care un muncitor poate termina
lucrarea. Apoi, aflm factorul necunoscut: x =

15 20
, deci x=12(zile).
25

n practic, exist obiceiul ca, dup determinarea tipului de proporionalitate,


modul de aflare a necunoscutei s fie indicat, direct pe schem, printr-o sgeat care
indic nmulirea i scrierea literelor "d.p." (pentru proporionalitate direct), respectiv
"i.p." (pentru proporionalitate invers).
66

Model 1.
d.p.
18kg mere..............................126000lei
13kg mere..............................

x=

126000 13
= 91000lei
18

Model 2.
i.p.
15muncitori..............................20zile
25muncitori.............................. x

x=

20 15
= 12zile
25

Probleme rezolvate
R4.9.1. Un motociclist mergnd cu viteza de 60km/h strbate o distan n
48minute. Cu ce vitez trebuie s mearg pentru a parcurge aceeai distan n
45minute?
Soluie. Efectum transformrile:

48 min =

48
4
45
3
h = h, 45min=
h = h.
60
5
60
4

Prin regula de trei simpl, datele problemei se aeaz astfel:


i.p.

4
h..............................60km/h
5
3
h.............................. x
4

x=

4
5 = 60 4 4 = 64 (km/h)
3
5 3
4

60

67

R4.9.2. Un muncitor efectueaz 130 piese n 4 ore 20 min. Cte piese


realizeaz muncitorul n 8 ore?
Soluie. Efectum transformarea: 4 h 20 min= 4

20
1
h = 4 h.
60
3

Prin regula de trei simpl, datele problemei se aeaz astfel:


d.p.

1
4 h..............................130piese
3
8h

x=

.............................. x

130 8
3
= 130 8 = 240 (piese)
13
13
3

Probleme propuse
P4.9.1. Dac din 80kg fin se produc 180 pini, ce cantitate de fin este
necesar pentru obinerea a 72 de pini?
P4.9.2. Trei robinete, avnd acelai debit, umplu un rezervor n 6ore. n ct
timp vor umple rezervoarele dou robinete cu acelai debit?
P4.9.3. Un copil a economisit 15 bancnote de cte 10000lei. Cte bancnote de
50000lei primete n schimbul lor?
P4.9.4. Un muncitor face n 6 ore, 108 piese. Dac lucreaz n acelai ritm cte
piese, de acelai fel, face n 5 ore?
P4.9.5. La o ferm se planificase o cantitate de furaje pentru 40 vite pe timp de
60 zile. Pentru cte zile va ajunge aceeai cantitate de furaje, dac s-au mai cumprat 8
vite?
P4.9.6. Pentru transportul lemnelor de la munte la un depozit s-au comandat
40 de vagoane cu o capacitate de 15t fiecare. S-au folosit, ns, vagoane cu o capacitate
de 20t. Cte vagoane au fost necesare?
P4.9.7. O brigad de 24 de muncitori trebuia s sape 120m de an. 4
muncitori nu au lucrat. Ci metri de an au spat ceilali muncitori?
P4.9.8. La un magazin s-a adus 500 sticle de ulei pentru care trebuia s se
ncaseze 19.000.000lei. Dup ce s-au vndut 300 sticle, ce sum urmeaz s se
ncaseze?
P4.9.9. La acoperirea unei podele erau necesari 50m linoleum lat de 0,75m.
Ci metri linoleum sunt necesari pentru acoperirea aceleiai podele, dac se folosete
linoleum lat de 1,2m?
P4.9.10. Din 120kg ap de mare se obin 300g sare. Ce cantitate de ap de
mare este necesar pentru a obine 15kg sare?

68

Rspunsuri
R: P4.9.1. 32kg fin
R: P4.9.2. 9 ore
R: P4.9.3. 3 bancnote
R: P4.9.4. 90 piese
R: P4.9.5. 50 zile
R: P4.9.6. 30 vagoane
R: P4.9.7. 100m
R: P4.9.8. 7.600.000lei
R: P4.9.9. 31,25m
R: P4.9.10. 6000kg
Regula de trei compus
Vom considera acum probleme n care intervin mai multe mulimi de cte dou
numere, ntre unele din ele existnd o proporionalitate direct, iar ntre altele o
proporionalitate invers.
Regula de trei compus este un procedeu de aflare a unui numr necunoscut,
ntr-o problem n care intervin mai multe mrimi, cu cte dou valori, ntre unele
existnd o proporionalitate direct, iar ntre altele o proporionalitate invers.
Aplicarea acestui procedeu, numit regula de trei compus, pornete de la
aezarea datelor problemei ntr-o schem; apoi, se stabilesc tipurile de proporionalitate
ce exist ntre mrimea necunoscut i fiecare din celelalte mrimi, indicndu-se
nmulirea prin sgei, iar n final se efectueaz nmulirile i mpririle ce conduc la
aflarea numrului necunoscut. La acest procedeu s-a ajuns datorit modului de
rezolvare a acestui tip de probleme cu ajutorul aplicrii succesive a regulii de trei
simpl.
Model. Cinci muncitori pot termina o lucrare n 15zile, dac lucreaz cte 8ore
pe zi. n ct timp vor termina aceeai lucrare 10 muncitori, lucrnd cte 6ore pe zi?
Soluie. Datele problemei se aeaz dup urmtoarea schem:
5 muncitori..............................8h/zi..............................15zile
10muncitori..............................6h/zi.............................. x
Pentru a avea doar dou mrimi i a putea aplica, astfel, regula de trei simpl,
considerm constant numrul muncitorilor i problema se transform n:
i.p.
5 muncitori..............................8h/zi..............................15zile
5 muncitori..............................6h/zi.............................. y

y=

69

15 8
= 20 (zile)
6

Deci, 5 muncitori, lucrnd cte 6 ore pe zi termin lucrarea n 20 de zile. Acum


numrul de 6h/zi, fiind constant, trebuie s aflm n ct timp vor termina lucrarea cei
10 muncitori:
i.p.
5 muncitori..............................6h/zi..............................20zile
10muncitori..............................6h/zi.............................. x

x=

20 5
= 10 (zile)
10

Practic, regula de trei compus cuprinde urmtoarele etape:


1) Aezarea datelor problemei n schem:
5 muncitori..............................8h/zi..............................15zile
10muncitori..............................6h/zi.............................. x
2) Stabilirea tipului de proporionalitate ce exist ntre mulimea ce conine
necunoscuta i, succesiv, celelalte mulimi:
ntre mulimea zilelor i cea a muncitorilor exist o proporionalitate invers
ntre mulimea zilelor i cea a orelor de lucru zilnic exist o proporionalitate
invers.
Precizm aceste proporionaliti prin sgei, pe schem:
i.p.
i.p.
5 muncitori..............................8h/zi..............................15zile
10muncitori..............................6h/zi.............................. x

x=

15 5 8
= 10 (zile)
10 6

Remarc. Aceeai problem se poate rezolva prin metoda reducerii la unitate


fcnd urmtorul raionament: dac 5 muncitori, lucrnd cte 8h/zi, termin lucrarea n
15zile, atunci 1muncitor, lucrnd cte 8h/zi, termin lucrarea n 515zile. Rezult c
1muncitor, lucrnd cte o or pe zi, termin lucrarea n 5158zile. Rezult, n
continuare, c un muncitor, lucrnd cte 6h/zi, va termina lucrarea n

70

5 15 8
zile.
6

Deci, 10muncitori, lucrnd cte 6h/zi, va termina lucrarea n

5 15 8
zile, adic n
6 10

10zile. Acest raionament se aeaz sub forma urmtoarei scheme:


5muncitori..........................8h/zi..........................15zile
10muncitori........................6h/zi.......................... x
1muncitor...........................8h/zi..........................155
1muncitor...........................1h/zi..........................1558

15 5 8
6
15 5 8
10muncitori.........................6h/zi.........................
= 10 zile
6 10
1muncitor...........................6h/zi..........................

Probleme rezolvate
R4.9.3. ntr-o tabr, n 12zile, 150 de elevi consum 900kg pine. Ce
cantitate de pine este necesar pentru 70 de elevi, pentru 18 zile?
Soluie. 1) Aezarea datelor problemei n schem
150elevi..............................12zile..............................900kg
70elevi................................18zile.............................. x
2) Stabilirea tipului de proporionalitate ce exist ntre mulimea ce conine
necunoscuta i, succesiv, celelalte mulimi, preciznd aceste proporionaliti prin
sgei, pe schem:
150elevi..............................12zile..............................900kg
d.p.
d.p.
70elevi................................18zile.............................. x

x=

900 70 18
= 630 kg
150 12

R4.9.4. 6 muncitori pot termina o lucrare n 12zile. Dup 4zile de lucru


echipei de muncitori i se altur nc 2 muncitori. n ct timp se va executa toat
lucrarea?
Soluie. Dac echipa poate termina lucrarea n 12zile, atunci dup 4zile de
lucru echipa a efectuat
muncitori fac

4
1
, adic
din lucrare. Deci, trebuie s aflm n cte zile 8
12
3

2
din lucrare.
3
71

Aezm datele problemei i stabilim tipul de proporionalitate ce exist ntre


mulimea ce conine necunoscuta i, succesiv, celelalte mulimi, preciznd aceste
proporionaliti prin sgei, pe schem:
i.p.
6muncitori..............................1lucrare..............................12zile
d.p.
8muncitori..............................

x=

12 6
8 1

2
lucrare............................. x
3

2
3 = 6 zile

Lucrarea s-a efectuat n 4zile+6zile, deci n 10zile.


R4.9.5. O echip de 20 muncitori, lucrnd cte 6ore pe zi, pot face 24piese n
10zile. Cte zile sunt necesare pentru ca o alt echip de 15 muncitori s fac 360piese,
lucrnd cte 8ore pe zi?
Soluie. Aezm datele problemei n schem i stabilim tipul de proporionalitate ce exist ntre mulimea ce conine necunoscuta i, succesiv, celelalte
mulimi, preciznd aceste proporionaliti prin sgei, pe schem:
i.p.
i.p.
20muncitori....................6h/zi....................240piese....................10zile
d.p.
15muncitori....................8h/zi....................360piese.................... x

x=

10 20 6 360
= 15 zile
15 8 240

72

5. Numere ntregi
n cadrul temei se vor studia i aplica noiunile de numr ntreg, opusul unui
numr, modulul unui numr ntreg, divizibilitatea n mulimea numerelor ntregi,
determinarea valorii unei expresii ce depind de un exponent natural, rezolvarea de
ecuaii i inecuaii.
5.1. Divizibilitatea n
Reamintim noiunile necesare abordrii temei.
Mulimea numerelor ntregi se noteaz cu .
= {,., 3; 2; 1; 0; +1; +2; +3; .}
* = {0}
Definiia 5.1.1. Opusul unui numr ntreg a este numrul ntreg a cu
proprietatea c punctele de pe ax corespunztoare numerelor a respectiv a se afl la
egal distan fa de origine.
Exemplu: opusul numrului 7 este numrul 7
opusul numrului 6 este 6.
Scriem: (2) = 2.
Definiia 5.1.2. Distana pe ax ntre punctul corespunztor numrului ntreg a
i origine se numete modulul numrului a sau valoarea absolut i se noteaz cu a.
Exemple:5 = 5; 1 = 1; 0 = 0; 3 = 3; 1 = 1; 7 = 7.
Definiia 5.1.3.
a dac a > 0
() a ; a =

0 dac a = 0
a dac a < 0

Proprieti:
1. Modulul oricrui numr ntreg pozitiv este mai mare dect zero.
() a *; a 0
2. Modulul unui numr ntreg este egal cu zero dac i numai dac numrul este
egal cu zero.
a = 0 a = 0
3. Dou numere opuse au modulele egale.
() a ; a = a
4. Dac o sum de module este egal cu 0 atunci fiecare modul este egal cu zero.
5. Dac a1+ a2 + .+ an= 0 atunci a1= 0 i a2 = 0 i
an= 0
Definiia 5.1.4. Numrul ntreg a divide numrul ntreg b dac exist numrul
ntreg c astfel nct b = ac.
Notm: ab (numrul a divide numrul b).
ba ( numrul b se divide cu numrul a).
Numrul a se numete divizorul lui b, iar b este multiplul lui a dar i a lui c.
Exemplu: 728 pentru c exist numrul 4 astfel nct 28 = (7)4
73

1.
2.
3.
4.
5.
6.
7.
8.
9.
10.

Proprieti ale relaiei de divizibilitate:


1a, oricare ar fi a ;
aa, oricare ar fi a ;
a0, oricare ar fi a *;
Dac ab i bc atunci ac, () a, b * i c ;
Dac ab i ba atunci a = b, () a, b *;
Dac ab sau ac atunci abc,() a *i b, c ;
Dac ab i ac atunci a(bc),() a *i b, c ;
Dac ac i bc i (a; b) = 1 atunci (ab)c,() a, b *i b, c ;
Dac a1 sau a1 atunci a = 1.
ab(a)ba(b)(a)(b).

Definiia 5.1.5. Un numr ntreg d este divizor comun al numerelor a i b dac


da i db.
Definiia 5.1.6. Numrul ntreg d este cel mai mare divizor comun al
numerelor a i b dac:
1) da i db;
2) orice divizor d a lui a i b, comun, divide pe d.
Definiia 5.1.7. Dou numere ntregi nenule a i b se numesc prime ntre ele
dac (a; b) = 1.
Definiia 5.1.8. Numrul ntreg m este cel mai mic multiplu comun al
numrului a i b dac:
1) a i b au multiplu comun pe m;
2) orice alt multiplu comun a lui a i b este i multiplul lui m.
Probleme rezolvate
R5.1.1. Determinai mulimea divizorilor numrului:
a) 12;
b) 20;
c) 7
Soluie:
D12 = {1; 2; 3; 4; 6;12}.
D20 = {1; 2; 4; 5; 10;20}.
D7 = {1; 7}. Numrul 7 se numete prim pentru c are ca divizor numai pe 1 i 7.
R5.1.2. Determinai n astfel nct (2n + 1)(3n + 4).
Soluie:
2n + 13n +4 i 2n + 12n + 1 atunci
2n + 12(3n + 4)
i
2n + 16n + 8 (6n + 3)
2n + 13(2n + 1)
2n + 16n + 8 6n 3
2n + 152n + 1 D5 = { 1; 5}

74

2n + 1= 11

2n + 1= 11

2n = 0: 2
n=0

2n = 2: 2
n = 1

2n + 1= 5
2n = 5 1
n = 4:2
n=2

2n + 1= 5
2n = 6: 2
n = 3

n {0, 1, 2, 3}
Soluia a II-a:
Determinai numerele ntregi n pentru care:

3n + 4

2n + 1
2(3n + 4)
3n + 4
dac

2n + 1
2n + 1
6n + 8 6n + 3 + 5 6n + 3
5
=
=
+
=
2n + 1
2n + 1
2n + 1 2n + 1
5
5
dac

=3+
2n + 1
2n + 1
2n + 15 2n+ 1 D5 = {1; 5}

n {0; 1; 2; 3}.
R5.1.3. Determinai numerele ntregi x i y astfel nct: (x + y)(2y 1) = 43
Soluie:
Din 43 = (x + 1)(2y 1) rezult c
x + 143 i 2y 143 i x + 1 i 2y 1 au acelai semn, atunci avem:
1) x + 1 = 1
2) x + 1 = 43
3) x + 1 = 1
2y 1 = 43
2y 1 = 1
2y 1 = 43
Soluiile sunt:
1) x = 0
y = 22
2) x = 42
y=1
3) x = 2
y = 21
4) x = 44
y=0

75

4) x + 1 = 43
2y 1 = 1

R5.1.4. Determinai n pentru care fracia

2n + 5
este reductibil.
n +1

Soluie:
Fie d cel mai mare divizor comun al numerelor 2n + 5 i n + 1 atunci avem:
d2n + 5 i dn + 1
d2n + 5 i d2(n + 1)
d2n + 5 2(n + 1)
d2n + 5 2n 2
d3, 3 este numr prim, rezult c d = 3.
Atunci 32n + 5 i 3n + 1
32n + 5 (n + 1)
32n + 5 n 1
3n + 4
n + 4 = 3k, k
n = 3k 4
5.2. Determinarea valorii unei expresii ce depinde de un exponent natural
Definiia 5.2.1. Dac a este numr ntreg i n numr natural, n 2 atunci
puterea n a lui a este: an = aaaa
n factori
a se numete baza puterii;
n exponentul puterii.
Dac a > 0 atunci an > 0
Dac a < 0 atunci

an > 0 dac n = 2k, k


an < 0 dac n = 2k + 1, k

Oricare ar fi a i n = 1 avem a1 = a.
Oricare ar fi a * i n = 0 avem a0 = 1.
00 nu are sens.
Reguli de calcul cu puteri
1. aman = a m+n, a , m, n
2. am:an = a mn, a , m, n , m n
3. (am)n = amn, a , m, n ;
4. (ab)n = anbn, a , b i n ;
5. (a:b)n = an:bn, a, b , b 0, n ;

76

Probleme rezolvate
R5.2.1. Fie E(n) = (1)nn, n *. S se calculeze:
E(1) + E(2) + E(3) + + E(2003)
Soluie:
E(1) = (1)11 = 1
E(2) = (1)22 = 12 =2
E(3) = (1)33 = 13 = 3
E(4) = (1)44 = 14 = 4

E(2003) = (1)20032003 = 12003 = 2003.

E(1) + E(2) + E(3) + E(4) ++E(2001) + E(2002) + E(2003) =


= (1 + 2) + (3 + 4) + + (2001 + 2002) 2003 =
= 1 + 1 + 1+ + 1 2003 =
1001 termeni
= 1001 2003 = 1002
R5.2.2. Calculai:
[2333 3222+2775 8115+72917 51237 3225 + 930]: (23222 4167)
Soluie:
2333 = (23)111 = 8111
8111 < 9111
3222 = (32)111 = 9111
2333 < 3222 2333 3222 < 0
2333 3222 = 2333 + 3222
2775 = (33)75 = 3225
3225 > 360 2775 > 8115
8115 = (34)15 = 360
2775 8115 > 0 2775 8115=2775 8115
72937 = 3637 = 3222 > 2333
51237 = 2937 = 2333
3222 2333 > 0 3222 2333= 3222 2333
Avem:
(3222 2333 + 3225 360 + 3222 2333 3225 + 360 ): (23222 4167) =
= (23222 22333):(23222 2334) = (23222 2334):(23222 2334) = 1
R5.2.3. Fie numrul:

A = 243(1)n 342(1)n + 1 + 456 ( 1)


654 ( 1)
unde k, p, n . Artai c A 5.
Soluie:
k2 k + 1990 = k(k 1) + 1990
k(k 1)2 ( produs de nr.nat.consec.)
19902

k 2 k +1990
(k2 k + 1990) 2 ( 1)
=1
p2 + p + 1 = p(p + 1) + 1
p 2 + p +1
p(p + 1)2 (produs de nr.nat.consec.) p (p + 1) + 1 impar ( 1)
= 1
Atunci: A = (1)n243 342(1)n+1 + 456 + 654 = 243(1)n 342(1)n+1 + 1110 =
= (1)n[243 342(1)] + 1110 = (1)n585 + 1110
1) n nr. pare A = 585 + 1110 = 1695
2) n nr. impare A = 585 + 1110 = 525
k 2 k +1990

77

p 2 + p +1

5.3. Ecuaii n mulimea numerelor ntregi. Inecuaii n mulimea numerelor ntregi


n cadrul temei se vor studia ecuaii de forma ax + b = 0, x (a *; b
) ax + b = c, x ( a *; b, c ), i ecuaii reductibile la acestea, ecuaii de
forma ax2+b = 0, x (a *; b ), ecuaii cu modul, (ax + b)(cx + d) = m, x
(a, c *; b,d , m ).
Definiia 5.3.1. O ecuaie este o propoziie cu o variabil scris sub forma
unei egaliti, semnul egal apare o singur dat n scrierea ei.
Exemplu:
1) 3x 6 = 0
x ;
2) 2x 7 = 4
x
3) 2(x + 1) = 3x x {1, 2, 3}
x {1; 2; 2; 5}
4) x2 4 = 0
Variabila ntr-o ecuaie se numete necunoscut.
Exist ecuaii cu o necunoscut, cu dou necunoscute i aa mai departe.
Necunoscutele se noteaz cu: x, y, z
Mulimea valorilor necunoscutelor pentru care propoziia este adevrat este
mulimea soluiilor ecuaiei.
Exemplu:
2(x + 1) = 3x
pt. x = 1
2(1 + 1) = 31, pt x = 2 2(2 + 1) = 32
22 = 31 (F)
23 = 32 (A)
pt. x = 3
2(3 + 1) = 33;
24 = 33 (F)
Mulimea soluiilor este {2}. i se noteaz S = {2}.
A rezolva o ecuaie nseamn a determina mulimea soluiilor.
Definiia 5.3.2. Dou ecuaii cu o necunoscut sunt echivalente dac au
aceeai mulime de soluii.
O ecuaie de forma ax + b = 0, x , unde a i b sunt numere ntregi, iar a 0, va fi
numit ecuaie liniar cu o necunoscut (sau ecuaie de gradul I cu o necunoscut).
Proprieti de echivalen:
1. Dac adunm sau scdem acelai numr ntreg n fiecare membru al unei
ecuaii, obinem o ecuaie, echivalent cu prima.
2. Dac nmulim sau mprim ambii membri ai unei ecuaii cu acelai
numr ntreg, diferit de zero, obinem o ecuaie echivalent cu prima.
Probleme rezolvate
R5.3.1. Rezolvarea ecuaiei ax + b = 0, x , (a *, b )
ax + b = 0b
ax + b b = b
ax = b:a
x = b : a
Dac ab atunci x = b : a este soluia.
Dac ab atunci ecuaia nu are soluii n .
R5.3.2. Rezolvarea ecuaiei:
ax + b = c, x (a *, b,c ).
ax + b = cb
ax + b b = c b
ax = c b:a
Dac a(c b) atunci ecuaia are soluia: x = (c b) : a
Dac a (c b) atunci ecuaia nu are soluie n .
Exemple:
78

2) 3x + 7 = 07
3x + 7 7 = 0 7

1) 2x 4 = 0+4
2x 4 + 4 = 0 + 4
2x = 4:2
x=4:2

3x = - 7:3

x=2

S=

x= 7 : 3

S = {2}

4) 7x + 11 = 411
7x + 11 11 = 4 11

3) 5x + 21 = 621
5x + 21 21 = 6 21
5x = 15:5

7x = 15:7

x = 3

x = 13 : 7
S=

S = {3}
R5.3.3. Ecuaii care se rezolv prin metode speciale:

1. Dac x, y , rezolvai ecuaiile:


a) xy = 5
Trebuie s determinm dou numere ntregi a cror produs este egal cu 5
Soluiile sunt:
1) x = 1
2) x = 5
3) x = 1
4) x = 5
y=5
y=1
y = 5
y =1
S = {(1; 5); (5; 1); (1; 5); (5; 1)}

1)

b) 2xy 3x = 7
Scoatem factor comun i obinem x(2y 3) = 7, avem urmtoarele situaii:
x=1
x=1
x=1
x =1

y = 5
2y 3 = 7
2y = 7 + 3
2y = 10

2)

x=7
2y 3 = 1

x=7
2y = 1 + 3

x=7
2y = 4

x =7
y=2

3)

x = 1
2y 3 = 7

x = 1
2y = 7 + 3

x = 1
2y = 4

y = 2

x =1

4)

x = 7
2y 3 = 1

x = 7
2y = 1 + 3

x = 7
2y = 2

x =7
y=1

2y = 2
x=5

y =1
x=5

S = {(1; 5); (7; 2); (1; 2); (7; 1)}


c) Rezolvai n x ecuaiile
2xy 3x 4y + 9 = 0
2xy 3x 4y + 6 + 3 = 0
x(2y 3) 2(2y 3) = 3
(2y 3)(x 2) = 3
Avem urmtoarele situaii:
1)
2y 3 = 1 =
2y= 1 + 3

x2=3
x=3+2

79

2)

2y 3 = 3
x 2 = 1

2y= 3 + 3
x 2 = 1

2y = 6
x = 1 + 2

y =3
x=1

3)

2y 3 = 1
x 2 = 3

2y= 1 + 3
x = 3 + 2

2y = 4
x = 1

y =2
x = 1

4)

2y 3 = 3 =

x2=1

2y= 3 +3
x=1+2

2y = 0
x=3

y =0
x=3

S = {(5; 1); (1; 3); (1; 2); (3; 0)}


2. Ecuaii cu modul:
Rezolvai n ecuaiile:
a) x= 5 x = 5
Aplicm proprietatea modulului:
() x , avem x= x
b) x 3= 3 (x 3) = 3
1) x 3 = 3 x = 3 + 3 x = 6
2) x 3 = 3 x = 3 3 x = 0
S = {0, 6}
c) () x , avem x 0
Dac x + 5= 2 S = pentru c x + 5 0 () x
d) x 2 + y + 1= 0
() a1, a2,, an, dac a1+a2++ an= 0 a1 = a2 = = an = 0
Din x 2+y + 1= 0
x 2= 0
i
y + 1= 0
x2=0
i
y+1=0
x=2
i
y=1
S = {(2; 1)}

80

e) x + 1+y 2= 2
Dac x, y i x + 1+y 2= 2, atunci avem:
1)

x + 1= 0

x+1=0
]

a)

y 2= 2
x + 1= 0

b)

y2=2
x + 1= 0

y 2 = 2
x = 1
]
y=4
x = 1
]

y 2 = 2
2)

y=0

x + 1= 2
]
a)

y 2= 0
x + 1= 2

b)

y2=0
x + 1= 2

x + 1 = 2
y2=0
x=1

]
y=2
x = 3
]

3)

y2=0

y=2

x + 1= 1

x + 1 = 1

]
a)

y 2 = 1
x=0

y 2= 1
x + 1= 1
]
y2=1

b)

y=3

x + 1= 1

x=0
]

y 2 = 1
c)

y=1

x + 1= 1

x = 2
]

d)

y2=1
x + 1= 1

y=3
x = 2
]

y 2 = 1

y=1

81

Probleme propuse
Rezolvai cu ecuaiile:
P5.3.1. 5(x 1) = 2x + 7
P5.3.2. 2(4x + 1) = 2(5x 7)
P5.3.3. 2(x + 5) = 8(3x + 2) + 56
P5.3.4. 5(x 2) 7(x + 3) = 35
P5.3.5. 3(x + 1) 1 = x + 6
P5.3.6. (x 1)(x + 2) = 0
P5.3.7. (2x + 8)(3x + 9) = 0
P5.3.8. xy 2x = 7
P5.3.9. xy + x 2y 4 = 0
P5.3.10. xy x + 3y 3 = 0
P5.3.11.x 2= 3
P5.3.12. 3x 1= 14
P5.3.13.2x 6 = 8
P5.3.14. 2 10 + 3x= 5
P5.3.15. x= 3
P5.3.16. 10 x + 2= 8
P5.3.17. 2x 1+ 31 x= 10
P5.3.18. 3x 222 x= 11
P5.3.19. x 3+x + 5= 0
P.5.3.20.x+y 2= 3
P5.3.21. x 1+y + 3= 1
P5.3.22. x + 21= 3
P5.3.23.x + 53y 12= 0
P5.3.24. Se dau mulimile:
A= {x / 2x 1(8)}, B = {y /12(3y 1)}
a) Determinai elementele celor dou mulimi;
b) Aflai AB i AB.
P5.3.25.
a) x y+ (2x 6)2 = 0
b) x 3+ (2x y)2 = 0
P5.3.26. 3x 2y = 15 i x= 5
Inecuaii n mulimea numerelor ntregi
Definiia 5.3.3. O propoziie cu o variabil scris sub forma unei inegaliti se
numete inecuaie.
Exemple: ax + b > 0, ax + b < 0; ax + b 0; ax + b 0 n care a, b , sunt
fixate, a 0, iar x este variabila ntreag, x se numete necunoscuta inecuaiei.
Pentru a rezolva o inecuaie folosim proprietile inegalitii numerelor ntregi:
82

oricare ar fi a, b, c i a > b atunci a + c > b + c


oricare ar fi a, b , c +*, i a > b, atunci a:c > b:c, dac c divide pe a i c
divide pe b.
oricare ar fi a, b i c -* i a > b, atunci a:c < b.c, dac c divide pe a i c
divide pe b.
Probleme rezolvate
R5.3.4. Rezolvai inecuaia: 5x + 3 3x 5, x .
Soluie:
5x + 3 3x 53x
5x + 3 3x 3x 5 3x
5x 3x + 3 53
2x + 3 3 5 3
2x 8:2
x 4
x {, 5, 4} S = {, 5, 4}
R5.3.5. Rezolvai inecuaia:
x 3 3, x
1) x 3= 3x 3 = 3 a) x = 6
b) x = 0
2) x 3= 2x 3 = 2 a) x = 5
b) x = 1
3) x 3= 1x 3 = 1 a) x = 4
b) x = 2
4) x 3= 0 x = 3
S = {0, 1, 2, 3, 4, 5, 6}
R5.3.6. Determinai numerele ntregi care verific inegalitatea:x(y 2) > 0
Soluie:
x(y 2) > 0
x > 0, dac x 0

y2>0
y>2

1) Dac x * atunci y {3, 4, 5,}


2) Dac x = 0 inecuaia nu are soluie.

83

Bibligrafie
C. Hrbor, D.Svulescu, I. Chec, A.ifrea: Matematic pentru clasele V-VIIIOlimpiadele judeene,interjudeene,naionale, Ed. Teora 1996, pag 72-76
D. Andrica, E. Jecan, D. Vlcan, I. Bogdan, Probleme calitative n matematica de
gimnaziu, Ed. Gil Zalu 1998, pag 21-43
Gheorghe i Alina Drugan; Ion i Mihaela Ghica, Matematica n concursurile colare,
Ed. Teora 1998, pag 5-23
D. Brnzei, D. Zaharia, M. Zaharia : Aritmetic-Algebr-Geometrie, Ed.Paralela 45
2002, pag 5-43
D. Andrica, V. Berinde, Al. Blaga, G.Both, O. Pop, Concursul Grigore Moisil Ed. IXV, Ed. Hub Press 22 Baia-Mare 2001, pag 9,12,40
C. Moroti, M. Giurgiu, D. Radu, R. tefan, A. Ciupitu, G. Drugan, I. Ghica,
Matematic-exerciii i probleme pentru clasa a VI-a, Ed. Meteor Press 2002, pag 44-49, semII
D. Mihe, D. Angelescu, I. Chera, C. Popescu i colectivul, Olimpiadele de matematic
1990-1998, clasa a VI-a, Ed. Gil Zalu 1999, pag 37-43
C. Hrbor, D.Svulescu, I. Chec, A.ifrea: Matematic pentru clasele V-VIII-Olimpiadele
judeene,interjudeene,naionale, Ed. Teora 1996, pag 72-76
D. Brnzei

i colectivul: Matematica n concursurile colare, Ed. Paralela 45,

2000,2001,2002 pag 27-54,119-135(2000);pag 27-54,117-130(2001);pag 18-34,82-92 (2002)


D. Brnzei, D. Zaharia, M. Zaharia : Aritmetic-Algebr-Geometrie, Ed.Paralela 45
2002, pag 5-43
C. Popovici, I. Ligor, V. Alexianu, Matematic-Aritmetic-Algebr, EDP Bucureti
1996
G. Turcitu, I. Rizea, C. Basarab, M. Duncea, Manual clasa a VI-a, Ed. Radical 1998
T. Udrea, D. Nuescu, Manual clasa a VI-a, EDP 1998
D. Brnzei, D. Zaharia, M. Zaharia : Aritmetic-Algebr-Geometrie PII, Ed. Paralela
45 2002, pag 39-43

84

GEOMETRIE
1. Segmente
n cadrul temei se vor studia noiunile de punct, dreapt, plan, semidreapt,
segment de dreapt, dar i aplicaii n care se vor determina poziiile unor puncte pe o
dreapt, lungimea unui segment, distana dintre mijloacele a dou segmente i
congruena a dou segmente.
1.1 Puncte, drepte, semidrepte, segmente de dreapt
n vederea abordrii temei reamintim noiunile absolut necesare.
Considerm figura:
O
B
A
X
Vom spune c punctul B este ntre O i A.
Definiia 1.1.1 Fiind date punctele O i A pe o dreapt, mulimea format din
punctele dreptei OA situate ntre O i A mpreun cu punctele X de pe dreapt pentru
care A este ntre O i X se numete semidreapt. Punctul O este originea semidreptei
Semidreapt deschis
A
(
Notaie: (OA- semidreapt deschis
Semidreapta nchis:
A
Notaie: [OA- semidreapt nchis ; [OA = (OA {A}
O definiie mai puin riguroas dar uor de reinut de ctre elevi este:
semidreapta este o parte dintr-o dreapt, limitat la unul dintre capete, numit originea
semidreptei.
Definiia 1.1.2 Fiind date dou puncte A i B, mulimea punctelor ce aparin
dreptei AB situate ntre A i B se numete segment deschis i se noteaz (AB)
(
)
A
B
Segmentul nchis [AB] = (AB) {A, B}
[
]
A
B
Punctele A i B se numesc extremitile sau capetele segmentului.
Definiia 1.1.3 Distana dintre punctele a i B exprimat ntr-o unitate de
msur se numete lungimea segmentului [AB].
Notaie: AB = 5 cm.

85

Definiia 1.4. Dou segmente se numesc congruente dac au msuri egale.


Notm [AB] [CD].
A
3,5 cm
B
C
3,5 cm
D
Definiia 1.1.5 Mijlocul unui segment este un punct care mparte segmentul n
dou segmente congruente.
M
A
B
Notm M este mijlocul segmentului [AB] sau M (AB) i [AM] [MB]
Definiia 1.1.6 Fiind date dou segmente, vom num segmentul sum al lor un
segment care are msura egal cu suma msurilor celor dou segmente, iar segmentul
diferen segmentul care are msura egal cu diferena msurilor segmentelor date.
Dac AB = 15 cm, CD =7 cm i MN= 22 cm, atunci
MN=AB+CD
(22=15+7), [MN] este segmentul sum.
Dac AB = 15 cm, CD =7 cm i PQ=8 cm, atunci PQ=AB CD (8=15 7),
[PQ] este segmentul diferen
Probleme rezolvate
R1.1.1
Fiind date 10 puncte distincte dou cte dou i necoliniare trei cte trei,
aflai numrul de drepte determinate de cte dou dintre ele.
Soluie
Dou puncte distincte determin o dreapt i numai una. Fie A1,A2,A3.......,A10
cele 10 puncte punctul A1 determin cu punctele A2,A3.......,A10, 9 drepte.
Mergnd cu raionamentul din fiecare punct putem duce (10-1) drepte, ns n
acest caz fiecare dreapt o considerm de dou ori, deci numrul dreptelor este dat de

10(10 1) 10 9
=
= 45
2
2

Dac numrul punctelor este n atunci numrul dreptelor este

n (n-1)
.
2

R1.1.2
Considerm punctele A,B,C,D pe dreapta d, astfel nct AB=a cm, AC=b
cm, BD=c cm, BC=(a+b)cm, CD=(a+b-c)cm i AD=(c-a)cm. Determinai ordinea
punctelor pe dreapt.
Soluie
Din AB=a, AC=b i BC=a+b rezult c A este ntre B i C.
Dac B aparine lui (AD) atunci CD=a+b+c ceea ce este fals.
C

B
A
D
C
Dac C este ntre B i D atunci CD=c-a-b ceea ce este fals, deci D este ntre B
i C i cum AD=c-a rezult c D este ntre Ai C deci ordinea punctelor este:
86

1) C,D,A,B sau 2) B,A,D,C


R1.1.3 Punctul M1 este mijlocul segmentului [AB], punctul M2 este mijlocul
segmentului [AM1].
Repetnd procedeul punctul M10 este, mijlocul segmentului AM9. Dac
AB=2113cm, calculai msura segmentului AM10.
Soluie

AB
211 3
; AM1 =
;
2
2
AM1
211 3
AM1 =
; AM1 =
;
2
2112
AM 2
2 3
AM3 =
; AM3 =
;
2
23

AM1 =

AM10 =

AB
211 3
; AM10 = 10 = 2 3 = 6;
2
2

Bibliografie
I. Petric i colectivul, Manual pentru clasa a VI-a, Ed. Petrion 1998
G. Turcitu, I. Rizea, C. Basarab, M. Duncea, Manual clasa a VI-a, Ed. Radical 1998
T. Udrea, D. Nuescu, Manual clasa a VI-a, EDP 1998
D. Brnzei, D. Zaharia, M. Zaharia : Aritmetic-Algebr-Geometrie, Ed.Paralela 45
2002, pag 97-109
C. Hrbor, D.Svulescu, I. Chec, A.ifrea: Matematic pentru clasele V-VIIIOlimpiadele judeene,interjudeene,naionale, Ed. Teora 1996, pag 107
D. Brnzei, D. i M. Goleteanu, S. Ulmeanu, V. Gorgot, I. erdean: Matematica n
concursurile colare, Ed. Paralela 45, 2000,2001,2002
D. Constantinescu: Olimpiadele colare, Ed. Teora 1997-2002
D. Mihe, N. Angelescu, I. Chera, C. Popescu i colectivul:Olimpiadele de matematic
1990-1998, Ed. Gil Zalu ,clasa a VI-a, pag 56-61
Edwin E. Moise, Floyd I. Downs jr. : Geometria , EDP 1983, pag 38-59

87

2. Unghiul
n cadrul temei se vor utiliza noiunile de unghi, unghiuri adiacente, bisectoarea
unui unghi, msura unui unghi, unghiuri congruente, unghi ascuit, drept, obtuz,
unghiuri opuse la vrf, unghiuri n jurul unui punct, drepte perpendiculare, mediatoarea
unui segment i aplicaii n care se vor determina msuri de unghiuri, congruena unor
unghiuri, perpendicularitatea unor drepte i coliniaritatea unor puncte.
2.1 Noiuni teoretice necesare abordrii temei
Definiia 2.1.1
Se numete unghi figura geometric format din dou semidrepte nchise care au
aceeai origine.

O
A
[OA i [OB laturile unghiului
O vrful unghiului
Notaie: AOB
Unghi alungit

AOB este alungit dac laturile lui sunt semidrepte opuse


Unghi nul

AOB este unghi nul dac [OA=[OB (semidreptele coincid)


Unitatea de msur pentru unghiuri este gradul. Unghiul de un grad reprezint a
180-a parte dintr-un unghi alungit.

88

Definiia 2.1.2
Msura unui unghi este un numr care ne arat de cte ori se cuprinde unitatea
de msur n unghiul pe care l msurm.
Notaie: Dac unghiul AOB are msura de 30 de grade notm m(AOB)=300
Observaie:
10 Msura unui unghi alungit este 1800
20 Msura unui unghi nul este 00
Definiia 2.1.3
Dou unghiuri se numesc congruente dac au msurile egale
Notaie: AOB AOB (unghiul AOB este congruent cu unghiul AOB)
Definiia 2.1.4.
Se numete bisectoare interioar a unui unghi nenul o semidreapt interioar
unghiului, cu originea n vrful unghiului, care formeaz cu laturile acestuia dou
unghiuri congruente.
Definiia 2.1.5
Unghiul care are msura de 900 se numete unghi drept.
Definiia 2.1.6
Unghiul cu msura cuprins ntre 00 i 900 se numete unghi ascuit.
Definiia 2.1.7
Unghiul cu msura cuprins ntre 900 i 1800 se numete unghi obtuz.
Definiia 2.1.8
Dou unghiuri se numesc adiacente dac au vrf comun, o latur comun i
interioarele disjuncte.
Definiia 2.1.9
Dou unghiuri se numesc complementare dac suma msurilor lor este 900.
Definiia 2.1.10
Dou unghiuri se numesc suplementare dac suma msurilor lor este 1800.
Definiia 2.1.11
Dou unghiuri se numesc opuse la vrf dac au acelai vrf i laturile unuia se
gsesc n prelungirea laturilor celuilalt.
Propoziia 2.1.1
Dou unghiuri opuse la vrf sunt congruente.
Propoziia 2.1.2
Suma msurilor unghiurilor formate n jurul unui punct este 3600.
Definiia 2.1.12
Dou drepte se numesc perpendiculare dac formeaz un unghi drept.
Notaie: ab (dreptele a i b sunt perpendiculare)
Definiia 2.1.13
Prin distana de la un punct la o dreapt nelegem distana de la acel punct la
piciorul perpendicularei pe acea dreapt.

89

a
A

ab, Aa
ab={A}
A este piciorul perpendicularei din A pe b
AA este distana de la punctul A la dreapta b i notm AA =d(A;b)
Definiia 2.1.14
Mediatoarea unui segment este dreapta perpendicular pe segment n mijlocul
acestuia.
Probleme rezolvate
R2.1.1 Se consider punctele A,O,B coliniare n aceast ordine. n acelai
semiplan determinat de dreapta AB se duc semidreptele [OC i [OD astfel nct [OC
este n interiorul unghiului AOD, iar m(COD)=900. Dac [OE i [OF sunt
bisectoarele unghiurilor AOC respectiv BOD s se afle msura unghiului EOF.
Soluie
AOB alungit m(AOB)=1800
m(AOC)+m(BOD) =1800 m(COD)=1800 900 =900

m(AOC)
2
m(BOD)
[OF bisectoarea BOD m(BOF)=m(FOD)=
2

[OE bisectoarea AOCm(AOE)=m(EOC)=

m(EOF)=m(COD)+m(EOC)+m(DOF)=
0
m(AOC) + m(BOD)
0 90
= 90 +
= 1350
= 90 +
2
2
0

R2.1.2 Se consider unghiul drept AOB i d o dreapt care trece prin O i nu


are puncte n interiorul unghiului sau pe laturile acestuia. Fie punctul C pe dreapta d,
situat n semiplanul determinat de dreapta OB i punctul A, iar D un punct pe dreapta d
astfel ca semidreptele [OC i [OD s fie opuse.
a) Demonstrai c unghiul AOC este ascuit;

90

b) Dac punctul M este n interiorul unghiului AOB astfel nct AOC s


fie congruent cu AOM, demonstrai c [OB este bisectoarea unghiului
MOD.
Soluie

M
D

a) COD unghi alungitm(COD)=1800


AOB unghi drept m(AOB)=900
m(AOC)=1800 [m(AOB)+m(BOD)]=
=1800 m(AOB) - m(BOD)= 1800 900 m(BOD)=
=900- m(BOD) m(AOC)<900AOC unghi ascuit
b) m(AOC)+m(BOD)=1800 m(AOB)=1800 900=900
m(BOD)=900 m(AOC) (1)
m(AOM)+m(MOB)=900 m(BOM)=900 m(AOM) (2)
m(AOM)=m(AOC) (3)
Din relaiile (1), (2) i (3) m(MOB)=m(BOD) [OB bisectoarea
MOD
R2.1.3 Se consider unghiul ascuit XOY. n semiplanul determinat de [OX
i n care nu se afl semidreapta [OY , se duc perpendicularele [OA o [OB respectiv
pe [OX i [OY. Se noteaz cu [OC bisectoarea unghiului BOX.
a) Dac msura AOC este cu 160 mai mare dect msura XOY,
determinai m( XOY);
b) Artai c dac [OB este bisectoarea AOC , atunci [OX este bisectoarea
COY

91

Y
Soluie

A
C
B
OAOX m(AOX)=900m(AOB)=900 m(XOB) (1)
OBOY m(BOY)=900 m(XOY)=900 m(BOX) (2)
Din relaiile (1) i (2) m(AOB)=m(XOY)
m(AOC)=m(AOB)+m(BOC)=m(XOY)+m(BOC) (3)
m(AOC) m(XOY)=160 (4)
nlocuind m(AOC) din relaia (3) n relaia (4) obinem:
m(XOY)+m(BOC) m(XOY)=160 m(BOC)=160
m(BOX)=2m(BOC)=320
m(XOY)=900 m(BOX)=900 320=580

Bibliografie
I. Petric i colectivul, Manual pentru clasa a VI-a, Ed. Petrion 1998
G. Turcitu, I. Rizea, C. Basarab, M. Duncea, Manual clasa a VI-a, Ed. Radical 1998
T. Udrea, D. Nuescu, Manual clasa a VI-a, EDP 1998
D. Brnzei, D. Zaharia, M. Zaharia : Aritmetic-Algebr-Geometrie, Ed.Paralela 45
2002, pag 110-124
C. Hrbor, D.Svulescu, I. Chec, A.ifrea: Matematic pentru clasele V-VIIIOlimpiadele judeene,interjudeene,naionale, Ed. Teora 1996, pag 108-109, 109-112
D. Brnzei, D. i M. Goleteanu, S. Ulmeanu, V. Gorgot, I. erdean: Matematica n
concursurile colare, Ed. Paralela 45, 2000,2001,2002
D. Constantinescu: Olimpiadele colare, Ed. Teora 1997-2002
D. Mihe, N. Angelescu, I. Chera, C. Popescu i colectivul: Olimpiadele de matematic
1990-1998, Ed. Gil Zalu, clasa a VI-a, pag 61-70,71-77
Edwin E. Moise, Floyd I. Downs jr. : Geometria , EDP 1983, pag 81-105

92

3. Geometria bazat pe raionament i demonstraii


n problemele de geometrie ne vom baza, n stabilirea unor proprieti pe
judecat (raionament). Punctele principale de plecare ale judecilor pe care le vom
face vor fi cazurile de congruen ale triunghiurilor oarecare. Scopul urmrit va fi acela
de a forma i dezvolta raionamentul geometric i de a deduce cu ajutorul lui
proprietile cele mai importante ale figurilor geometrice.
n matematic (deci i n geometrie) se ntlnesc unele propoziii care exprim
adevruri ce se admit fr demonstraii i care se numesc "axiome" (Cuvntul "axiom"
vine din limba greac: axioma = opinie, tez admis. Termenul a fost folosit, ncepnd
din sec. 6-5 .H. de ctre matematicii din coala lui Pitagora). Spre exemplu: "Prin dou
puncte distincte "trece" o singur dreapt" (axioma dreptei).
Propoziii matematice care exprim adevruri ce trebuie s fie dovedite se
numesc "teoreme" (cuvntul "teorem" vine din limba greac: theorema = examinare,
cercetare. Termenul a fost folosit pentru prima dat de filozoful grec Aristotel n sec. 4
.H.). Spre exemplu: "Dou unghiuri opuse la vrf sunt congruente".
n enunul fiecrei teoreme deosebim dou pri: "ipoteza" (sau premisa), care
este format din toate faptele pe care enunul teoremei le presupune adevrate i
"concluzia", care este format din ceea ce enunul teoremei afirm c se poate deduce
din ipotez. (Cuvntul "ipotez" este compus din dou cuvinte provenite din limba
greac: hypo = sub i thesis = punere. Cuvntul "premis" vine din limba latin:
praemisus = pus nainte, anterior. Cuvntul "concluzie" vine din limba latin: conclusio
= ncheiere).
n exemplul de mai sus, ipoteza este: "dou unghiuri sunt opuse la vrf", iar
concluzia: "aceste dou unghiuri sunt congruente".
n unele cazuri teoremele sunt enunate sub forma unor propoziii ipotetice
(condiionale): ipoteza ncepe cu cuvntul "dac", iar concluzia cu cuvntul "atunci".
Cum teoreme se ntlnesc i n aritmetic, nu numai n geometrie, vom da un exemplu
de teorem prezent sub forma unei propoziii ipotetice (condiionale), ntlnit la
aritmetic n clasa a V-a: "Dac un numr este divizibil cu 3 i cu 5, atunci el este
divizibil cu 15". Se reine cu uurin c "un numr este divizibil cu 3 i cu 5" este
ipoteza, iar "el este divizibil cu 15" este concluzia.
Teoremele trebuie s fie "demonstrate", adic adevrurile din concluzie trebuie
s fie "dovedite" cu ajutorul unor argumente, care sunt adevrurile din ipotez i alte
adevruri (axiome sau teoreme demonstrate anterior).
Pentru primul exemplu:
Ipotez. AOB i A'OB' opuse la vrf
Concluzie. AOBA'OB'.
B'

O
A'

93

Demonstraie. tim c m(AOB)+m(AOB')=180, iar m(A'OB') +


m(AOB') =180, deci m(AOB)=m(A'OB'), de unde rezult c AOBA'OB'.
Pentru al doilea exemplu:
Ipotez. nM3 , n M 5
Concluzie: nM15
Demonstraie. tim c: "dac n se divide cu a i b, iar a i b sunt prime ntre ele, atunci
n se divide cu produsul ab".
nM3, nM5, (3,5) = 1 nM15 .
Dac se schimb ntre ele ipoteza i concluzia unei teoreme, se obine o
propoziie nou, care se numete "propoziie reciproc" (Cuvntul "reciproc" vine din
limba latin: reciprocus = care se ntoarce de unde a venit, care inverseaz).
Spre exemplu, "reciproca" primului exemplu ar afirma c: "Dac dou unghiuri
sunt congruente, atunci ele sunt opuse la vrf". Aceast propoziie este fals, deoarece
concluzia ei nu este ntotdeauna adevrat. Pentru a dovedi falsitatea ei este suficient s
dm un singur exemplu din care s rezulte aceasta. Dac vom privi, spre exemplu,
unghiurile CDE i FGH, care sunt congruente (avnd aceeai msur), ne vom da
seama imediat c ele nu sunt opuse la vrf:

Un exemplu care arat c uneori concluzia unei propoziii nu este adevrat, se


numete contraexemplu.
Dac reciproca unei teoreme este o propoziie fals, atunci aceast reciproc nu
este teorem i deci teorema dat nu admite "teorem reciproc".
"Reciproca" celui de al doilea exemplu afirm c: "Dac un numr este
divizibil cu 15, atunci el este divizibil cu 3 i cu 5". Aceast propoziie este adevrat:
Ipotez. nM15
Concluzie: nM3 i nM5 .
Demonstraie. tim c: "dac n se divide cu m, atunci n se divide cu toi divizorii lui m".
nM15, 15M3, 15M5 nM3 i nM5 .
Dac reciproca unei teoreme este o propoziie adevrat, atunci aceast
reciproc devine "teorem reciproc".

94

Se obinuiete ca, fa de teorema reciproc, teorema iniial s se numeasc


"teorem direct". Se mai spune c cele dou teoreme sunt una reciproca celeilalte,
ceea ce nseamn c oricare dintre ele ar putea fi considerat teorem direct.
Dac teorema direct i reciproca ei sunt ambele adevrate, atunci le putem
concentra ntr-o singur teorem, folosind n formularea enunului expresia "dac i
numai dac". Iat un exemplu: "Un numr este divizibil cu 15 dac i numai dac el
este divizibil cu 3 i cu 5". Pentru demonstrarea unei astfel de teoreme trebuie s
facem, de fapt, dou demonstraii: directa i reciproca.
Sunt situaii cnd o teorem (propoziie) poate s admit mai multe reciproce.
Aceasta se ntmpl atunci cnd ipoteza sau concluzia teoremei (propoziiei) date (sau
chiar ambele) conine (conin) dou sau mai multe afirmaii (pri). n acest caz, numim
propoziie reciproc a unei propoziii date acea propoziie n care "ipoteza" este
format din concluzia propoziiei date (sau numai o parte a concluziei) i o parte din
ipoteza propoziiei date, iar "concluzia" este format din partea rmas a ipotezei
propoziiei date (i ceea ce a mai rmas din concluzia propoziiei date).
3.1 Congruena triunghiurilor
Cazurile de congruen a triunghiurilor oarecare
Fie ABC i MNP dou triunghiuri oarecare. Notaia ABCMNP o citim:
"triunghiul ABC este congruent cu triunghiul MNP" i nelegem prin aceasta ase
congruene, care au loc n acelai timp i anume:
[AB] [MN], [BC] [NP], [CA] [PM] ;
BAC NMP, ABC MNP, ACB MPN .
Pentru a scrie cele ase congruene se ine seama c:
1) Laturile i unghiurile celor dou triunghiuri se corespund n ordinea dat
(scris) de congruena celor dou triunghiuri. Ele se mai numesc i elemente (laturi sau
unghiuri) "omoloage". (Cuvntul "omolog" vine din limba greac: homologus = n
armonie).
2) Laturile i unghiurile celor dou triunghiuri congruente, care se corespund
(omoloage), sunt congruente.
Se observ c din ABCMNP nu rezult ABCMPN, dar rezult c
ACBMPN i nc alte patru astfel de relaii (BACNMP, BCANPM,
CABPMN, CBAPNM).
Ilustrm grafic elementele care sunt respectiv congruente:
M

95

Pentru a arta c dou triunghiuri sunt congruente nu este necesar s


demonstrm toate cele ase congruene ntre elementele lor.
Urmtoarele afirmaii, care se numesc "cazurile de congruen a triunghiurilor
oarecare" sunt adevrate:
Cazul 1 (LUL). Dou triunghiuri oarecare care au cte dou laturi i unghiul
cuprins ntre ele respectiv congruente sunt congruente.
M

Dac [AB][NM], [BC][NP] i ABCMNP, atunci rezult c


ABCMNP. Ca urmare, rezult c [AC][MP], ACBMPN i BACNMP.
Cazul 2 (ULU). Dou triunghiuri oarecare care au cte o latur i unghiurile
alturate ei respectiv congruente sunt congruente.
A

Dac [BC][NP], ABCMNP, ACBMPN, atunci rezult c


ABCMNP. Ca urmare, rezult c [AB][MN], [AC][MP] i BACNMP.
Cazul 3 (LLL). Dou triunghiuri oarecare care au laturile respectiv congruente
sunt congruente.

96

Dac [AB][MN], [BC][NP], [AC][MP], atunci rezult c ABCMNP.


Ca urmare, rezult c BACNMP, ABCMNP i ACBMPN.
Se obinuiete a se rezuma astfel: cazaul 1: LUL (adic, latur-unghi-latur),
cazul 2: ULU (unghi-latur-unghi) i cazul 3: LLL (latur-latur-latur).
Aceste trei cazuri rezult direct din construcia triunghiului.
Remarc. Cazul LUU (latur-unghi-unghi): Dou triunghiuri oarecare care au
cte o latur, unghiul opus ei i un unghi alturat ei, respectiv congruente sunt
congruente.

Dac [BC][NP], BACNMP, ABCMNP, atunci rezult c


ABCMNP. Ca urmare, [AB][MN], [AC][MP] i ACBMPN.
Observaii. Cazul LUU nu rezult din construcia triunghiului, dar
considernd c suma msurilor unghiurilor unui triunghi este 180, adic
m(BAC)+m(ABC)+m(ACB)=180
i m(NMP)+m(MNP)+m(MPN)=180 i innd cont c m(BAC)=m(NMP) i
m(ABC)=m(MNP), rezult c i m(ACB)=m(MPN). Deci, acest caz se reduce
la ULU.
Cazurile de congruen a triunghiurilor dreptunghice
Definiie. Triunghiul care are un unghi drept se numete triunghi dreptunghic.
Laturile unui triunghi dreptunghic ABC, (m(A)=90) se numesc:
catete laturile alturate unghiului drept
ipotenuz latura opus unghiului drept.

[AB], [AC] sunt catete


[BC] este ipotenuza.
Toate triunghiurile dreptunghice au cte un unghi drept. Este de ateptat ca n
cazul particular al triunghiurilor dreptunghice, att cazurile de construcie ct i
cazurile de congruen s fie redate ntr-o form simplificat.
97

Cazul C.C. (catet-catet): Dac dou triunghiuri dreptunghice au catetele


respectiv congruente, atunci ele sunt congruente.
C

C'

A'

B'

Fie triunghiurile dreptunghice ABC i A'B'C' cu m(BAC)=m(B'A'C')=90,


[AB][A'B'] i [AC][A'C']. Putem afirma conform cazului de congruen LUL a
triunghiurilor oarecare c ABCA'B'C'.
Acest caz este o aplicaie direct a cazului LUL de la congruena triunghiurilor
oarecare.
Cazul C.U. (catet-unghi): Dac dou triunghiuri dreptunghice au o catet i
un unghi ascuit, la fel aezat fa de catet, respectiv congruente, atunci ele sunt
congruente.
Avem dou posibiliti:
I. Triunghiurile dreptunghice ABC i A'B'C' au congruente o catet i unghiul
ascuit alturat ei:
C

C'

A'

B'

Fie triunghiurile dreptunghice ABC i A'B'C' cu m(BAC)=m(B'A'C')=90,


[AB][A'B'] i ABCA'B'C'. rezult, conform cazului de congruen ULU a
triunghiurilor oarecare, c ABCA'B'C'.
Acest caz este o aplicaie direct a cazului ULU de la congruena triunghiurilor
oarecare.
II. Triunghiurile dreptunghice ABC i A'B'C' au congruente o catet i unghiul
ascuit opus ei:
C

C'

A'

98

B'

Fie triunghiurile dreptunghice ABC i A'B'C' cu m(BAC)=m(B'A'C')=90,


[AB][A'B'] i ACBA'C'B'. Rezult, conform cazului de congruen LUU a
triunghiurilor oarecare, c ABCA'B'C'.
Acest caz este o aplicaie direct a cazului LUU de la congruena triunghiurilor
oarecare.
Cazul I.U. (ipotenuz-unghi): Dac dou triunghiuri dreptunghice au ipotenuza
i un unghi, diferit de unghiul drept, respectiv congruente, atunci sunt congruente.
C

C'

A'

B'

Fie triunghiurile ABC i A'B'C' cu m(BAC)=m(B'A'C')=90, [BC][B'C']


i ABCA'B'C'. Rezult, conform cazului de congruen LUU a triunghiurilor
oarecare, c ABCA'B'C'.
Acest caz este o aplicaie direct a cazului LUU de la congruena triunghiurilor
oarecare.
Cazul I.C. (ipotenuz-catet): Dac dou triunghiuri dreptunghice au ipotenuza
i o catet respectiv congruente, atunci ele sunt congruente.
C

C'

A'

B'

P'

Fie triunghiurile dreptunghice ABC i A'B'C' cu m(BAC)=m(B'A'C')=90,


[BC][B'C'] i [AC][A'C']. Trebuie s demonstrm c ABCA'B'C'.
Prelungim [AC] cu [AP][AC] i [A'C'] cu [A'P'][A'C']; cum [AC][A'C'],
rezult c [AP][A'P'] i [CP][C'P']. tim c [AC][AP], m(BAC)=m(BAP)=90,
[AB] latur comun, rezult ABCABP, conform cazului de congruen catetcatet. De aici, [BC][BP] (1).
Vom demonstra la fel c triunghiurile A'B'C' i A'B'P' sunt congruente:
[A'C'][A'P'], m(B'A'C')=m(B'A'P')=90 i [A'B'] latur comun, rezult c
A'B'C'A'B'P', conform cazului de congruen catet-catet. De aici, [B'C'][B'P'] (2).
Din [BC][B'C'] i relaiile (1) i (2) rezult c [BP][B'P'].
99

Vom demonstra c triunghiurile BPC i B'P'C' sunt congruente: [BC][B'C'],


[BP][B'P'] i [CP][C'P'], rezult conform cazului de congruen LLL c
BPCB'P'C'. De aici, ACBA'C'B'.
Acum putem arta c triunghiurile ABC i A'B'C' sunt congruente:
[AC][A'C'], [BC][B'C'] i ACBA'C'B', rezult conform cazului de congruen
LUL c ABCA'B'C', ceea ce trebuia s demonstrm.
Probleme rezolvate
R3.1.1 n figura alturat [AB][BD], [AC][DC]. Atunci:
a) ABCDBC
b) ABEDBE
c) ACEDCE.
A

Soluie. a) Dac [AB][BD], [AC][DC], [BC] latur comun, atunci rezult,


conform cazului de congruen LLL, c ABCDBC.
b) Din a), ABCDBC, rezult c ABCDBC sau ABEDBE. Dac
[AB][BD], ABEDBE i [BE] este latur comun, rezult conform cazului de
congruen LUL, c ABEDBE.
c) Din b), ABEDBE, rezult c [AE][DE]. Dac [AC][CD], [AE][DE]
i [CE] este latur comun rezult conform cazului de congruen LLL, c
ACEDCE.
R3.1.2 n figura alturat: APBBQA i [AC][BE]. Atunci:
a) APEBQC
B
A
b) BAEABC
c) APQBQP.
O
Soluie. a) tim c
APBBQA, de unde rezult c
[AP][BQ],
[PB][QA],
PABQBA. APBBQA i
PBAQAB.
tim
c E

C
100

[AC][BE] i [AQ][PB], iar prin scdere membru cu membru AC - AQ = BE - PB ,


de unde rezult c [QC][EP]. m(APE)=180-m(APB) i m(BQC)=180m(BQA), dar APBBQA, de unde rezult c APEBQC.
Am artat c [AP][BQ], [EP][CQ] i APEBQC, deci, conform cazului
de congruen LUL, rezult c APEBQC.
b) tim c PABQBA, iar din a) rezult c PAEQBC, iar prin
nsumare, m(PAB)+m(PAE)=m(QBA)+m(QBC), de unde rezult c
m(BAE)=m(ABC), deci BAEABC. Avem [AB] latur comun,
BAEABC i ABEBAQ (este de fapt, PBAQAB) i conform cazului de
congruen ULU rezult c BAEABC.
c) tim c [AP][BQ], [AQ][BP], din APBBQA i [PQ] este latur
comun, deci APQBQP, conform cazului LLL de congruen a triunghiurilor.
R3.1.3 n figura alturat: [DP][AP], [PB][PC]. Atunci:
a) DPCAPB
b) CDABAD
c) DCBABC.

C
D

P
A
B
Soluie. a) tim c [DP][AP], [PC][PB] i DPCAPB, fiind unghiuri
opuse la vrf, rezult c DPCAPB, conform cazului de congruen LUL.
b) Se d: DP=AP, PB=PC i prin adunare membru cu membru rezult
DP+PB=AP+PC, adic DB=AC, de unde rezult c [DB][AC].
tim c [DB][AC], [AB][DC] (din a) DPCAPB) i [AD] latur comun,
rezult c CDABAD, conform cazului LLL de congruen a triunghiurilor.
c) tim c [DB][AC], [DC][AB] (din a)) i [BC] latur comun, rezult c
DCBABC, conform cazului LLL de congruen a triunghiurilor.
R3.1.4 n figura alturat: DACBCA, DCABAC, [DM][BN].
Atunci:
a) ADCCBA
b) AMCCNA
c) ADMCBN.

101

Soluie. a) tim c DACBCA, DCABAC i [AC] latur comun,


rezult c ADCCBA, conform cazului ULU de congruen a triunghiurilor.
b) Din a) rezult c [AB][DC], tim c [DM][BN], deci
AB BN = DC DM , de unde AN=CM, deci [AN][CM]. Avem: [AN][CM],
[AC] latur comun i MCANAC, de unde rezult conform cazului LUL, c
AMCCNA.
c) Din a) rezult c [AD][BC]; din c) rezult c [AM][CN] i se d
[DM][BN], deci ADMCBN, conform cazului LLL de congruen a triunghiurilor.
3.2. Metoda triunghiurilor congruente

n geometrie se folosete o metod de stabilire a adevrului caracteristic


tiinelor matematice: demonstraia. Demonstraia este o succesiune de judeci, de
argumente prin care se stabilete un adevr. n particular metoda triunghiurilor
congruente este o metod de demonstraie prin care se demonstreaz de regul c dou
segmente sau dou unghiuri sunt congruente.
Pentru rezolvarea problemelor de geometrie:
citii cu atenie problemele i folosind instrumentele potrivite, desenai figura
geometric
figura desenat s respecte proporiile elementelor date n enumul problemei
figura s fie suficient de mare i clar
pentru a demonstra c dou segmente sau c dou unghiuri sunt congruente, cutai
s le ncadrai n dou triunghiuri a cror congruen poate fi demonstrat
vei trage concluzia c segmentele sau unghiurile respective sunt congruente, dac
sunt elemente omoloage n triunghiuri congruente.
Probleme rezolvate

R3.2.1 Fie segmentele [AE] i [BD] care au un punct comun C, astfel nct
[AC][CD] i [CB][CE]. S se demonstreze c:
a) [AB][DE]
b) DEBABE
102

c) CADCDA.
D
E

B
A

Ipotez: [AE][BD]={C}
[AC][CD]
[CB][CE]
Concluzie:
a) [AB][DE]
b) DEBABE
c) CADCDA.
Demonstraie. a) Pentru a demonstra c dou segmente sunt congruente cutm
s le ncadrm n dou triunghiuri a cror congruen poate fi demonstrat.
Vom demonstra c triunghiurile ACB i DCE sunt congruente, deoarece:
[AC][DC], [CB][CE] i ACBDCE, fiind unghiuri opuse la vrf; rezult
conform cazului de congruen LUL c ACBDCE, de unde rezult c [AB][DE].
b) Pentru a demonstra c dou unghiuri sunt congruente cutm s le ncadrm
n dou triunghiuri a cror congruen poate fi demonstrat. Vom arta c triunghiurile
DEB i ABE sunt congruente.
Avem [AC][CD] i [EC][CB], din ipotez, de unde AC+CE=DC+CB, deci
AE=DB, adic [AE][DB]. Deci, [AE][DB], [AB][DE] (din punctul a) al problemei)
i [BE] latur comun, rezult c DEBABE, conform cazului de congruen a
triunghiurilor LLL. De aici rezult c: DEBABE.
c) Pentru a demonstra c dou unghiuri sunt congruente cutm s le ncadrm
n dou triunghiuri a cror congruen poate fi demonstrat. Vom arta c triunghiurile
EAD i BDA sunt congruente.
Avem [AE][BD], [DE][AB] (din punctul a) al problemei) i [AD] latur
comun, rezult c EADBDA, conform cazului de congruen a triunghiurilor
LLL. De aici rezult c: CADCDA.
R3.2.2 n ABD, O este mijlocul laturii [BD]. Se prelungete segmentul [AO]
cu segmentul [OC] astfel nct CDBABD. S se arate c:
a) [DC][AB]
b) OBCODA.

103

C
O

A
Ipotez: ABD

O[BD], [OB][OD]
O[AC]
CDBABD
Concluzie:
a) [DC][AB]
b) OBCODA
Demonstraie. a) Vom demonstra c triunghiurile ODC i OBA sunt
congruente. Avem: [OD][OB], CDOABO (din ipotez) i DOCBOA, fiind
unghiuri opuse la vrf. Rezult conform cazului de congruen a triunghiurilor ULU c
ODCOBA. De aici rezult c [DC][AB].
b) Din ODCOBA rezult c [OC][OA]. Vom demonstra c triunghiurile
AOD i COB sunt congruente: [OD][OB] (din ipotez), [OA][OC] i
AODCOB, fiind unghiuri opuse la vrf. Rezult conform cazului de congruen a
triunghiurilor LUL c AODCOB. De aici rezult c ODAOBC.
R3.2.3 Fie segmentele congruente [OA] i [OB], astfel nct m(AOB)<90;
AEOB, EOB, BFOA, FOA, AEBF={P}.
S se demonstreze c:
a) [AE][BF]
b) [OP este bisectoarea unghiului AOB.
Ipotez: [OA][OB], m(AOB)<90
AEOB, EOB
BFOA, FOA
AEBF={P}
Concluzie:
a) [AE][BF]
b) AOPBOP

104

A
F

Demonstraie. a) Vom demonstra c triunghiurile AEO i BFO sunt


congruente. tim c [OA][OB], m(AEO)=m(BFO)=90 i AOB unghi comun
(din ipotez), de unde rezult conform cazului de congruen a triunghiurilor
dreptunghice I.U. c AEOBFO, de unde [AE][BF].
b) Din congruena AEOBFO rezult c [OE][OF]. tim c OA=OB,
OF=OE, rezult c OA-OF=OB-OE, deci FA=EB, de unde [FA][EB].
Avem: m(PFA)=m(PEB)=90 (din ipotez), [FA][EB] i FPAEPB
(unghiuri opuse la vrf), de unde rezuzlt conform cazului de congruen a
triunghiurilor dreptunghice C.U. c AFPBEP. De aici rezult c [PF][PE]. Avem:
[PF][PE], m(PFO)=m(PEO)=90 (din ipotez) i [OP] latur comun, de unde
rezult, conform cazului de congruen a triunghiurilor dreptunghice I.C. c
PFOPEO. De aici rezult c FOPEOP sau AOPBOP.
R3.2.4 Dac un punct aparine bisectoarei unui unghi, atunci el este egal
deprtat de laturile unghiului.
Reciproc: Dac un punct din interiorul unui unghi este egal deprtat de laturile
unghiului, atunci punctul aparine bisectoarei unghiului.
Ipotez: AOCCOB
M[OC
MEOA, EOA
MFOB, FOB
Concluzie:
[ME][MF]

105

A
E

C
M

Demonstraie. Vom demonstra c triunghiurile MEO i MFO sunt congruente:


m(MEO)=m(MFO)=90, MOEMOF (din ipotez), [OM] este latur comun,
rezult conform cazului de congruen a triunghiurilor dreptunghice I.U. c
MEOMFO. De aici rezult c [ME][MF].
Reciproc:
Ipotez: AOB
MIntAOB
MEOA, E[OA]
MFOB, F[OB]
[ME][MF]
MOAMOB
Concluzie:
Demonstraie. Vom demonstra c triunghiurile MEO i MFO sunt congruente:
m(MEO)=m(MFO)=90, [ME][MF] (din ipotez), [OM] este latur comun,
rezult conform cazului de congruen a triunghiurilor dreptunghice I.C. c
MEOMFO. De aici rezult c MOEMOF sau MOAMOB.
Observaie. Cele dou teoreme de mai sus pot fi formulate ntr-o singur
teorem care se numete proprietatea bisectoarei unui unghi: Un punct aparine
bisectoarei unui unghi dac i numai dac este egal deprtat de laturile unghiului.
Remarc. Se numete loc geometric o mulime de puncte care sunt
caracterizate printr-o aceeai proprietate. Deci, bisectoarea unui unghi este locul
geometric al punctelor din interiorul unghiului egal deprtate de laturile unghiului.
Pentru ca figura presupus s fie loc geometric, trebuie satisfcute urmtoarele
dou propoziii:
a) orice punct al figurii s aib proprietatea enunat
b) orice punct care are proprietatea enunat s aparin figurii.
R3.2.5 Dac un punct aparine mediatoarei unui segment, atunci el este egal
deprtat de extremitile segmentului.
Reciproc: Dac un punct este egal deprtat de extremitile unui segment,
atunci punctul aparine mediatoarei segmentului.
Ipotez: [AB]
dAB
106

Concluzie:

dAB={M}
[MA][MB]
Pd
[PA][PB]

d
Demonstraie. Vom demonstra c triunghiurile PMA i PMB sunt congruente:
m(PMA)=m(PMB)=90, [MA][MB] (din ipotez) i [PM] latur comun, rezult
c PMAPMB, conform cazului de congruen a triunghiurilor dreptunghice C.C.
De aici rezult c [PA][PB].
Reciproc:
Ipoteza: [AB]
[PA][PB]
Concluzie:
P aparine mediatoarei segmentului [AB].
Demonstraie. Construim perpendiculara din P pe AB, care intersecteaz AB n
M. Vom demonstra c triunghiurile PMA i PMB sunt congruente:
m(PMA)=m(PMB)=90, [PA][PB] (din ipotez) i [PM] latur comun, rezult c
PMAPMB, conform cazului de congruen a triunghiurilor dreptunghice I.C. De
aici rezult c [AM][MB], dar PMAB, rezult c PM este mediatoarea segmentului
[AB].
Observaie. Cele dou enunuri de mai sus pot fi formulate ntr-o singur
teorem care se numete proprietatea mediatoarei unui segment: Un punct aparine
mediatoarei unui segment dac i numai dac este egal deprtat de extremitile
segmentului.
Remarc. Mediatoarea unui segment este locul geometric al punctelor din plan
egal deprtate de extremitile segmentului.
R3.2.6 Fie triunghiurile ABC i A'B'C' n care construim ADBC, DBC,
A'D'B'C', D'B'C' i medianele [AM] i [A'M']. Dac [BC][B'C'], [AD][A'D'],
[AM][A'M'], demonstrai c ABCA'B'C'.
Ipotez: ABC, A'B'C'
ADBC, DBC
A'D'B'C', D'B'C'
M[BC], [MB][MC]
107

M'[B'C'], [M'B'][M'C']
[BC][B'C']
[AD][A'D']
[AM][A'M']
A'

B'

D'

M'

C'

Demonstraie. Dac [BC][B'C'] i M, M' sunt mijloacele segmentelor [BC],


respectiv [B'C'] rezult c [BM][MC][B'M'][M'C']. Putem demonstra c
triunghiurile
ADM
i
A'D'M'
sunt
congruente,
deoarece:
m(ADM)=m(A'D'M')=90, [AD][A'D'] i [AM][A'M'] (din ipotez), rezult,
conform cazului de congruen I.C. c ADMA'D'M'. De aici rezult c:
[DM][D'M']. Dar [BM][B'M'], de unde BM-DM=B'M'-D'M', deci BD=B'D', atunci
[BD][B'D'].
Putem demonstra c triunghiurile ADB i A'D'B' sunt congruente, deoarece:
m(ADB)=m(A'D'B')=90, [AD][A'D'] (din ipotez) i [BD][B'D'], rezult
conform cazului de congruen C.C. c ADBA'D'B'. De aici rezult c [AB][A'B']
i ABCA'B'C'.
Acum putem demonstra c triunghiurile ABC i A'B'C' sunt congruente,
deoarece: [AB][A'B'], ABCA'B'C' i [BC][B'C'] (din ipotez), rezult conform
cazului de congruen LUL c ABCA'B'C'.
3.3 Paralelism
3.3.1 Drepte paralele
Reamintim urmtoarele definiii i teoreme ce se vor folosi n acest paragraf:

Dou drepte distincte care au intersecia egal cu mulimea vid se numesc


drepte paralele.

Dac dou drepte intersectate de o secant formeaz o pereche de unghiuri


alterne interne congruente, atunci dreptele sunt paralele.

Dac dou drepte intersectate de o secant formeaz o pereche de unghiuri


alterne externe congruente, atunci dreptele sunt paralele.

Dac dou drepte intersectate de o secant formeaz o pereche de unghiuri


corespondente congruente, atunci dreptele sunt paralele.

Dac dou drepte intersectate de o secant formeaz o pereche de unghiuri


interne de aceeai parte a secantei suplementare, atunci dreptele sunt paralele.

108


Dac dou drepte intersectate de o secant formeaz o pereche de unghiuri
externe de aceeai parte a secantei suplementare, atunci dreptele sunt paralele.

Axioma paralelelor (axioma lui Euclid). Printr-un punct exterior unei drepte
date se poate duce o singur paralel la acea dreapt.
Observaie. Axioma lui Euclid ne asigur att de existena unei paralele dus
printr-un punct exterior la dreapt, ct i de unicitatea acestei paralele.
Euclid (Eukleides) a fost matematician grec (sec. 3 .H.). El a ntemeiat o
coal n Alexandria (Egipt). Este autorul primei expuneri sistematice a cunotinelor
de geometrie intitulat "Elemente", care a constituit cartea de cpti a geometriei timp
de 2000 de ani.

Consecinele axiomei paralelelor:


1) Dou drepte paralele cu o a treia sunt paralele ntre ele.
2) Dac dou drepte sunt paralele, atunci orice dreapt care se intersecteaz cu
una dintre ele se va intersecta i cu cealalt.

Dac dou drepte paralele sunt intersectate de o secant, atunci ele formeaz
unghiuri alterne interne congruente dou cte dou, unghiuri alterne externe
congruente dou cte dou, unghiuri corespondente congruente dou cte dou,
unghiuri interne de aceeai parte a secantei suplementare i unghiuri externe de aceeai
parte a secantei suplementare.
Remarc. Teoremele de mai sus pot fi enunate i n felul urmtor:
Dou drepte tiate de o secant sunt paralele dac i numai dac unghiurile:
a) alterne interne sunt congruente dou cte dou;
b) alterne externe sunt congruente dou cte dou;
c) corespondente sunt congruente dou cte dou;
d) interne de aceeai parte a secantei sunt suplementare;
e) externe de aceeai parte a secantei sunt suplementare.
Unghiuri cu laturile respectiv paralele
Teorem. Dou unghiuri cu laturile respectiv paralele sunt congruente dac
sunt ambele ascuite sau ambele obtuze i sunt suplementare dac unul este ascuit, iar
cellalt este obtuz.
A1

A'1

A'

O1

B1

A''
O'

B'1

B'

O'1
A 1''

B''

109

B1''

Demonstraie. n primul caz, cnd ambele unghiuri sunt ascuite, de exemplu:


fie AOB, A'O'B' dou unghiuri astfel nct OA||O'A' i OB||O'B'. Notnd
OBO'A'={C}, din OA||O'A' intersectate de secanta OB rezult AOBA'CB
(unghiuri corespondente), apoi din OB||O'B' intersectate de secanta O'A' rezult
A'CBA'O'B' (unghiuri corespondente), deci AOBA'O'B'.
Teorema este adevrat i pentru A''O'B'', care este opus la vrf un unghiul
A'O'B', adic avem AOBA''O'B''.
n al doilea caz, cnd un unghi este ascuit i cellalt obtuz, fie A1O1B1 i
A1' O1' B1' dou unghiuri astfel nct O1A1 || O1' A1' , O1B1 || O1' B1' , m(A1O1B1)<90,

m(A1' O1' B1' ) > 90 . Fie [O1' B1'' semidreapta opus semidreptei [O1' B1' ; rezult c
unghiurile A1O1B1 A1' O1' B1'' , fiind unghiuri cu laturile respectiv paralele ambele
ascuite, dar A1' O1' B1'' i A1' O1' B1' sunt unghiuri suplementare, atunci i A1O1B1
i A1' O1' B1' sunt suplementare.
Teorema este adevrat i pentru A1'' O1' B1'' , care este opus la vrf cu unghiul
A1' O1' B1' , adic avem m(A1O1B1 ) + m(A1'' O1' B1'' ) = 180 .
Model 1. Segmentel [AB] i [CD] au acelai mijloc O. S se demonstreze c
AC||BC.
Ipotez: ABCD={O}
[OA][OB]
[OC][OD]
Concluzie:
AC||BD
B

O
C

Demonstraie. Putem demonstra c triunghiurile AOC i BOD sunt congruente:


[OA][OB], [OC][OD] (din ipotez), AOCBOD (opuse la vrf), atunci
AOCBOD (cazul LUL), de unde rezult c OACOBD; deci, dreptele AC i
BD intersectate de secanta AB formeaz o pereche de unghiuri alterne interne
congruente, rezult c AC||BD.
Model 2. n triunghiul ABC, D(AC), E(AB), M(BC), astfel nct
DM||AB, EM||AC. Dac m(EMB)=67 i m(DMC)=43, s se calculeze msurile
unghiurilor triunghiului ABC.
Ipotez: ABC, D(AC), E(AB), M(BC)
DM||AB, EM||AC
m(EMB)=67
m(DMC)=43
Concluzie:
m(BAC)
110

m(ABC)
m(ACB)

Demonstraie. Dreptele paralele DM i AB intersectate de secanta BC


formeaz
o
pereche
de
unghiuri
corespondente
congruente,
deci
m(DMC)=m(ABC)=43; dreptele paralele EM i AC intersectate de secanta BC
formeaz
o
pereche
de
unghiuri
corespondente
congruente,
deci
m(EMB)=m(ACB)=67. Putem afla m(EMD):
m(EMD)=180-(m(DMC)+m(EMB)),
m(EMD)=180-(67+43)=180-110=70.
Dreptele paralele AB i DM tiate de secanta EM formeaz o pereche de
unghiuri alterne interne congruente, deci m(BEM)=m(EMD)=70; dreptele paralele
EM i AC tiate de secanta AB formeaz o pereche de unghiuri corespondente
congruente, deci m(BEM)=m(BAC)=70.
Msurile unghiurilor triunghiului ABC sunt: m(BAC)=70, m(ABC)=43
i m(ACB)=67.

111

Probleme rezolvate

R3.3.1 S se demonstreze c paralele duse prin vrfurile unui triunghi la


laturile opuse determin un triunghi n care vrfurile triunghiului dat sunt mijloace de
laturi.
Ipotez: ABC
APN, PN||BC
BPM, PM||AC
CMN, MN||AB
Concluzie:
[AP][AN]
[BP][BM]
[CM][CN]
A

Demonstraie. Dreptele paralele PN i BC intersectate de secanta AC formeaz


o pereche de unghiuri alterne interne congruente, NACACB; dreptele paralele AB
i MN intersectate de secanta AC formeaz o pereche de unghiuri alterne interne
congruente, ACNBAC.
Avem: NACACB, ACNBAC, [AC] latur comun, atunci
ABCCNA (cazul ULU), de unde rezult c:
(1)
[AN][BC]
i
(2)
[CN][AB]
Dreptele paralele PN i BC intersectate de secanta AB formeaz o pereche de
unghiuri alterne interne congruente, PABABC; dreptele paralele PM i AC
intersectate de secanta AB formeaz o pereche de unghiuri alterne interne congruente,
PBABAC.
Avem: PABABC, PBABAC, [AB] latur comun, atunci
ABCBAP (cazul ULU), de unde
(3)
[AP][BC]
i
(4)
[BP][AC]
Dreptele paralele AB i MN intersectate de secanta BC formeaz o pereche de
unghiuri alterne interne congruente, BCMABC; dreptele paralele PM i AC
intersectate de secanta BC formeaz o pereche de unghiuri alterne interne congruente,
CBMACB.

112

Avem: BCMABC, CBMACB, [BC] latur comun, atunci


MCBABC (cazul ULU), de unde
(5)
[CM][AB]
i
(6)
[BM][AC]
Din relaiile (1) i (3) rezult c [AN][AP].
Din relaiile (2) i (5) rezult c [CN][CM].
Din relaiile (4) i (6) rezult c [BP][BM].
R3.3.2 n triunghiul ABC, [BD] este median, D(AC). Construim CE||BD,
unde EAB. Demonstrai c B este mijlocul segmentului [AE].
Ipotez: ABC
D(AC), [DA][DC]
CE||BC, EAB
Concluzie:
[AB][BE]
A

D
B

F
E

Demonstraie. Pentru demonstrarea acestei probleme facem urmtoarea


construcie auxiliar: BF||AC, F(CE).
Vom demonstra c triunghiurile BCD i CBF sunt congruente: DBCBCF
(unghiuri alterne interne, BD||CE i BC secant), DCBCBF (unghiuri alterne
interne, BF||AC i BC secant), [BC] latur comun, atunci BCDCBF (cazul
ULU), de unde rezult c [CD][BF], dar [CD][AD] (din ipotez), deci [AD][BF].
Vom demonstra c triunghiurile FBE i DAB sunt congruente: [AD][BF],
EBFBAD (unghiuri corespondente, BF||AC i AE secant), EFBECA
(unghiuri corespondente, BF||AC i EC secant), dar ECABDA (unghiuri
corespondente, BD||EC i AC secant), deci EFBBDA; atunci FBEDAB
(cazul ULU), de unde rezult c [BE][AB].
R3.3.3 Fie M, N, P mijloacele laturilor [AB], [BC], [AC] ale unui triunghi
ABC. Fie D i E astfel nct [MD][MC], M(DC), [PE][PN], P(NE). S se
demonstreze c punctele A, D, E sunt coliniare.
Ipotez: ABC
M(AB), [MA][MB]
N(BC), [NB][NC]
113

Concluzie:

PAC), [PA][PC]
M(DC), [MC][MD]
P(EN), [PN][PE]
A, D, E coliniare
A

Demonstraie. Vom demonstra c triunghiurile MAD i MBC sunt congruente:


[MA][MB], [MD][MC] (din ipotez), AMDBMC (unghiuri opuse la vrf),
atunci MADMBC (cazul LUL), de unde rezult c MABMBC, deci dreptele
AD i BC intersectate de secanta AB formeaz o pereche de unghiuri alterne interne
congruente, rezult c AD||BC.
Vom demonstra c triunghiurile APE i CPN sunt congruente: [AP][PC],
[PE][PN] (din ipotez), APENPC (unghiuri opuse la vrf), atunci APECPN
(cazul LUL), de unde rezult c EAPPCN, deci dreptele AE i BC intersectate de
secanta AC formeaz o pereche de unghiuri alterne interne congruente, rezult c
AE||BC.
Avem AD||BC, AE||BC i conform axiomei paralelelor, dreptele AD i AE
coincid, deci A, D, E sunt coliniare.
3.3.2. Linia mjlocie ntr-un triunghi

ntr-un triunghi, segmentul determinat de mijloacele a dou laturi se numete


linie mijlocie.
Remarc. Un triunghi are trei linii mijlocii.
A

Fie M, N, P mijloacele laturilor [AB], [AC], respectiv [BC] ale triunghiului


ABC. Segmentele [MN], [NP], [PM] sunt cele trei linii mijlocii ale triunghiului ABC.
Teorem. Segmentul care unete mijloacele a dou laturi ale unui triunghi este
paralel cu cea de a treia latur i are lungimea egal cu jumtate din lungimea acestei
laturi.

114

Conform acestei teoreme, avem MN||BC, MN =


MP||AC, MP =

BC
AB
, NP||AB, NP =
,
2
2

AC
.
2

Reciproca. Dac prin mijlocul M al laturii [AB] din triunghiul ABC, se duce

MN||BC, N(AC), atunci N este mijlocul laturii (AC) i MN =

BC
.
2

N'

Demonstraie. Demonstraia se face prin metoda reducerii la absurd.


Presupunem c N nu este mijlocul laturii [AC], atunci exist un punct N', N'(AC),
N'N, astfel nct [N'A][N'C]. Conform definiiei liniei mijlocii, rezult c [MN'] este
linie mijlocie, deci MN'||BC, dar MN||BC (din ipotez) i MN'MN; contradicie cu
axioma paralelelor.
Presupunerea fcut este fals, rezult c N este mijlocul lui [AC]; deci [MN]
este linie mijlocie, atunci MN =

BC
.
2

Model. Fie M, N, P mijloacele laturilor [AB], [AC] respectiv [BC] ale


triunghiului ABC.
a) Artai c unghiurile triunghiurilor AMN, MBP, NPC, PNM sunt
congruente cu unghiurile triunghiului ABC.
b) Triunghiurile AMN, MBP, NPC, PNM sunt congruente. (Triunghiul PNM
se numete triunghi median sau triunghi complementar triunghiului ABC, M, N, P
fiind "picioarele" medianelor triunghiului ABC).
Ipotez: ABC
M(AB), [MA][MB]
N(AC), [NA][NC]
P(BC), [PB][PC]
Concluzie:
a) AMN, MBP, NPC, PNM au unghiurile congruente
cu unghiurile ABC
b) AMNMBPNPCPNM.

115

Demonstraie. a) Conform definiiei liniei mijlocii, [MN], [NP] i [MP] sunt


liniile mijlocii ale triunghiului ABC, deci MN||BC, NP||AB i MP||AC.
n triunghiul AMN avem: MANBAC, AMNABC (unghiuri
corespondente, MN||BC, AB secant), ANMACB (unghiuri corespondente,
MN||BC, AC secant).
n triunghiul MBP, la fel, avem: MBPABC, BMPBAC,
BPMACB (unghiuri corespondente, MP||AC, AB secant, respectiv BC).
n triunghiul NPC, la fel, avem: PCNACB, PNCBAC,
NPCABC (unghiuri corespondente, NP||AB, AC secant, respectiv BC).
n triunghiul MPN avem: MPNPNC (alterne interne, MP||AC, PN
secant), dar PNCBAC, atunci MPNBAC; MNPNPC (alterne interne,
MN||BC, PN secant), dar NPCABC, atunci MNPABC; NMPMPB
(alterne interne, MN||BC, MP secant), dar MPBACB, atunci NMPACB.
c) Segmentele [MN], [NP], [MP] sunt liniile mijlocii ale triunghiului ABC,
rezult

MP =

AC
= AN = NC .
2

MN =

BC
= BP = PC ,
2

NP =

AB
= AM = MB ,
2

Avem AMNMBPNPCPNM (cazul LLL).


Probleme rezolvate

R3.3.4 n triunghiul ABC punctele D i E sunt mijloacele laturilor [AB],


respectiv [BC]. Dac G este punctul de intersecie al dreptelor AE i CD, demonstrai
c 2DG=GC i 2EG=GA.
Ipotez: ABC
E(BC), [BE][EC]
D(AB), [DA][DB]
AECD={G}
Concluzie:
2DG=GC
2EG=GA

116

M
D
G
N
B

Demonstraie. Fie M i N mijloacele segmentelor [AG], respectiv [CG].


Segmentele [MN] i [DE] sunt linii mijlocii n triunghiurile GAC, respectiv ABC, de
unde rezult c MN||AC, MN =

AC
AC
i DE||AC, DE =
, deci MN||DE,
2
2

[MN][DE].
Vom demonstra c triunghiurile GMN i GED sunt congruente: [MN][DE],
GMNGED, GNMGDE (alterne interne, MN||DE, ME secant, respectiv
ND), atunci GMNGED, de unde rezult c [GM][GE] i [GN][GD], dar
[GM][AM] i [GN][NC], deci GA=2GM=2EG i GC=2GN=2GD.
R3.3.5 S se demonstreze c dreapta determinat de vrful A al unui triunghi
ABC i mijlocul medianei din B intersecteaz latura [BC] ntr-un punct E, astfel nct

1
BE = BC .
3
Ipotez: ABC
D(AC), [DA][DC]
P(BD), [PB][PD]
APBC={E}
Concluzie:

1
BE = BC
3
A

Demonstraie. Fie DF||AE, F(BC).


n AEC aplicm reciproca liniei mijlocii: D este mijlocul laturii [AC],
DF||AE, F(EC), atunci F este mijlocul laturii [EC], deci [EF][FC].
117

n BDF aplicm reciproca liniei mijlocii: P este mijlocul laturii [BD], PE||DF,
E(BF), atunci E este mijlocul laturii [BF], deci [BE][EF], dar [EF][FC], rezult c

1
BE = EF = FC = BC .
3
R3.3.6 Dou triunghiuri ABC i ADE au mediana [AM] comun,
{M}=BCDE. Fie P, Q, R, S, respectiv mijloacele laturilor [AB], [AC], [AD], [AE].
Artai c RQ||PS i [RQ][PS].
Ipotez: ABC, ADE
{M}=BCDE
[MB][MC]
[MD][ME]
P(AB), [PA][PB]
Q(AC), [QA][QC]
R(AD), [RA][RD]
S(AE), [SA][SE]
Concluzie:
RQ||PS, [RQ][PS]
A
R
Q
D

S
C
M

Demonstraie. Din ipotez avem [MB][MC] i [ME][MD], iar


BMECMD (unghiuri opuse la vrf), atunci BMECMD, de unde rezult c
[BE][CD] i MBEMCD, deci dreptele CD i BE intersectate de secanta BC
formeaz o pereche de unghiuri alterne interne congruente, rezult c ele sunt paralele,
CD||BE. Segmentele [RQ] i [PS] sunt linii mijlocii n triunghiurile ADC, respectiv
ABE, deci RQ||DC, RQ =

1
1
DC i PS||BE, PS = BE , dar DC||BE i [DC][BE],
2
2

rezult c RQ||PS i [RQ][PS].

3.3.3. Suma msurilor unghiurilor unui triunghi

n rezolvarea problemelor din acest paragraf vom folosi urmtoarele rezultate,


demonstrate la orele de geometrie:

Suma msurilor unghiurilor unui triunghi este 180.


Consecine:
118

1) Toate unghiurile triunghiului echilateral au msura de 60.


2) n orice triunghi dreptunghic, unghiurile ascuite sunt complementare.
Unghiurile ascuite ale unui triunghi dreptunghic isoscel au msura de 45.
3) n orice triunghi poate exista cel mult un unghi drept sau obtuz.

Msura unui unghi exterior al unui triunghi este egal cu suma msurilor celor
dou unghiuri ale triunghiului, neadiacente lui.
Remarc. Bisectoarea unui unghi exterior al unui triunghi se numete
bisectoare exterioar a triunghiului, corespunztoare unghiului respectiv.
[BE este bisectoare interioar
[BF este bisectoare exterioar
A
F
E

Bisectoarea interioar i bisectoarea exterioar duse din acelai vrf al unui


triunghi sunt perpendiculare.
Demonstraie. Unghiurile ABC i ABD sunt adiacente suplementare, deci

m(ABC)+m(ABD)=180. de unde avem

1
1
m(ABC) + m(ABD) = 90 ,
2
2

adic m(ABE)+m(ABF)=90, deci m(EBF)=90, de unde rezult c BEBF.


Probleme rezolvate

R3.3.7 n interiorul unui triunghi isoscel ABC, [AB][AC] i


m(BAC)<120, se consider un punct M, astfel nct m(MBC)=30 i
m(MCB)=15. Notm cu P intersecia dreptei BM cu nlimea din A a triunghiului
ABC. S se afle msurile unghiurilor PMC i BPC.
Ipotez: ABC: [AB][AC]
m(BAC)<120
MIntABC
m(MBC)=30
m(MCB)=15
ADBC, D(BC)
ADBM={P}
Concluzie:
m(PMC), m(BPC)

119

Demonstraie. Unghiul PMC este unghi exterior triunghiului MBC, de unde


rezult c m(PMC)=m(MBC)+m(MCB)=30+15=45.
Vom demonstra c triunghiurile ADB i ADC sunt congruente:
m(ADB)=m(ADC)=90, [AB][AC] (din ipotez), [AD] este latur comun, atunci
ADBADC (cazul I.C.), de unde rezult c [DB][DC].
Acum putem demonstra congruena triunghiurilor PDB i PDC:
m(PDB)=m(PDC)=90 (din ipotez), [DB][DC], [PD] latur comun, atunci
PDBPDC (C.C.), de unde rezult c PBCPCB, dar m(PBC)=30 (din
ipotez), atunci
m(BPC)=180-2m(PBC)=180-230=120.
R3.3.8 n triunghiul ABC avem m(B)=3m(A). Mediatoarea laturii [BC]
intersecteaz dreapta AC n punctul E, astfel nct BAEBEA. S se calculeze
msurile unghiurilor triunghiului ABC.
Ipotez: ABC: m(B)=3m(A)
F(BC), [FB][FC]
FEBC, E(AC)
BAEBEA
Concluzie: m(BAC), m(ABC), m(ACB)
A

Demonstraie. Notm m(BAC)=x, atunci m(ABC)=3x i m(BEA)=x.


Dac suma msurilor unghiurilor unui triunghi este 180, atunci m(ACB)=180deci
m(ACB)=180-4x
i
m(ABE)=180(m(BAC)+m(ABC)),
120

(m(BAE)+m(BEA)), deci m(ABE)=180-2x. Atunci, m(EBC)=m(ABC)m(ABE), deci m(EBC)=3x-(180-2x), efectund calculele se obine
(1)
m(EBC)=5x-180.
Vom demonstra c triunghiurile EFB i EFC sunt congruente:
m(EFB)=m(EFC)=90, [FB][FC] (din ipotez), [EF] latur comun, atunci
EFBEFC (cazul C.C.), de unde rezult c EBFECB, dar
m(ECB)=m(ACB)=180-4x, atunci
(2)
m(EBF)=m(EBC)=180-4x
Din relaiile (1) i (2) deducem c 5x-180=180-4x sau 9x=360, de unde
x=40.
Vom avea: m(ABC)=340=120, m(BAC)=40, m(C)=20.
R3.3.9 Unghiurile A, B, C ale unui triunghi ABC au msurile invers
proporionale cu numerele 0,(3);

1
; 0,125. Fie M(AB) i N(AC), astfel nct
7

m(ACM)=40 i m(ABN)=20. S se determine msurile unghiurilor triunghiului


ABC i msura unghiului ANM.

1 1 1
3 7 8

Ipotez: {m(A),m(B),m(C)}, i.p. , ,

Concluzie:

M(AB), N(AC)
m(ACM)=40
m(ABN)=20
m(A), m(B), m(C)
m(ANM)

1 1 1
3 7 8

Demonstraie. ntre mulimile {m(A),m(B),m(C)} i , , se


stabilete o proporionalitate invers, atunci ntre {m(A),m(B),m(C)} i {3,7,8}
se
stabilete
o
proporionalitate
direct,
deci
se
poate
scrie:

121

m(A) m(B) m(C)


=
=
, de unde aplicnd proprietatea irului de rapoarte egale,
3
7
8
avem mai departe:

m(A) m(B) m(C) m(A) + m(B) + m(C ) 180


=
=
=
=
= 10
3
7
8
3+ 7 +8
18
m(A)
m(B)
= 10 , rezult c m(A)=30; din
= 10 , rezult c
Din
3
7
m ( C )
= 10 , rezult c m(C)=80.
m(B)=70; din
8

tim c m(ACM)=40, atunci m(MCB)=m(ACB)-m(ACM), efectund


obinem m(MCB)=40.
tim c m(ABN)=20, atunci m(NBC)=m(ABC)-m(ABN), efectund
obinem m(NBC)=50.
Fie BNCM={P}. n triunghiul PBC:
m(PBC)+m(PCB)=50+40=90,
deci m(BPC)=90, de unde rezult c CPBN. n triunghiul CPN, m(PCN)=40 i
m(CPN)=90, rezult c
m(CNP)=90-m(PCN)=50.
Vom demonstra c triunghiurile CPB i CPN sunt congruente:
m(CPB)=m(CPN)=90, [CP] latur comun i m(CBP)=m(CNP)=50, atunci
CPBCPN (cazul C.U.), de unde rezult c [PB][PN]. Acum putem demonstra c
triunghiurile MPB i MPN sunt congruente: m(MPB)=m(MPN)=90, [MP] latur
comun i [PB][PN], atunci MPBMPN (cazul C.C.), de unde rezult c
MNPMBP, dar m(MBP)=20, de unde rezult c MNPMBP, dar
m(MBP)=20, atunci m(MNP)=20. Acum putem afla m(ANM):
m(ANM)=180-m(MNP)+m(PNC)), nlocuind obinem m(ANM)=180(20+50), deci m(ANM)=110.
3.4. Triunghiul isoscel i triunghiul echilateral

n rezolvarea problemelor din acest paragraf vom folosi urmtoarele definiii i


teoreme ce sunt demonstrate n manuale:
Definiie. Triunghiul care are dou laturi congruente se numete triunghi
isoscel.
Notaie. ABC, [AB][AC]
[BC] se numete baza triunghiului isoscel.

122

Teorem. Unghiurile opuse laturilor congruente ale unui triunghi isoscel sunt
congruente.
Reciproc. Dac un triunghi are dou unghiuri congruente, atunci el este
triunghi isoscel (laturile opuse unghiurilor congruente sunt congruente).
Remarc. Un triunghi este isoscel dac i numai dac are dou unghiuri
congruente.
Teorem. Dac un triunghi este isocel i se consider bisectoarea unghiului
opus bazei, atunci ea este i mediana corespunztoare bazei i nlimea corespunztoare bazei i este inclus n mediatoarea bazei.
Afirmaiile de mai sus rmn valabile i pentru mediana corespunztoare bazei
i pentru nlimea corespunztoare bazei.
Teorem. Dac un triunghi este isoscel i se
A
consider mediana corespunztoare bazei, atunci ea
este i bisectoarea unghiului opus bazei i nlimea
corespunztoare bazei i este inclus n mediatoarea
bazei.
Demonstraie. Fie ABC isoscel, [AB][AC]
i [AD] mediana corespunztoare bazei [BC]; D(BC)
i [DB][DC]. Avem conform cazului de congruen a
triunghiurilor LLL c ADBADC, de unde rezult
BADCAD,
ADBADC,
dar
c
D
deci B
m(ADB)+m(ADC)=180,
C
m(ADB)=m(ADC)=90, ADBC; ADBC i
[DB][DC], D(BC), rezult c dreapta AD este mediatoarea segmentului [BC].
Teorem. Dac un triunghi este isoscel i se consider nlimea
corespunztoare bazei, atunci ea este i bisectoarea unghiului opus bazei i mediana
corespunztoare bazei i este inclus n mediatoarea bazei.
Demonstraie. Fie ABC isoscel, [AB][AC] i [AD] nlimea
corespunztoare bazei, ADBC, D(BC).

123

Avem conform cazului de congruen a


A
triunghiurilor dreptunghice I.C., c ADBADC,
de unde rezult c BADCAD i [BD][CD];
ADBC i [DB][DC], D(BC), rezult c dreapta
AD este mediatoarea segmentului [BC].
Observaie. n triunghiul isoscel ABC,
[AB][AC], dreapta AD, care conine att
bisectoarea unghiului BAC, ct i nlimea,
mediana i mediatoarea corespunztoare laturii [BC],
este ax de simetrie a triunghiului.
D
C
Vom demonstra i alte proprieti ale B
triunghiului isoscel.
Teorem. Dac ntr-un triunghi bisectoarea unui unghi este i mediana
corespunztoare laturii opuse unghiului, atunci triunghiul este isoscel.
Ipotez: ABC
BADCAD, D(BC)
[DB][DC]
Concluzie:
[AB][AC]

Demonstraie.
Demonstrarea
acestei
teoreme necesit o construcie auxiliar: fie EAD,
D(AE), [AD][DE].
Vom demonstra c triunghiurile ADB i
EDC sunt congruente: [AD][DE] (prin
construcie), [BD][CD] (din ipotez) i
ADBEDC (unghiuri opuse la vrf); rezult
conform cazului de congruen a triunghiurilor
LUL c ADBEDC. De aici, rezult c
[AB][CE] i BADCED, dar BADCAD
(din ipotez), atunci CEDCAD, deci CAE
este un triunghi isoscel, [CA][CE]. Avem:
[AB][CE] i [AC][CE], de unde rezult c
[AB][AC].
Teorem. Dac ntr-un triunghi bisectoarea
unui unghi este i nlime, atunci triunghiul este
isoscel.
Ipotez: ABC

BADCAD, D(BC)

124

Concluzie:

ADBC
[AB][AC]

Demonstraie. Se poate demonstra c triunghiurile ADB i ADC sunt


congruente: m(ADB)=m(ADC)=90 (din ipotez), BADCAD (din ipotez),
[AD] latur comun; rezult conform cazului de congruen a triunghiurilor
dreptunghice C.U. c ADBADC, de unde [AB][AC].
Teorem. Dac ntr-un triunghi mediana corespunztoare unei laturi este i
nlime, atunci triunghiul este isoscel.
Ipotez: ABC
D(BC), [DB][DC]
ADBC
Concluzie:
[AB][AC]

Demonstraie. Vom demonstra c triunghiurile ADB i ADC sunt congruente:


m(ADB)=m(ADC)=90 (din ipotez), [DB][DC] (din ipotez), [AD] latur
comun; rezult conform cazului de congruen a triunghiurilor dreptunghice C.C. c
ADBADC, de unde [AB][AC].

125

n rezolvarea problemelor din acest paragraf sunt utile definiia triunghiului


echilateral i urmtoarele proprieti ale sale, considerate cunoscute:
Definiie. Triunghiul care are toate laturile congruente se numete triunghi
echilateral.
Notaie. [AB][BC][AC]

Teorem. Unghiurile unui triunghi echilateral sunt congruente, avnd msurile


egale cu 60.
Reciproc. Dac ntr-un triunghi unghiurile sunt congruente, atunci triunghiul
este echilateral.
Observaie. Reciproca de mai sus se poate enuna i n felul urmtor: dac un
triunghi are dou unghiuri cu msurile de 60, atunci el este echilateral. (Este evident
c i al treilea unghi al triunghiului are msura de 60, deci cele trei unghiuri ale
triunghiului sunt congruente).
Remarc. Un triunghi isoscel care are un unghi cu msura de 60 este un
triunghi echilateral.
Comparnd definiia triunghiului echilateral cu cea a triunghiului isoscel vor
rezulta noi proprieti specifice triunghiului echilateral (deoarece triunghiul echilateral
poate fi considerat ca fiind triunghi isoscel cu oricare din laturi ca baz):
Teorem. ntr-un triunghi echilateral toate liniile importante ce pornesc din
acelai vrf coincid.
Observaie. Triunghiul echilateral are trei axe de simetrie.

126

Cu ajutorul proprietilor triunghiului isoscel i echilateral putem demonstra


dou proprieti ale triunghiului dreptunghic ce vor fi foarte des folosite n rezolvarea
problemelor de geometrie.
Teorem. Dac ntr-un triunghi dreptunghic msura unui unghi este de 30,
atunci lungimea catetei opuse acestui unghi este jumtate din lungimea ipotenuzei.
Ipotez: ABC: m(A)=90
m(B)=30
Concluzie:

AB =

BC
2

Demonstraie. Fie C'AC, astfel nct A(CC'), [AC][AC'].


n triunghiul BCC', [BA] este nlime (din ipotez) i median (din
construcie), deci el este un triunghi isoscel, dar m(BCA)=60, rezult c BCC este
triunghi echilateral. Avem AC =

AC =

CC'
(din construcie), dar CC'=BC, rezult c
2

BC
.
2

Teorem. ntr-un triunghi dreptunghic, lungimea medianei corespunztoare


ipotenuzei este jumtate din lungimea ipotenuzei.

127

Ipotez: ABC: m(A)=90


O(BC), [OB][OC]
Concluzie:

AO =

BC
2

O
A

Demonstraie. Fie PAO, astfel nct O(AP) i [OA][OP]. Vom demonstra


c triunghiurile AOB i POC sunt congruente: [AO][PO] (prin construcie),
[BO][CO] (din ipotez), AOBCOP (unghiuri opuse la vrf); rezult conform
cazului de congruen a triunghiurilor LUL c AOBPOC, de unde [AB][PC] i
OABOPC; din ultima congruen rezult c CP||AB (deoarece tiate de secanta
AP formeaz o pereche de unghiuri alterne interne congruente). Avem ABAC,
CP||AB, atunci CPAC. Vom demonstra c triunghiurile PCA i BAC sunt
congruente: m(PCA)=m(BAC)=90, [AC] latur comun, [PC][BA]; atunci,
conform cazului de congruen a triunghiurilor dreptunghice C.C. avem c
PCABAC, deci [AP][BC] i cum AO =

AP
BC
, rezult c AO =
.
2
2

Observaie. Se remarc relaia [AO][BO][CO], punctul O este egal deprtat


de punctele A, B, C, este centrul cercului circumscris triunghiului dreptunghic ABC.

128

Probleme rezolvate

R3.4.1 Se d triunghiul isoscel ABC, cu [AB][AC]. Pe latura [AB] se iau


punctele M, N, P, astfel nct [AM][MN][NP][PB] i pe latura [AC] se iau punctele
E, F, G, astfel nct [AE][EF][FG][GC]. Fie {S}=PEBC, {T}=MGBC i
{H}=EPMG. Artai c:
a) [MG][PE]; b) [MT][ES]; c) AHBC; d) SMPTEG.
Ipotez: ABC: [AB][AC]
M, N, P(AB)
[AM][MN][NP][PB]
E, F, G(AC)
[AE][EF][FG][GC]
PEBC={S}
MGBC={T}
EPMG={H}
Concluzie:
a) [MG][PE]
b) [MT][ES]
c) AHBC
d) SMPTEG.

Demonstraie. a) Avem

AM = MN = NP = PB =

AB
AC
i AE = EF = FG = GC =
,
4
4

dar AB=AC, de unde rezult c


129

AM = MN = PB = AE = EF = FG = GC =

AB
(1).
4

Vom demonstra c triunghiurile AMG i AEP sunt congruente: [AM][AE],


A este unghi comun, AG = AP =

3AB
(din relaia (1)), atunci, conform cazului de
4

congruen a triunghiurilor LUL avem c AMGAEP, de unde rezult c


[MG][PE].
b) Din AMGAEP, rezult c AMGAEP, de unde i suplementele lor
vor fi egale, deci BMTCES.
Avem: BMTCES, MBTECS (la baza triunghiului isoscel ABC),

MB = EC =

3AB
(din relaia (1)). Atunci, conform cazului de congruen a
4

triunghiurilor ULU, MBTECS, de unde rezult c [MT][ES].


c) Din MBTECS rezult c MTSEST, atunci triunghiul HST este
isoscel, avnd dou unghiuri congruente, deci [HT][HS]; dar [MT][ES] i scznd
membru cu membru obinem MT-HT=ES-HS, de unde MH=EH. Avem:

AM = AE =

AB
(din relaia (1)), [MH][EH], [AH] latur comun, atunci
4

AMHAEH (cazul LLL de congruen a triunghiurilor), de unde rezult c


MAHEAH. Avem: n triunghiul isoscel ABC de baz [BC], [AH este bisectoarea
unghiului BAC, deci AH este i nlimea corespunztoare bazei, prin urmare AHBC.
d) Din AEPAMG, rezult c APEAGM, deci ele au suplemente
egale, prin urmare MPSEGT.
Din AEPAMG, rezult c [PE][GM]; avem [ES][MT] i scznd
membru cu membru se obine ES-PE=MT-GM, de unde [PS][GT].

Avem: MP = EG =

AB
(din relaia (1)), MPSEGT, [PS][GT], atunci,
2

rezult conform cazului de congruen LUL a triunghiurilor c SMPTEG.


R3.4.2 Demonstrai c dou triunghiuri ABC i A'B'C' care au perimetrele
egale, [CA][C'A'] i CC', sunt congruente.
Ipotez: ABC, A'B'C'
PABC=PA'B'C'
[CA][C'A]
CC'
ABCA'B'C'
Concluzie:

A'

D'

130

B'

C'

Demonstraie. Se consider punctele D i D' astfel nct B(DC), [BD][AB],


[B'D'][A'B'];
DC=DB+BC=AB+BC=PABC-AC
i
B'(D'C'),
D'C'=D'B'+B'C'=A'B'+B'C'=PA'B'C'-A'C', dar [AC][A'C'] (din ipotez), de unde rezult
c [DC][D'C'].
Vom demonstra c triunghiurile ADC i A'D'C' sunt congruente: [DC][D'C'],
[AC][A'C'] i CC' (din ipotez); atunci avem ADCA'D'C' (cazul LUL), de
unde rezult c ADCA'D'C' i DACD'A'C'.
Triunghiul ABD este isoscel, [AB][BD], atunci unghiurile de la baz sunt
congruente, BDABAD; la fel, triunghiul A'B'D' este isoscel, [A'B'][B'D'], atunci
B'D'A'B'A'D', dar ADBA'D'B', deci ADBA'D'B'BADB'A'D' i
DACD'A'C', de unde rezult c BACB'A'C' (diferene de unghiuri
congruente).
Avem: BACB'A'C', [AC][A'C'] i CC', atunci ABCA'B'C'
(cazul ULU de congruen a triunghiurilor).
R3.4.3 n triunghiul ABC, cu AB =

2
AC i m(BAC)=60, ducem mediana
3

[CM], M(AB). Demonstrai c [CM][CB].


Ipotez: ABC

2
AB = AC
3
Concluzie:

m(BAC)=60
M(AB): [MA][MB]
[CM][CB]

Demonstraie. Se face o construcie auxiliar: se ia NAB, B(MN) i


[BM][BN],

dar

1
1 2
1
MB = MA = AB = AC = AC ,
2
2 3
3

131

deci

1
1
AM = MB = BN = AC , de unde rezult c AN = 3AM = 3 AC , AN=AC.
3
3
Avem: [AN][AC] i m(BAC)=60, de unde rezult c ANC este echilateral.
Avem: [AM][BN], MACBNC (unghiuri ale triunghiului echilateral
ANC) i [AC][NC], atunci MACBNC (conform cazului LUL de congruen a
triunghiurilor), de unde rezult c [CM][CB].
R3.4.4 Un triunghi este isoscel dac i numai dac are dou nlimi
congruente.
I. nlimile corespunztoare laturilor congruente ale unui triunghi isoscel sunt
congruente.
Ipotez: ABC: [AB][AC]
BEAC, EAC
CFAB, FAB
Concluzie:
[BE][CF]
Distingem dou cazuri, dup cum m(BAC)<90 sau m(BAC)>90:
A

F
A

F
B

E
C

Demonstraie. Artm c triunghiurile FBC i ECB sunt congruente:


m(CFB)=m(BEC)=90, [BC] latur comun, FBCECB (la baza triunghiului
isoscel), atunci FBCECB (cazul I.U.), de unde rezult c [CF][BE].
II. Dac un triunghi are dou nlimi congruente, atunci el este triunghi
isoscel.
Ipotez: ABC
BEAC, EAC
CFAB, FAB
[BE][CF]
Concluzie:
[AB][AC]
Demonstraie. Artm c triunghiurile FBC i ECB sunt congruente:
m(CFB)=m(BEC)=90. [BC] latur comun, [CF][BE], atunci FBCECB
(cazul I.C.), de unde rezult c FBCECB, deci ABC este isoscel, [AB][AC],
avnd dou unghiuri congruente.

132

Remarc. Demonstraiile celor dou probleme de mai sus se pot face


considernd triunghiurile AEB i AFC.
R3.4.5 Un triunghi este isoscel dac i numai dac are dou mediane
congruente.
I. Medianele corespunztoare laturilor congruente ale unui triunghi isoscel sunt
congruente.
Ipotez: ABC: [AB][AC]
M(AC), [MA][MC]
N(AB), [NA][NB]
Concluzie:
[BM][CN]
A

M
G

Demonstraie. Avem: AM = MC =

1
1
AC , AN = NB = AB , dar AB=AC,
2
2

deci [AM][MC][AN][NB].
Vom demonstra c triunghiurile NBC i MCB sunt congruente: [BC] latur
comun, [NB][MC], NBCMCB (la baza triunghiurilui isoscel ABC), atunci
NBCMCB, de unde rezult c [BM][CN].
Remarc. Demonstraia se putea face considernd triunghiurile AMB i ANC.
Observaie. Notm BMCN={G}, centrul de greutate al triunghiului ABC. n
plus se poate demonstra c GBC este isoscel (din NBCMCB rezult c
NCBMBC) i c AGBC ( NG = NC - GC = MB - GB = MG ; [AN][AM] i
[AG] latur comun, atunci ANGAMG (LLL), de unde rezult c NAGMAG,
dar ABC este isoscel de baz [BC], deci bisectoarea unghiului BAC este i nlime,
atunci AGBC).
II. Dac un triunghi are dou mediane congruente, atunci el este isoscel.
Ipotez: ABC
M(AC). [MA][MC]
N(AB), [NA][NB]
[BM][CN]
Concluzie:
[AB][AC]
133

Demonstraie. Pentru a putea arta c triunghiurile NBC i MCB sunt


congruente, trebuie s demonstrm c NCBMBC.
Construcia auxiliar: fie MP||NC, PBC. Avem NCBMPB, fiind
unghiuri corespondente i PMCNCB, fiind unghiuri alterne interne.
Conform definiiei, [MN] este linie mijlocie n ABC, deci MM||BC, de unde
rezult c NMCMCP, unghiuri alterne interne.
Avem: NMCMCP, NCMPMC, [MC] latur comun, atunci
NMCPCM (ULU), de unde rezult c [MP][NC], dar [MB][NC], deci
[MP][MB]. Triunghiul MBP este isoscel, [MP][MB], de unde rezult c
MPBMBC, dar MPBNCB, deci MBCNCB. Avem: MBCNCB,
[BC] latur comun, [MB][NC] (din ipotez), atunci NBCMCB (cazul LUL), de
unde rezult c NBCMCB, deci ABC este isoscel, [AB][AC] deoarece are
dou unghiuri congruente.
R3.4.6 Un triunghi este isoscel dac i numai dac are dou bisectoare
congruente.
I. Bisectoarele unghiurilor congruente ale unui triunghi isoscel sunt
congruente.
Ipotez: ABC: [AB][AC]
ABDDBC, D(AC)
ACEECB, E(AB)
Concluzie:
[BD][CE]
Demonstraie. Unghiurile de la baza unui triunghi isoscel sunt congruente i
[BD, [CE sunt bisectoarele lor, deci ABDDBCACEECB. Avem:
EBCDBC (la baza triunghiului isoscel), ECBDBC, [BC] latur comun,
atunci EBCDCB (ULU), de unde rezult c [BD][CE].
134

Remarc. Demonstraia se putea face


considernd triunghiurile ADB i AEC.
Observaie. Notm BDCE={I}, centrul
cercului nscris n triunghiul ABC. Avem
IBCICB, de unde rezult c triunghiul IBC este
isoscel. Se mai poate demonstra c AIBC
( EI = EC - IC = DB - IB = DI ;
AE = AB - EB = AC - DC = AD ;
[AI]
latur
E
comun, atunci AIEAID (LLL), de unde rezult
c EAIDAI, dar ABC este isoscel de baz [BC],
bisectoarea unghiului BAC este i nlime, deci
AIBC).
II. Dac un triunghi are dou bisectoare
congruente, atunci el este isoscel.
B
Ipotez: ABC
ABDDBC, D(AC)
ACEECB, E(AB)
[BD][CE]
Concluzie:
[AB][AC]

D
I
C

A
F

Demonstraie. Folosim metoda reducerii la absurd. Presupunem c


Ar nsemna, de exemplu, c AC<AB. Ar rezulta c
m(ABC)<m(ACB), deci m(DBC)<m(ECB) (1) i s-ar obine
DC<EB (2).
Vom considera paralele prin D la BE i prin E la BD i vom nota cu F
intersecia acestor paralele.

[AB] / [AC] .

135

Avem: BEDEDF (alterne interne), BDEDEF (alterne interne) i


[DE] latur comun, atunci EDBDEF (cazul ULU), de unde rezult [BE][DF].
Inegalitatea (2) s-ar scrie DC<DF, atunci n CDF, am avea
m(DFC)<m(DCF) (3).
Din EDBDEF rezult c EBDEFD, dar i EBDDBC, atunci
EFDDBC. innd seama de (1) am putea scrie: m(EFD)<m(ECB) sau
m(EFD)<m(ECD) (4).
Adunnd membru cu membru inegalitile (3) i (4) am obine:
m(DFC)+m(EFD)<m(DCF)+m(ECD) sau m(EFC)<m(ECF). Ar nsemna c
n triunghiul ECF: EC<EF (unghiului cu msur mai mic i se opune o latur mai
mic). Dar cum [EC][BD] (din ipotez), ar nsemna c BD<EF, ceea ce este absurd
deoarece [BD][EF] (din EDBDEF).
Am ajuns la un rezultat absurd pentru c am pornit de la o presupunere fals,
anume aceea c [AB] / [AC] . Deci, triunghiul ABC este isoscel, [AB][AC].
R3.4.7 n triunghiul dreptunghic ABC, m(A)=90 i m(B)=15; se
construiesc nlimea [AD], bisectoarea [AE] i mediana [AO]. Dac DE=a i CD=b, s
se calculeze perimetrul triunghiului AOE.
Ipotez: ABC: m(A)=90
m(B)=15
ADBC, D(BC)
CAEEAB, E(BC)
O(BC), [OB][OC]
Concluzie:
PAOE

Demonstraie. Mediana corespunztoare ipotenuzei are lungimea egal cu


jumtate din lungimea ipotenuzei, deci AO=OB=OC, de unde rezult c AOB este
isoscel i m(OAB)=m(OBA)=15, iar m(AOC)=30, fiind unghi exterior
triunghiului OAB.
Avem [AE bisectoarea unghiului BAC, deci
m(EAB)=m(EAC)=45, m(OAB)=15,
atunci
m(EAO)=45-15=30, dar m(EOA)=30,
deci triunghiul AOE este isoscel, [AE][EO].
136

Avem triunghiul dreptunghic ADC (m(ADC)=90) i m(C)=75, deci


m(CAD)=15, iar m(DAE)=m(CAE)-m(CAD)=45-15=30. n triunghiul
dreptunghic ADE (m(ADE)=90), m(DAE)=30, iar DE=a, rezult c AE=2DE=2a
(cateta opus unghiului de 30 este jumtate din ipotenuz), dar [AE][EO], atunci
AE=EO=2a.
tim c CD=b, DE=a, EO=2a, OC=CD+DE+EO, deci OC=b+3a, dar OC=OA,
rezult OA=3a+b. Putem calcula perimetrul triunghiului AOE:
PAOE=AO+AE+EO=3a+b+2a+2a=7a+b.
R3.4.8 n triunghiul ABC, [AB][AC], m(BAC)=20. Fie E(AB) i
D(AC), astfel nct m(ACE)=30. m(ABD)=20. Aflaia msura unghiului
AED.
Ipotez: ABC: [AB][AC]
m(BAC)=20
E(AB)
D(AC)
m(ACE)=30
m(ABD)=20
Concluzie:
m(AED)

Demonstraie. n triunghiul isoscel ABC, [AB][AC], m(BAC)=20, atunci

m(ABC) = m(ACB) =

180 20
= 80 ,
2

m(ACE)=30,

rezult

m(ECB)=m(ACB)-m(ACE)=80-30=50.
n triunghiul BEC calculm
m(BEC)=180-m(EBC)+m(ECB))=180-(80+50)=50,
deci m(BEC)=m(BCE)=50, atunci BEC este isoscel, [BC][BE] (1).
Construim F(AC) astfel nct m(FBC)=20. dar m(BCF)=80, rezult c
m(BFC)=180-(m(BCF)+m(FBC))=180-100=80, atunci BFC este isoscel,
[BC][BF] (2).
137

Din (1) i (2) rezult c [BF][BE] i cum m(EBF)=60, atunci BEF este
echilateral, [EF][BF] (3).
Avem:
m(DBF)=m(EBC)-(m(EBD)+m(FBC))=80-220=40
i m(BDC)=180-(m(DBC)+m(DCB))=180-(60+80)=40, deci FDB este
isoscel, [FD][FB] (4).
Din (3) i (4) rezult c [EF][FD], deci FED este isoscel, dar m(EFD)=180(m(EFB)+m(BFC))=180-(60+80)=40, atunci

180 - 40
= 70 , i cum m(FEB)=60, rezult c
2
m(AED) = 180 (m(FEB) + m(FED)) =
= 180 (60 + 70) = 180 130 = 50 .

m(FED) = m(FDE) =

3.5. Patrulatere

Importana se reflect n locul pe care-l ocup n programa colar, patrulaterele fiind studiate att n clasa a VI-a ct i n clasa a VII-a. Poate i multitudinea
aplicaiilor practice i confer temei un mare avantaj.
Vom reaminti cteva definiii i teoreme de care avem nevoie n abordarea
temei.
Definiia 3.5.1. Un patrulater se numete convex dac, oricare ar fi o latur a
sa, celelalte dou vrfuri, nesituate pe latura considerat, se afl de aceeai parte a
dreptei n care este inclus latura respectiv.
Teorema 3.5.1. Suma msurilor unghiurilor unui patrulater convex este de
360.
Paralelogramul
Definiia 3.5.2. Patrulaterul convex cu laturile opuse paralele se numete
paralelogram.
Proprieti
Teorema 3.5.2. n orice paralelogram laturile opuse sunt congruente i reciprocele:
- orice patrulater convex n care laturile opuse sunt congruente este paralelogram
- orice patrulater convex n care dou laturi opuse sunt congruente i paralele
este paralelogram.
Teorema 3.5.3. n orice paralelogram unghiurile opuse sunt congruente i
reciproc, orice patrulater convex n care unghiurile opuse sunt congruente este paralelogram.
Teorema 3.5.4. n orice paralelogram diagonalele se intersecteaz n pri
congruente i reciproc, orice patrulater convex n care diagonalele se intersecteaz n
pri congruente este paralelogram.

138

Construcia paralelogramelor
1) Trasm dou drepte paralele pe care le intersectm cu alte dou drepte
paralele. Punctele de intersecie vor fi vrfurile paralelogramului.
2) Desenm dou segmente paralele i congruente. Extremitile lor vor fi
vrfurile paralelogramului.
3) Intersectm dou segmente necongruente care au acelai mijloc.
Extremitile acestor segmente vor fi vrfurile paralelogramului.
Dreptunghiul
Definiia 3.5.3. Paralelogramil cu un unghi drept se numete dreptunghi.
Proprieti
Teorema 3.5.5. n orice dreptunghi toate unghiurile sunt congruente i deci
drepte i reciproc orice patrulater convex n care toate unghiurile sunt congruente i
deci drepte este dreptunghi.
Teorema 3.5.6. n orice dreptunghi diagonalele sunt congruente i reciproc,
orice paralelogram cu diagonalele congruente este dreptunghi.
Construcia dreptunghiului
1) Desenm un triunghi dreptunghic i prin vrfurile unghiurilor ascuite
ducem paralele la catete care se vor intersecta ntr-un punct ce va fi al patrulea vrf al
dreptunghiului (vrfurile triunghiului dreptunghic vor fi celelalte trei vrfuri).
2) Desenm dou segmente congruente care s aib acelai mijloc.
Extremitile lor vor fi vrfurile dreptunghiului.
Rombul
Definiia 3.5.4. Paralelogramul care are dou laturi consecutive congruente se
numete romb.
Proprieti
Teorema 3.5.7. ntr-un romb toate laturile sunt congruente i reciproc orice
patrulater cu toate laturile congruente este romb.
Teorema 3.5.8. ntr-un romb diagonalele sunt perpendiculare i reciproc,
orice paralelogram cu diagonalele perpendiculare este romb.
Teorema 3.5.9. ntr-un romb diagonalele sunt bisectoarele unghiurilor i reciproc, orice paralelogram n care o diagonal este i bisectoarea unui unghi este romb.
Construcia rombului
1) Desenm dou drepte perpendiculare. Fixm pe fiecare dreapt dou puncte
simetrice fa de a doua dreapt astfel ca segmentele formate pe prima dreapt s nu fie
congruente cu cele de pe a doua dreapt. Cele patru puncte sunt vrfurile rombului.
2) Desenm un triunghi isoscel (vrfurile lui vor fi trei dintre vrfurile
rombului). Construim simetricul vrfului triunghiului fa de baz. Acesta va fi al
patrulea vrf al rombului.

139

Ptratul
Definiia
numete ptrat.

3.5.5. Dreptunghiul cu dou laturi consecutive congruente se

Construcia ptratului
1) Desenm dou drepte perpendiculare, iar cu centrul n punctul lor de
intersecie trasm un cerc. Interseciile cercului cu cele dou drepte perpendiculare vor
fi cele patru vrfuri ale ptratului.
2) Desenm un unghi drept i lum pe laturile lui dou segmente congruente,
ambele avnd unul din capete n vrful unghiului. Prin capetele segmentelor diferite de
vrful unghiului ducem paralele la laturile unghiului care se vor intersecta ntr-un
punct (al patrulea vrf al ptratului).
3) Desenm un triunghi dreptunghic isoscel i apoi construim simetricul
unghiului drept fa de ipotenuz.
Trapezul
Definiia 3.5.6. Patrulaterul care are dou laturi paralele i celelalte dou
neparalele.
Trapezul este isoscel cnd laturile neparalele sunt congruente.
Teorema 3.5.10. Unghiurile alturate bazelor unui trapez isoscel sunt congruente i reciproc, dac unghiurile alturate bazelor unui trapez sunt congruente, trapezul
este isoscel.
Teorema 3.5.11. Diagonalele unui trapez isoscel sunt congruente i reciproc
dac diagonalele unui trapez sunt congruente trapezul este isoscel.

Vom prezenta n continuare probleme rezolvate n care sunt evideniate


proprietile patrulaterelor.
Probleme rezolvate

R3.5.1 Pe laturile unui paralelogram, ca baze, se construiesc n afar


triunghiuri echilaterale. S se demonstreze c vrfurile acestor triunghiuri, diferite de
vrfurile paralelogramului sunt vrfurile unui paralelogram.
Demonstraie. Fie ABCD paralelogramul i triunghiurile ABF, ADE, DQC,
CBG, echilaterale construite pe laturile paraleloagramului.

140

Avem: FGBQED (LUL), deoarece


(BF)(QD)
FBGQDE (240-m(B) i 240-m(D), DB)
(BG)(DE)
Din congruena triunghiurilor de mai sus obinem c
(1)
(FG)(EQ)
i triunghiurile AEF i CGQ sunt congruente (LUL), deoarece
(AE)(CG)
(EAF)(GCQ)
(AF)(CQ)
Din congruena triunghiurilor AEF i CGQ rezult
(2)
(EF)(GQ)
Din relaiile (1) i (2) rezult c patrulaterul EQGF are laturile opuse
congruente i deci este paralelogram.
R3.5.2 n patrulaterul convex ABCD, L este mijlocul laturii (AB) i P este
mijlocul laturii (CD). S se demonstreze c mijloacele segmentele AP, CL, BP, DL
sunt vrfurile unui paralelogram.
Demonstraie. Fie T mijlocul lui (AP), R mijlocul lui (DL), E mijlocul lui
(LC), iar Q mijlocul lui (PB) i O mijlocul lui (PL).

141

P
D

E
T

n triunghiul APL, [TO] este linie mijlocie. Deci TO||AL i TO =


triunghiul LPB, [OQ] este linie mijlocie. deci OQ||LB i OQ =

AL
. n
2

LB
. Prin punctul O
2

trece o singur paralel la AB, deci punctele T, O, Q sunt coliniare. Fiindc (AL)(LB)
i TO =

AL
LB
iar OQ =
, obinem c (TO)(QO), adic O este mijlocul
2
2

segmentului (TQ) (1).


n triunghiul DLP, (RO) este linie mijlocie, deci
RO||DP i RO =

DP
.
2

n triunghiul PLC, (OE) este linie mijlocie, deci


OE||PC, OE =

PC
.
2

Prin O trece o singur paralel la DC, rezult c punctele R, O, E sunt


coliniare. Din (DP)(PC) i RO =

DP
PC
, EO =
rezult
2
2

(RO)(OE)
(2)
Din (1) i (2) rezult c diagonalele patrulaterului TRQE se taie n pri
congruente, deci el este paralelogram.
R3.5.3 Se consider patrulaterul convex ABCD n care m(A)+m(C)=180.
n exteriorul patrulaterului se construiesc dreptunghiurile ABPF, BCRH, CDTQ,
DAML astfel ca (BP)(CD), (AM)(BC), (DT)(AB), (CR)(AD). Demonstrai c
centrele acestor dreptunghiuri sunt vrfurile unui nou dreptunghi.

142

T
Q
L

O3

D
C
O4
M

O2
A

B
O1
F

Fie O1, O2, O3, O4 centrele dreptunghiurilor ABPF, CBHR, CQTD, DLMA.
Trebuie s artm c O1O2O3O4 este dreptunghi, adic paralelogram cu un unghi drept.
[O1O2] este linie mijlocie n triunghiul FBR. Deci O1O 2 =

FR
.
2

n triunghiul MQD, [O4O3] este linie mijlocie, deci O 4 O 3 =

MQ
.
2

Pentru a demonstra c [O1O2][O4O3] este suficient s artm c [FR][MQ].


Suma msurilor unghiurilor formate n jurul unui punct este 360.
n punctul B, m(ABP)+m(ABC)+m(CBH)+m(HBP)=360, dar
m(ABP)+m(CBH)=180, deci
(1)
m(ABC)+m(HBP)=180
Suma msurilor unghiurilor unui patrulater convex este 360. Deci
m(ABC)+m(BCD)+m(CDA)+m(DAB)=360.
Din ipotez m(DAB)+m(BCD)=180, i atunci obinem c
(2)
m(ABC)+m(CDA)=180
Comparnd relaiile (1) i (2) obinem:
(3)
m(HBP)=m(CDA)
Din triunghiurile dreptunghice congruente FPB i QCD ce au (FP)(CQ) i
(BP)(CD) obinem c
(4)
(FBP)(QDC), (FB)(DQ)
i triunghiurile dreptunghice BRH i DMA sunt congruente, avnd
(BH)(AD) i (RH)(AM) i atunci
(4*)
(RBH)(MDA), (BR)(MD)
Atunci
143

m(FBR)=360-[m(FBP)+m(PBH)+m(HBR)]

(5)
iar
(6)

m(MDQ)=360-[m(QDC)+m(CDA)+m(MDA)]
Din (5) i (6) obinem c
FBRMDQ
(7)
deoarece (FBP)(QDC), (PBH)(CDA) i (HBR)(MDA).
Din relaiile (4), (4*), (7) obinem c:
FBRQDM (LUL)
(8)
Din congruena triunghiurilor de la (8) obinem c (FR)(QM), de unde

FR QM
=
, adic
2
2
O1O2=O4O3
(9)
Analog se arat c i (O1O4)(O3O2).
Atunci patrulaterul O1O2O3O4, avnd laturile opuse congruente este
paralelogram. Pentru a fi dreptunghi mai trebuie s artm c are i un unghi drept.
Avem m(O2O1O4)=m(AO1B)-[m(AO1O4)+m(BO1O2)] i
m(O4O3O2)=m(CO3D)+m(CO3O2)+m(DO3O4).
Triunghiurile AO1O4 i CO3O2 sunt congruente deoarece

AP CT
=
= CO 3
2
2
AL CH
AO 4 =
=
= CO 2
2
2
O1 O 4 = O 2 O 3
AO1 =

Din congruena celor dou triunghiuri rezult c AO1O4CO3O2.


Analog se arat c BO1O2DO3O4. Atunci
m(O4O1O2)+m(O2O3O4)=m(AO1B)+m(CO3D)
Triunghiurile O1BP i DO3C, avnd toate laturile congruente sunt congruente,
de unde rezult c m(DO3C)m(BO1P). Obinem
m(O4O1O2)+m(O2O3O4)=m(AO1B)+m(BO1P)=180
Deci m(O4O1O2)+m(O2O3O4)=180. Dar ntr-un paralelogram unghiurile
opuse sunt congruente, deci O4O1O2O2O3O4 atunci fiind i suplementare rezult
c m(O4O1O2)=90.
Paralelogramul O4O1O2O3 avnd un unghi drept este dreptunghi.
R3.5.4 Se consider ptratul ABCD iar M i N sunt mijloacele laturilor AB,
respectiv AD. Dac BDCN={L}, BDCM={K} i BCNK={T} s se arate c
patrulaterul ALCK este romb.
Demonstraie. Fie O intersecia diagonalelor ptratului ABCD.
n triunghiul ABC, CM i BO sunt mediane, rezult c punctul K este centrul
de greutate al triunghiului. Deci

1
OK = OB
3
144

(1)

n triunghiul ADC, CN i DO sunt mediane, rezult c punctul L este centrul


de greutate al triunghiului. Deci

1
OL = DO
3

(2)

Dar (OB)(DO) i atunci din (1)


M
i (2) obinem c OK=OL, adic O este
A
B
mijlocul
segmentului
(LK).
Dar
(OA)(OC) i ACLK. Patrulaterul
ALCK are diagonalele perpendiculare i
K
T
se njumtesc, rezult c el este romb.
R3.5.5 Se consider triunghiul
isoscel ABC cu (AB)(AC) i AB=2BC.
O
Se duce nlimea AP, mediana BM, i N
bisectoarea CL. Pe latura AC se ia un
punct D astfel ca (BD)(BC).
L
S
se
Fie
{N}=BDCL.
demonstreze c patrulaterul BLMN este
romb.
Demonstraie. Fie {O}=LNBM.
D
C
n triunghiul isoscel BCM cu (BC)(CM),
CL este bisectoare, deci i median i nlime. Rezult c diagonalele patrulaterului
LBNM sunt perpendiculare.

M
L

O
N

Punctele N i L fiind situate pe mediatoarea segmentului [BM] avem


relaiile:(NB)(NM) i (LM)(LB). Triunghiul ABC fiind isoscel cu BC, iar
triunghiul CBD isoscel cu BCDCDB, rezult c CBDCAB.
n triunghiul isoscel BCM avem:
(1)
m(CMB)m(CBM)
145

Unghiul CMB este exterior BMA, atunci


m(CMB)=m(MAB)+m(MBA)

(2)

i
(3)
m(CBM)=m(CAB)+m(DBM)
Din (1), (2) i (3) rezult c m(MBA)=m(DBM), adic (BM este
bisectoarea NBL.
n triunghiul BLN, (BO fiind bisectoare i nlime rezult c el este isoscel cu
(BL)(BN). Obinem c patrulaterul BNML are toate laturile congruente i deci el este
paralelogram n care diagonalele sunt perpendiculare, deci el este romb.
R3.5.6 Pe laturile unui ptrat ABCD se construiesc, n exterior triunghiurile
echilaterale ABE, BCT, CDR, DAP, care au respectiv centrele de greutate G1,G2,G3,G4.
Demonstrai c patrulaterul G1G2G3G4 este ptrat.
Demonstraie.
E

G1
A

G2

G4

C
G3

Triunghiurile echilaterale ABE, BCT, CDR, DAP sunt congruente. Atunci


segmentele AG1, BG1, BG2, CG2, CG3, DG3, DG4, AG4 sunt congruente avnd fiecare
lungimea ca fiind

2
din nlimea unui triunghi echilateral (din cele considerate).
3

Unghiurile G4AG1, G1BG2, G3CG2, G4DG3 sunt congruente avnd msura


(30+90+30) 150. (Am inut seama c mediana n triunghiul echilateral este i
bisectoare.) Atunci triunghiurile isoscele G4AG1, G1BG2, G2CG3, G3DG4 sunt
congruente (LUL). Din congruena celor patru triunghiuri de mai sus obinem c
(G1G2)(G2G3)(G3G4)(G4G1), adic patrulaterul G1G2G3G4 este romb.

146

n triunghiul isoscel G1AB avem m(G1AB)=m(G1BA)=30. Din triunghiul


isoscel AG1G4 cu m(G1AG4)=150 obinem m(AG1G4)=15.
Analog din triunghiul isoscel G1GB2 obinem m(BG1G2)=15. Atunci
m(G4G1G2)=m(AG1B)-[m(AG1G4)+m(BG1G2)]=
=120-(15+15)=90.
Am obinut c rombul G1G2G3G4 are un unghi drept, deci este ptrat.
Prin vrfurile unui ptrat ABCD se duc dreptele AE, BF, CT, DL cu E(BC),
F(CD), T(AD), L(AB), astfel nct
m(BAE)=m(DCT)=m(ADL)=m(CBF)=30.
Dac {M}=AEBF, {P}=BFCT, {R}=CTDL, {Q}=DLAE, s se
demonstreze c MPRQ este ptrat.
Demonstraie. Avem m(DAE)=60, m(DTC)=60 i au poziia de unghiuri
corespondente, rezult c
AE||TC
(1)
i unghiurile FBA i DLA au msura 60 i deci
LD||BF
(2)
L

Q
M

R
P
F

Din relaiile (1) i (2) rezult c QMPR este paralelogram avnd laturile opuse
paralele.
Triunghiurile dreptunghice ADL, BFC, CTD, ABE sunt congruente (C.U.).
Atunci obinem c
(3)
(AL)(FC)(TD)(BE)
Atunci i
(4)
(BL)(DF)(AT)(CE)
Din relaiile (3) i (4) rezult c (QM)(MP)(PR)(QR) (paralele cuprinse
ntre paralele). Atunci paralelogramul QMPR este romb.
n triunghiul FPC, m(FPC)=180-(30+60)=90.
Deci m(RPM)=90. Rombul QMPR avnd un unghi drept este ptrat.
R3.5.7 Se consider triunghiul isoscel ABC cu m(A)=90. Pe laturile (AB)
i (AC) construim spre exterior triunghiurile echilaterale ABD respectiv ACE. S se
demonstreze c patrulaterul BCED este trapez isoscel.

147

Demonstraie. Din ipotez rezult c (AD)(AE), deci ADE este isoscel cu


m(DAE)=360-(90+60+60)=150.
Atunci m(EDA)=m(DEA) (15).

A
E

Avem m(EDB)+m(CBD)=(15+60)+(45+60)=180, adic unghiurile


EDB i CBD sunt interne de aceeai parte a secantei suplementare i deci DE||BC,
adic BCED este trapez. Dar (BD)(CE) i atunci trapezul BCED este isoscel.
Pe baza mic [CD] a trapezului isoscel ABCD se construiete ptratul CDMN.
Demonstrai c patrulaterul ABMN este trapez isoscel.
Demonstraie. a) Ptratul este construit spre interior.
D
C

M
A

Din MN||DC i AB||DC rezult c MN||AB, deci ABNM este trapez. Trapezul
ABCD fiind isoscel are unghiurile alturate unei baze congruente. Deci
ADCBCD, de unde rezult c ADMBCN. Atunci ADMBCN (LUL)
avnd (AD)(BC) (ABCD trapez isoscel), ADMBCN, (DM)(CN) ca laturi ale
ptratului DCNM. Din congruena celor dou triunghiuri rezult c (AM)(BN), adic
trapezul ABNM este isoscel.
b) Ptratul este construit spre exterior.
N

148

Trapezul ABCD fiind isoscel are unghiurile alturate unei baze congruente.
Deci
ADCBCD
(1)
Atunci m(MDA)=360-[90+m(ADC)] sau
(2)
m(MDA)=270-m(ADC)
Analog
(3)
m(NCB)=270-m(BCD)
Din relaiile (1), (2) i (3) obinem:
MDANCB
(4)
Atunci MDANCB (LUL), avnd (MD)(MC), laturi ale ptratului MNCD,
MDANCB din relaia (4), (AD)(CB) din trapezul isoscel ABCD.
Din congruena celor dou triunghiuri obinem c
(5)
(MA)(NB)
Fiindc DC||MN, DC||AB obinem
MN||AB
(6)
Din relaiile (5) i (6) rezult c trapezul ABNM este isoscel.
3.6. Concurena liniilor importante n triunghi

Pentru demonstrarea concurenei liniilor importante din triunghi folosim


proprietile patrulaterelor, a liniei mijlocii n triunghi i proprietile de loc geometric
ale unor linii importante din triunghi.
Teorema 3.6.1. n orice triunghi medianele sunt concurente.
Demonstraie. Fie A',B',C' mijloacele laturilor [BC], [CA], [AB] ale
triunghiului ABC, iar G punctul de concuren a medianelor [AA'] i [CC'].
Considerm punctele T i E astfel nct T este mijlocul lui [AG] i E este mijlocul lui
[GC].
A

T
B'

C'
G
L

A'

Atunci [TE] este linie mijlocie n triunghiul ACG. Avem


ET||AC, ET =

1
AC
2

n triunghiul BCA, [A'C'] este linie mijlocie, rezult c


149

(1)

A'C'||AC i A' C' =

1
AC
2

(2)

Din relaiile (1) i (2) obinem c patrulaterul C'A'ET este paralelogram avnd
dou laturi opuse paralele i congruente. Cum G este punctul de intersecie al
diagonalelor paralelogramului C'A'ET avem relaiile:

1
1
GA' = GT = AA' i GC' = GE = CC'
3
3

(3)

Fie L mijlocul lui [BG]. Atunci [LT] este linie mijlocie n triunghiul GAB i
putem scrie relaiile:
TL||AB, TL =

AB
2

(4)

n triunghiul CAB, [A'B'] este linie mijlocie, atunci


A'B'||AB, A' B' =

1
AB
2

(5)

Din (4) i (5) rezult c patrulaterul A'B'TL este paralelogram i

1
GB' = GL = BB'
3

(6)

Din relaiile (3) i (6) rezult c {G}=AA'BB'CC' i G se gsete pe fiecare


median la

1
2
de baz i la
de vrf.
3
3

Teorema 3.6.2. n orice triunghi bisectoarele interioare sunt concurente.


Demonstraie. Fie [AA1 i [BB1 bisectoarele unghiurilor A i B, iar I
punctul lor de intersecie.
A

Q
B1

P
C1
I

T A1

Bisectoarele [AA1 i [BB1 nu pot fi paralele pentru c ar nsemna c ABB1 i


BAA1 ar fi interne de aceeai pare a secantei AB, i suma msurilor lor ar fi 180 ca
i suma msurilor unghiurilor triunghiului ABC, ceea ce este imposibil. Deci [AA1 i
[BB1 sunt concurente n I. Punctele de pe bisectoare fiind egal deprtate de laturi avem
relaiile:
(IP)(IT), (IP)(IQ), cu IPAB, ITBC, IQAC i
P(AB), Q(AC), T(BC).

150

Din relaiile de mai sus rezult c (IT)(IQ), deci punctul I se gsete i pe


bisectoarea ACB.
Teorema 3.6.3. Mediatoarele laturilor unui triunghi sunt concurente.
Demonstraie. Fie punctele Q i P mijloacele laturilor [AB] i [BC] ale
triunghiului ABC, iar O punctul de intersecie al mediatoarelor laturilor [AB] i [BC].
Aceste dou mediatoare sunt concurente, cci dac ar fi paralele, punctele A, B, C ar fi
coliniare, ceea ce este imposibil fiind vrfurile triunghiului ABC.
A
Q

M
O

Punctele de pe mediatoarea unui segment fiind egal deprtate de extremitile


segmentului; avem relaiile:
(OA)(OB) (OQ fiind mediatoarea laturii AB)
(OB)(OC) (OP fiind mediatoarea laturii BC)
Din relaiile de mai sus obinem (OA)(OC), adic punctul O este situat pe
mediatoarea laturii (AC).
Teorema 3.6.4. nlimile unui triunghi sunt concurente.
Demonstraie. Considerm triunghiul ABC cu nlimile AA', BB', CC'
(AA'BC, BB'AC, CC'AB).
C''

B''

A
B'

C'
H
B

A'

A''

Prin vrfurile triunghiului ABC ducem paralele la laturile opuse care se


intersecteaz dou cte dou n punctele A'', B'', C''. Patrulaterele ABCB'' i C''BCA,
avnd laturile opuse paralele sunt paralelograme, rezult c laturile opuse sunt
congruente, deci (AB'')(BC) i (AC'')(BC), de unde (AB'')(AC''), de unde rezult c
A este mijlocul lui [C''B'']. Deci AA' este mediatoarea lui [C''B''] (1).
Din paralelogramele AB''CB i ACA''B rezult c (AB)(B''C) i (AB)(CA''),
de unde (B''C)(A''C), adic C este mijlocul lui [A''B'']. Atunci CC' este mediatoarea
laturii [A''B''] (2).
Din paralelogramele ACA''B i ACBC'' rezult c (AC)(BA'') i (AC)(C''B),
de unde (C''B)(BA''), adic B este mijlocul lui [C''A'']. Atunci B''B este mediatoarea
laturii [C''A''] (3).
151

Din (1), (2) i (3) rezult c nlimile triunghiului ABC sunt i mediatoarele
laturilor triunghiului A1B1C1. Cum concurena mediatoarelor a fost demonstrat,
rezult c i nlimile sunt concurente.
3.7. Probleme de coliniaritate

Problemele a cror concluzie solicit demonstrarea apartenenei unor puncte la


o aceeai dreapt le vom numi probleme de coliniaritate.
n continuare enumerm cteva dintre procedeele cele mai des ntlnite pentru
soluionarea problemelor de coliniaritate la nivelul clasei a VI-a:
a) Demonstrarea coliniaritii cu ajutorul unghiului alungit (unghiuri
adiacente suplementare)
Dac punctele A i B sunt situate de o parte i de alta a dreptei CD i
m(ACD)+m(DCB)=180, atunci punctele A, C, B sunt coliniare.
b) Demonstrarea coliniaritii folosind reciproca teoremei unghiurilor
opuse la vrf
Dac punctul B este situat pe dreapta EF, iar punctele A i C sunt situate de o
parte i de alta a dreptei EF i ABFCBE, atunci punctele A, B, C sunt coliniare.
c) Demonstrarea coliniaritii prin identificarea unei drepte ce conine
punctele respective
d) Demonstrarea coliniaritii folosind postulatul lui Euclid
Dac dreptele AB i BC sunt paralele cu o dreapt d, atunci n baza
postulatului lui Euclid, punctele A, B, C sunt coliniare.
e) Demonstrarea coliniaritii folosind axioma de construcie a unghiului
Dac B i C sunt n acelai semiplan determinat de dreapta AA' i
A'ABA'AC, atunci A, B, C sunt coliniare.
f) Demonstrarea coliniaritii punctelor A, B, C demonstrnd c
AB+BC=AC
Vom exemplifica procedeele prin probleme rezolvate n continuare.
a) Demonstrarea coliniaritii cu ajutorul unghiului alungit

R3.7.1 Pe laturile consecutive AB i BC ale ptratului ABCD se construiesc


triunghiurile echilaterale ABE i BCF, primul interior i al doilea exterior ptratului. S
se arate c punctele D, E, F sunt coliniare.
Soluie. Fiindc triunghiul ABE este echilateral, m(BAE)=60 i atunci
m(DAE)=90-60=30. Triunghiul ADE este isoscel cu (AD)(AE) i atunci
m(ADE)=m(AED)=(180-30):2=75. Triunghiul EBF este dreptunghic deoarece
m(EBC)+m(CBF)=30+60=90. Fiindc (EB)(BF), triunghiul EBF este
Deci
dreptunghic
isoscel.
Atunci
m(BEF)=45.
m(DEA)+m(AEB)+m(BEF)=75+60+45=180, adic punctele D, E, F sunt
coliniare.
152

R3.7.2 S se demonstreze c ntr-un trapez mijloacele laturilor paralele i


intersecia diagonalelor sunt trei puncte coliniare.
Soluie. Fie O punctul de intersecie al diagonalelor AC i BD ale trapezului
ABCD, iar L i T mijloacele bazelor (AB) i (CD). Deoarece AB||DC rezult c
DTLBLT (alterne interne), iar TDBLBD (alterne interne) i atunci obinem c
DOTBOL
(1)
Mai avem c AODBOC (opuse la vrf) (2).
i ALTCTL (alterne interne), LACTCA, atunci obinem c
AOLCOT
(3)
Cu relaiile (1), (2), (3) n jurul punctului O avem:
2m(AOL)+2m(AOD)+2m(DOT)=360,
de unde
m(AOL)+m(AOD)+m(DOT)=180,
rezult c punctele T, O, L sunt coliniare.
A

R3.7.3 Pe ipotenuza (BC) a triunghiului dreptunghic ABC se consider un


punct arbitrar D. Fie K i P simetricele lui D fa de AB respectiv AC. S se arate c
punctele K, A, P sunt coliniare.
Soluie. Punctul A se gsete pe mediatoarea segmentului [DK], el va fi egal
deprtat de capetele segmentului. Deci (AK)(AD). n triunghiul ADK, nlimea AB
este i bisectoare, deci
KABBAD
(1)

153

Punctul A se gsete i pe mediatoarea [DP], rezult c (AD)(AP). n


triunghiul isoscel APD, nlimea AC este i bisectoare, deci
PACCAD
(2)
Din relaiile (1) i (2) obinem:
m(KAB)+m(BAC)+m(CAP)=2m(BAC)=180
deci punctele K, A, P sunt coliniare.
b) Demonstrarea coliniaritii folosind reciproca teoremei unghiurilor opuse la
vrf
R3.7.4 Se consider patrulaterul ABCD cu E mijlocul lui [AB] i R mijlocul
lui [CD]. Prin E se duc EF paralel la BC i EQ paralel la AD, iar prin vrfurile C i D
se duce cte o paralel la AB. Obinem paralelogramul BCFE i AEQD. S se arate c
vrfurile F i Q ale acestor paralelograme sunt coliniare cu R.
Demonstraie. BCFE este paralelogram deci
(1)
[CF][BE]
Din paralelogramul AEQD obinem
(2)
[EA][QD]
i
(3)
[BE][EA]
deoarece E este mijlocul lui AB.
Din (1), (2) i (3) rezult c
(4)
[CF][QD]
Punctul R este mijlocul lui [CD], deci
(5)
[CR][DR]
Mai avem c CF||BA||QD, deci CF||QD. atunci
FCRQDR
(6)
Din (4), (5), (6) obinem c triunghiurile CFR i DQR sunt congruente (LUL).
Din congruena celor dou triunghiuri obinem c CRFDRQ i cum F i Q sunt de
o parte i de alta a dreptei CD, deci punctele C, R, Q sunt coliniare.

154

C
B
R

c) Demonstrarea coliniaritii prin identificarea unei drepte ce conine punctele


respective
R3.7.5 n triunghiul ABC cu m(B)=2m(C), notm cu I intersecia
bisectoarelor [BB' i [AA', B'(AC), A'(BC). Perpendiculara din B pe AA'
intersecteaz perpendiculara din B' pe BC n D. S se demonstreze c punctele I, D, C
sunt coliniare.
Demonstraie. Din m(B)=2m(C) rezult m(A'BB')=m(C), deci
triunghiul B'BC este isoscel i atunci [B'D va fi bisectoarea BB'C. Unghiul BA'A
este exterior triunghoului AA'C i deci

A
m(BA' A) = m(C) + m

(1)

Unghiul BIA' este exterior triunghiului ABI. Deci

A
m(BIA') = m(IBA) + m(IAB) = m(C) + m

(2)

Din (1) i (2) rezult c triunghiul BIA' este isoscel. Atunci nlimea BD a
triunghiului IBA' este i bisectoare. Deci punctul D este punctul de intersecie a
bisectoarelor [BD i [B'D ale unghiurilor triunghiului B'BC. Fiindc [CI este
bisectoarea unghiului C ea conine i punctul D, deci punctele C, D, I sunt coliniare.
A

B'
I
D

A'

R3.7.6 Se consider triunghiul ascuitunghic ABC, iar AD bisectoarea


unghiului BAC, cu D(BC). Fie L i K simetricul lui D fa de AB, respectiv AC, iar
Q intersecia paralelei prin L la AC cu paralela prin K la AB. Artai c punctele A, D,
Q sunt coliniare.
Soluie. Fiindc LDAB i AB||QK, rezult c LDQK sau
155

LRQK
(1)
RKQ i KDAC, LQ||AC rezult c KDLQ sau
(2)
KPLQ
PLQ.
Din (1) i (2) rezult c punctul D este ortocentrul triunghiului LKQ, de unde
QDLK.
Fie {S}=ARAB i {T}=KPAC. Din congruena triunghiurilor dreptunghice
ADS i ADT obinem c (DS)(DT) sau 2DS=2DT, care se mai scrie DL=DK.
Deci triunghiul LDK este isoscel iar [DA este bisectoare (din ADSADT),
ea va fi i mediatoarea lui (LK). Deci DALK. Din unicitatea perpendicularei din D pe
(LK) rezult c punctele A, D, Q sunt coliniare.
A
K

L
S

D
B

C
R

P
Q

R3.7.7 Fie trapezul ABCD cu AD||BC. Bisectoarele interioare ale unghiurilor


A i B se intersecteaz n L, iar bisectoarele interioare din C i D se intersecteaz n Q.
Dac T este mijlocul diagonalei (AC). S se arate c punctele L, T, Q sunt coliniare.
Demonstraie. Fiindc DA||CB rezult c CBABAF (alterne interne).
dar

m(DAB)+m(BAF)=180

CBA
m(BAL) + m
= 90
2

m(BAL)+m(LBA)=90. Deci (ALB)=90, adic ALB este dreptunghic n L. Fie


R mijlocul lui (BA). Atunci LR este median n triunghiul dreptunghic BLA, deci

LR =

AB
. Obinem astfel c LRA este isoscel, de unde RLALARDAL, de
2

unde obinem c RL||AD. Fie F mijlocul lui [CD], atunci [QF] este median n
triunghiul dreptunghic CQD. Obinem QF =

CD
, adic triunghiul QFD este isoscel
2

cu FQDFDQQDA, de unde rezult c FQ||DA. [FR] este linie mijlocie n


trapez, deci TR||AD. Deci linia mijlocie a trapezului conine punctele Q, L i T (TR
este linie mijlocie n ABC, RT||BC).

156

F
Q

F
D

d) Demonstrarea coliniaritii folosind postulatul lui Euclid


R3.7.8 n triunghiul ABC, punctele B' i C' sunt mijloacele segmentelor [AC],
respectiv [AB]. Dac L este simetricul punctului B fa de B' i T este simetricul
punctului C fa de C', s se demonstreze c punctele T, A, L sunt coliniare.
Demonstraie. n patrulaterul ACBT, diagonalele [AB] i [TC] se taie n pri
congruente, rezult c ACBT este paralelogram. Deci
AT||BC
(1)
i diagonalele patrulaterului ABCL se mjumtesc i deci ABCL este
paralelogram. Atunci
AL||BC
(2)
Din (1) i (2) innd seama c prin A trece numai o singur paralel la BC
obinem c punctele T, A, L sunt coliniare.
A

C'

B'

R3.7.9 Se d un triunghi ABC. S se arate c mijloacele laturilor AB, BC i


proiecia vrfului B pe bisectoarea unghiului A sunt trei puncte coliniare.
Demonstraie. Fie T mijlocul lui [AB] i Q mijlocul lui [BC]. Fie BLAE,
L(AE). Atunci triunghiul ABL este dreptunghic, iar TL este median,
corespunztoare ipotenuzei [AB], deci TL =

AB
= AT . Obinem astfel c triunghiul
2

ATL este isoscel cu (TA)(TL), deci


TALTLA

(1)

Dar
CALLAT
Din (1) i (2) rezult c CALALT, deci
AC||TL
n triunghiul ABC, [TQ] este linie mijlocie i atunci

157

(2)
(3)

AC||TQ
(4)
Din (3) i (4), innd seama c prin T trece o
singur paralel la AC obinem c punctele T, L, Q
sunt coliniare.
Pe latura [BC] a triunghiului ABC se
T
consider un punct D astfel nct BC=3DC. Dac R
este mijlocul medianei CM, M(AB), s se arate c
L
punctele A, R, D sunt coliniare.
Soluie. Dac T este mijlocul lui BD, atunci
B
C
E Q
[MT] este linie mijlocie n triunghiul ABD i deci
AD||MT
(1)
n triunghiul CMT, [RD] este linie mijlocie, rezult c
RD||MT
(2)
Din (1) i (2) i faptul c prin punctul R trece numai o singur paralel la MT,
rezult c punctele A, R, D sunt coliniare.
A

3.8. Probleme de concuren

Vom numi problem de concuren o problem de geometrie a crei concluzie


cere demonstrarea faptului c trei sau mai multe drepte (cercuri) au ca intersecie
acelai punct.
Dintre procedeele mai des utilizate pentru demonstrarea concurenei (la nivelul
clasei a VI-a) amintim:
a) Demonstrm c punctul de intersecie a dou dintre drepte aparine i celei
de-a treia dreapt. (Trebuie demonstrat c punctul de intersecie a dou dintre drepte
exist, iar apartenena lui la cea de-a treia dreapt se demonstreaz de obicei artnd c
punctele acesteia sunt caracterizate de o anumit proprietate specific, pe care o are i
punctul obinut ca intersecie a celor dou drepte.)
b) Demonstrarea concurenei a trei drepte prin identificarea acestora cu trei
ceviene remarcabile concurente (bisectoare, nlimi, mediane, etc.) dintr-un triunghi
din configuraia problemei.
c) Demonstrarea concurenei prin coliniaritate.
Vom exemplifica procedeele enunate mai sus n rezolvarea unor probleme.
158

Probleme rezolvate

R3.8.1 Fie M un punct n interiorul triunghiului ABC. Dac D, E, T sunt


simetricele punctului M fa de mijloacele laturilor [BC], [CA], [AB], s se
demonstreze c dreptele AD, BE, CT sunt concurente.
Demonstraie. Fie G, H, K mijloacele laturilor [AB], [AC], [BC]. Diagonalele
patrulaterului AMBT se taie n pri congruente, rezult c el este paralelogram, deci
(1)
AT||MB, (AT)(MB)
Analog patrulaterul MCDB este paralelogram fiindc diagonalele lui se taie n
pri congruente. Atunci
(2)
MB||CD, (MB)(CD)
i
(3)
BD||MC, [BD][MC]
Din (1) i (2) obinem c patrulaterul ATDC este paralelogram, avnd 2 laturi
opuse paralele i congruente. Diagonalele lui se intersecteaz n O. Diagonalele
patrulaterului AMCE se taie n pri congruente i atunci el este paralelogram, deci
(4)
AE||MC, (AE)(MC)
A
T

E
H

G
O

K
D

Din relaiile (3) i (4) rezult c AE||BD i (AE)(BD), atunci ABDE este
paralelogram i BE intersecteaz pe AD n O. Obinem astfel c dreptele AD, BE, CT
sunt concurente n O.
R3.8.2 Pe diagonala (BD) a paralelogramului ABCD se consider punctele E
i M astfel nct (BE)(EM)(MD). Dac {T}=BCAE, {Q}=AMCD,
{L}=ABCE i {P}=ADCM, s se demonstreze c dreptele AC, EM i LQ sunt
concurente.
Demonstraie. Triunghiurile ADM i CBE sunt congruente avnd (AD)(BC),
(ADM)(CBE) (alterne interne), (DM)(BE). Din congruena acestor dou
triunghiuri obinem c
(1)
(AM)(CE)
i triunghiurile ADE i CBM sunt congruente avnd (AD)(BC),
(ADE)(CBM) i (DE)(BM). Din congruena lor obinem c
159

(AE)(CM)
(2)
Din relaiile (1) i (2) rezult c patrulaterul AECM, avnd laturile opuse
congruente este paralelogram, i deci (EM) trece prin mijlocul O al diagonalei (AC).
Patrulaterul ALCQ, avnd laturile opuse paralele este paralelogram (AQ||LC din faptul
c AECM este paralelogram). Deci diagonala (LQ) trece prin mijlocul O al diagonalei
(AC). Atunci dreptele AC, EM, LQ sunt concurente.
A

M
L

O
E
B

Dreptele care trebuie s demonstrm c sunt concurente sunt mediane,


bisectoare, nlimi, mediatoare, etc. pentru un anumit triunghi.
R3.8.3 Se consider triunghiul ABC dreptunghic n A i AD nlime,
D(BC). n C, pe cateta AC se ridic o perpendicular pe care se consider un segment
(CM)(AC), iar n B pe cateta AB se construiete o perpendicular pe care se ia un
segment BP cu (BP)(AB). S se demonstreze c dreptele BM, CP i AD sunt
concurente.
Soluie. Prelungim nlimea DA cu un segment (AR)(BC). Avem
m(RAC)=180=m(DAC)=180-[90-m(ACD)]=
=90+m(ACD)=m(BCM)
Atunci triunghiurile RAC i BCM sunt congruente fiindc au (RA)(BC),
m(RAC)m(BCM), (AC)(MC). Din congruena celor dou triunghiuri rezult c
(RCA)(CMB).
R

P
T

Fie {Q}=BMRC. Atunci


m(CQM)=180-[m(QCM)+m(CMQ)]=
=180-[m(QCM)+m(RCA)]=180-90=90
160

Deci BQRC, adic BQ este nlime n triunghiul RBC.


Analog artm congruena triunghiurilor RAB i CBP. Avem
m(RAB)=180-m(DAB)=180-(90-m(ABD))=
=90+m(ABD)=m(PBD)
Deci RABCBP avnd (RA)(BC), RABPBD i (AB)(BP).
Din congruena acestor triunghiuri obinem: (ABR)(BPC).
Fie {T}=PCAB. Avem
m(PTB)=180-[m(TPB)+m(PBT)]=
=180-[m(ABR)+m(PBT)]=180-90=90
Deci PTBR, adic CT este nlime n triunghiul RBC. Deci dreptele CT, BQ,
AD sunt concurente fiind nlimi n triunghiul RBC.
R3.8.4 Fie M un punct pe diagonala [BD] a ptratului ABCD. S se
demonstreze c perpendicularele duse din M, B, D respectiv pe AB, AM, CM se
ntlnesc pe diagonala [AC].
Soluie. Fie L i K picioarele perpendicularelor duse din B pe MA i respectiv
din M pe AB. n triunghiul MAB nlimile MK i BL se intersecteaz n P.
Diagonalele ptratului sunt perpendiculare, deci AO este nlime n triunghiul MAB,
deci perpendicularele PK i BL din M pe AB, respectiv B pe MA se ntlnesc pe
diagonala (AC). Fiindc MPAB, rezult c MPDC. Dar DMPC, i atunci M este
ortocentrul triunghiului DCP, rezult c CMDP. Astfel perpendicularele din M, B, D
respectiv pe AB, AM, CM se ntlnesc pe (AC).
D

C
M
T
O
L

3.9. Construcii geometrice

Prin probleme de construcie vom nelege acele probleme de geometrie n care


se cere construirea unor figuri geometrice ce satisfac anumite proprieti, folosind
numai rigla i compasul.
nainte de a considera probleme de construcie cu rigla i compasul, Edwin
Moise n lucrarea "Geometrie elementar" face cteva precizri:
i) Cnd vorbim de rigl i compas, nelegem o "rigl ideal" i un "compas
ideal", care traseaz liniile drepte i cercurile exact.
161

ii) Rigla nu are un marcaj pe ea. O putem utiliza pentru a desena drepte ntre
dou puncte date, dar aceasta este tot ceea ce putem face cu ea. Nu o putem utiliza
pentru a msura distanele dintre puncte sau pentru a vedea dac dou segmente sunt
congruente.
iii) Compasul se poate utiliza astfel. Fie un punct P i un punct Q n plan.
Putem desena atunci cercul cu centrul n P i care trece prin Q. Aceasta este tot ce
putem face cu el. Altfel spus, dndu-se un al treilea punct P' nu este permis s mutm
vrful compasului n P' i apoi s desenm un cerc cu centrul n P' i de raz PQ. Din
acest motiv compasul este numit nerigid; nu i se poate muta vrful deoarece "cnd
ridici vrful de pe hrtie, compasul se nchide".
S reamintim cteva dintre construciile elementare studiate i justificate la
clas.
3.9.1. Construcia mediatoarei unui segment i a mijlocului su

Considerm un segment [AB].


1) Cu o raz mai mare dect jumtate din lungimea segmentului [AB] desenm
dou arce de cerc cu centrul n A deoparte i de cealalt a dreptei AB.
2) Cu aceeai raz ca mai sus desenm dou arce de cerc cu centrul n B, de o
parte i de cealalt a dreptei AB, care intersecteaz arcele cu centrul n A n M i N.
3) Dreapta MN este mediatoarea segmentului [AB]. Punctul de intersecie P,
dintre AB i MN este mijlocul segmentului [AB].
M

P
A

3.9.2. Construcia bisectoarei unui unghi dat XOY

Considerm un punct O drept centru i desenm un arc de cerc care va


intersecta laturile unghiului n A i B. Cu aceeai raz mai mare dect

1
AB desenm
2

un arc de cerc cu centrul n A i unul cu centrul n B. Ce dou arce se intersecteaz n


M. Semidreapta [OM este bisectoarea unghiului XOY.

162

X
A
M

B
Y

3.9.3. Construcia perpendicularei pe o dreapt dintr-un punct al ei

Considerm punctul O pe dreapta a. Cu o deschidere oarecare a compasului se


construiesc dou arce ale unui cerc cu centrul n O i care intersecteaz dreapta a n A
i B. Perpendiculara cutat este mediatoarea segmentului AB.
3.9.4. Construcia perpendicularei pe o dreapt dintr-un punct exterior al ei

Considerm o dreapt a i un punct O nesituat pe ea. Desenm un arc de cerc


cu centrul n O i cu raza mai mare dect distana de la O la a, care intersecteaz pe a n
A i B. Mai construim un punct C egal deprtat de A i B i atunci OCa.

O
a
A

3.9.5. Construcia unui unghi congruent cu un unghi dat

Fiind dat unghiul XOY, trebuie s construim un unghi congruent cu el


X'O'Y'.
Cu centrele n O i O' desenm dou arce de cerc i obinem punctele A, B, C.
Descriem un arc de cerc cu centrul n C i raza AB. Obinem astfel punctul D. Latura
[O'Y' este semidreapta [O'D.

163

Y
Y'

Y
D

X' O'

X'

3.9.6. Construcia cercului circumscris triunghiul ABC

Se construiesc mediatoarele a dou laturi ale triunghiului. Intersecia lor, O, va


fi centrul cercului circumscris. Raza R va fi OA.
A

3.9.7. Construcia cercului nscris n triunghiul ABC

Se construiesc bisectoarele a dou unghiuri. Intersecia lor, I, va fi centrul


cercului nscris. Raza, r, va fi lungimea unei perpendiculare duse din I pe o latur.
A

I
r
B

3.9.8. Construcia centrului de greutate G

Construim mijloacele a dou laturi i apoi medianele corespunztoare.

164

3.9.9. Construcia ortocentrului H

Se construiesc perpendicularele din dou vrfuri ale triunghiului pe laturile


opuse.
3.9.10. Construcia tangentei la cercul C(O,R), ntr-un punct A al cercului

Se construiete perpendiculara CD pe raza OA.

3.9.11. Construcia tangentelor la cercul C(O,R) dintr-un punct exterior M

Se determin mijlocul segmentului [OM], O1, i apoi se construiete cercul de


diametru [OM]. Cercul astfel construit va intersecta cercului C(O,R) n A i B.
Tangentele cutate sunt MA i MB.
B

O1

3.9.12. Rezolvarea problemelor de construcii geometrice

n rezolvarea problemelor de construcii geometrice parcurgem urmtoarele


etape:
1) Analiza (gsirea soluiei). Se consider figura construit i identificm
elementele construibile folosind construciile elementare.
2) Construcia. Se prezint succesiunea elementelor care se construiesc.
165

3) Demonstraia. Conine argumentarea faptului c elementele construite


satisfac cerinele enunate n problem.
4) Discuia. Se comenteaz existena soluiilor i numrul lor.
Probleme rezolvate

R3.9.1 S se construiasc un triunghi dreptunghic cunoscnd suma lungimilor


catetelor (b+c) i lungimea ipotenuzei (a).
Demonstraie. Considerm problema rezolvat i fie BAC triunghiul cutat.
Construim triunghiul dreptunghic isoscel DAC cu (AC)(AD).
Analizm triunghiul BCD i deducem modul de construcie.
Construcia figurii
B

D
Pe segmentul DC=c+b (suma lungimilor catetelor), n punctul D, construim un
unghi de 45. Din B se duce un arc de cerc cu raza ct lungimea ipotenuzei a.
Triunghiul cerut este ABC.
Discuie. Dac arcul de cerc taie pe DF ntr-un singur punct avem soluie, dac
taie pe DF n dou puncte, avem dou soluii i dac nu-l taie n nici un punct nu avem
soluie.
F
C

45o
D

R3.9.2 S se construiasc un triunghi cunoscnd lungimile medianelor sale.


166

Demonstraie. Fie G1 simetricul centrului de greutate G al triunghiului ABC


fa de mijlocul A' al laturii [BC]. Atunci

1
2
GG1 = 2 GA' = 2 m a = m a
3
3
2
2
GC = m c , G 1C = GB = m b
3
3
A

G
B

A'
G1

Construim triunghiul GG1C de laturi

GG1 =

2
2
2
m a , GC = m c i G1C = m b
3
3
3

Simetricul lui G1 fa de G ne d vrful A. Simetricul lui C fa de mijlocul lui


[GG1] ne d vrful B.
Problema este posibil numai dac ma, mb, mc satisfac relaiile de condiie
dintre laturile unui triunghi.
R3.9.3 S se construiasc un triunghi dreptunghic cnd se cunosc nlimea i
mediana ce pleac din vrful unghiului drept.
Presupunem problema rezolvat. n triunghiul ABC cunoatem nlimea AD
i mediana AA'.
Construcia
A

A'

Construim triunghiul dreptunghic ADA' n care cunoatem cateta (AD) i


ipotenuza (AA'). Fiindc ntr-un triunghi dreptunghic mediana ce pleac din vrful
unghiului drept are lungimea egal cu jumtate din lungimea ipotenuzei, rezult c
167

putem construi segmentele (A'B) i (A'C) fiecare congruente cu AA', obinem astfel
vrfurile B i C i problema este rezolvat.
R3.9.4 S se construiasc un triunghi ABC cunoscnd mijloacele B', C' ale
laturilor [AC] i [AB] precum i piciorul D al nlimii care cade din vrful A pe [BC].
Demonstraie. Presupunem problema rezolvat i fie ABC triunghiul cutat. n
acest triunghi cunoatem punctul D piciorul nlimii din A, punctele B' i C' mijloacele
laturilor (AC) respectiv (AB).
A

B'

C'

[B'C'].

Analiznd figura deducem c A este simetricul lui D fa de linia mijlocie

Construcia
Unim C' cu B'. Construim punctul A ca simetricul lui D fa de [B'C']. Ducem
n D o perpendicular pe AD. Dreapta AC' ntlnete perpendiculara n B. Dreapta AB'
ntlnete perpendiculara n C. Triunghiul ABC astfel construit corespunde ipotezei.
3.9.13. Probleme de construcii imposibile ale antichitii

Problemele de construcie cu rigla i compasul au preocupat matematicienii


Greciei antice, care au dat soluii la unele, dar la altele, pentru a le putea aborda era
nevoie de ramuri ale matematicii pe care grecii nu le descoperiser.
Dintre problemele de construcii imposibile ale antichitii poate cele mai
celebre ar fi: problema triseciunii unghiului, duplicarea cubului, cuadratura cercului.
i) Triseciunea unghiului
Dndu-se un unghi BAC se cere s construim semidreptele AD i AE cu D,
E
situate
n
interiorul
unghiului
BAC
astfel
nct
s
avem:
m(BAD)=m(DAE)=m(EAC), folosind numai rigla i compasul.
B

A
E

168

Problema triseciunii unghiului este imposibil. (Exist unele unghiuri pentru


care semidreptele care realizeaz triseciunea nu pot fi construite. Sunt unele unghiuri
care pot fi mprite n trei pri congruente, exemplu unghiul drept.)
Problema triseciunii unghiului devine posibil dac schimbm puin ipoteza,
n sensul c permitem ca pe rigl s putem face dou gradaii.
ii) Duplicarea cubului
Fiind dat un segment AB, dorim s construim un segment CD astfel nct
cubul cu muchia CD s aib volumul exact de dou ori mai mare dect volumul
cubului cu muchia AB, adic CD3=2AB3.
iii) Cuadratura cercului
Dndu-se un cerc, dorim s construim un ptrat care s aib aceeai arie ca
cercul, adic l 2 = R 2 , adic l = R .
Problemele de mai sus au fost studiate peste dou mii de ani, descoperindu-se
n final c sunt imposibile. (Demonstraiile depesc cadrul de fa.)

Bibliografie
[1] I. I. Alexandrov, Probleme de construcii geometrice, Ed. Tehnic, 1951.
[2] G. Buicliu, Probleme de construcii geometrice cu rigla i compasul,
Ed. Tineretului, 1967.
[3] I. Dncil, Construcii cu rigla i compasul, Ed. Sigma, 2000.
[4] I. Petersen, Methodes et theories pour la resolution des problemes de
constructions geometriques. Paris, 1908.
[5] A. Toth, Noiuni de teoria construciilor geometrice, E.D.P., 1963.

169

S-ar putea să vă placă și